Вы находитесь на странице: 1из 106

1.

Hypofractionated stereotactic body radiation therapy (SBRT) of 60 Gy in 3 fractions is


most appropriate for a patient who has medically inoperable NSCLC with a:

A. 3-cm tumor involving the chest wall.


B. 3-cm tumor involving the right mainstem bronchus.
C. 2-cm tumor involving the right lower lobe with hilar adenopathy.
D. 2-cm tumor involving the carina.

Correct answer is A. RATIONALE: The ineligibility for RTOG stereotactic body radiation
therapy (SBRT) trial: Patients with T2 or T3 primary tumors of >5 cm or patients with T3 primary
tumors involving the central chest and structures of the mediastinum. The primary tumor of any
T-stage within or touching the zone of the proximal bronchial tree defined as a volume 2 cm in
all directions around the proximal bronchial tree (carina, right and left main bronchi, right and left
upper lobe bronchi, intermedius bronchus, right middle lobe bronchus, lingular bronchus, right
and left lower lobe bronchi). A higher rate of airway stenosis was reported in a prior dose-
escalating stud when 60 Gy was given in 3 fractions to tumors located in the central zone.

2. Which of the following types of radiation has the lowest LET?

A. 250-keV x-rays
B. 1.25-MeV γ-rays
C. 250-keV protons
D. 1-MeV carbon ions

Correct answer is B. RATIONALE: The high-energy photons have the lowest LET (i.e., they
set electrons in motion with a LET of 0.3-0.5 keV/µm), followed by low-energy photons (2.0-3.0
keV/µm) and low-energy protons (50-100 keV/µm). Carbon ions have the highest LET (~150
keV/µm). REFERENCES: Hall and Giaccia. Radiobiology for the Radiologist. 6th edition,
Chapter 7. Tannock, Hill, Bristow, and Harrington. The Basic Science of Oncology. 4th edition,
Chapter 14.

3. Which of the following statements about microsatellite instability in colorectal cancer is


FALSE?

A. It accounts for about 15% of sporadic colorectal cancers and most hereditary
nonpolyposis cancers.
B. It is associated with a mucinous histology and a surrounding lymphoid reaction.
C. These tumors tend to be more prevalent in the proximal colon.
D. These tumors are associated with worse survival than stage-matched microsatellite
stable tumors.

Correct answer is D. REFERENCES: Niv Y. Microsatellite instability and MLH1 promoter


hypermethylation in colorectal cancer. World Journal of Gastroenterology. Mar 28,
2007;13(12):1767-9. Lynch HT, de la Chapelle A. Genetic susceptibility to nonpolyposis
colorectal cancer. Journal of Medical Genetics. Nov 1999;36(11):801-18.
4. The quantity exposure can be described as the ionization equivalent of:

A. radiation kerma in air.


B. radiation kerma in tissue.
C. collision kerma in air.
D. collision kerma in tissue.

Correct answer is C. RATIONALE: According to its definition, exposure is a measure of


ionization produced in air by photons. When compared to kerma, this is equivalent to the
collisional component of kerma, which is the "kinetic energy released in a medium" when the
medium in this case is air. The radiation kerma is related to the energy loss due to radiative
losses (ie, bremsstrahlung).

5. Which of the following bones is most commonly involved in patients with osteosarcoma?

A. Humerus
B. Femur
C. Spine
D. Tibia

Correct answer is B. RATIONALE: The following bones are most commonly involved in
osteosarcoma: Femur = 40%. Humerus = 15%. Tibia = 15%. Spine = <5%. REFERENCE:
Halperin, et al. Pediatric Radiation Oncology.

6. What is the threshold dose for dysphagia from radiation therapy to the superior
constrictor muscles?

A. 75 Gy
B. 65 Gy
C. 55 Gy
D. 45 Gy

Correct answer is C. RATIONALE: The answer to this item is based on tolerance doses
established from MSKCC and multiple ASTRO 2007 abstracts.
7. Which of the following statements about the ECOG study evaluating the locoregional
failure rate in patients with breast cancer and positive lymph nodes 10 years after
undergoing mastectomy, adjuvant chemotherapy with or without tamoxifen, and no
irradiation is true?

A. The isolated locoregional failure rate was 21%.


B. The median number of positive lymph nodes was 3.
C. The median number of axillary lymph nodes examined was 8.
D. The locoregional recurrence rate was not affected by the number of axillary lymph
nodes examined.

Correct answer is B. RATIONALE: The rate of isolated locoregional recurrence was 13%. The
median number of positive lymph nodes was 3 (range 1 to 53). The median number of axillary
lymph nodes examined was 15 (range 2 to 63). The observed rate of locoregional recurrence
+/- distant failure (DF) varied by the number of nodes examined. When 2 to 5 lymph nodes were
examined, the locoregional failure (LRF) +/- DF was 27.2%; 6-10 examined was 22.2% and
20% for 11 or more lymph nodes examined. The LRF for patients with 1 to 3 positive lymph
nodes was 12.9%, and the LRF for patients with 4 or more positive lymph nodes was 28.7%.
Multivariate analysis showed that increasing tumor size, increasing numbers of involved nodes,
negative estrogen receptor status, and decreasing number of lymph nodes examined were
significant for increasing the rate of LRF +/- simultaneous DF. REFERENCE: Recht, et al.
Locoregional failure 10 years after mastectomy and adjuvant chemotherapy with or without
tamoxifen without irradiation: experience of the ECOG. Journal of Clinical Oncology(JCO).
1999;17(6):1689-1700.

8. Which of the following statements best describes why the single hit, multitarget survival
curve model was rejected in favor of the linear quadratic model?

Single-hit, multitarget model: S = 1 – (1 – e-D/D0)n


αD+βD2)
Linear-quadratic model: S = e-(

A. The single-hit, multitarget model did not provide a good fit to cell survival data for
high radiation doses.
B. The single-hit, multitarget model did not provide a good fit to cell survival data for
low radiation doses.
C. The linear-quadratic model is based on Poisson statistics, but the single-hit,
multitarget model was not.
D. The linear-quadratic model was more robust in terms of correcting for effects of
repopulation and cell-cycle redistribution.

Correct answer is B. RATIONALE: The single-hit, multitarget survival curve model was
ultimately rejected in favor of the linear-quadratic model because it did not account for the fact
that survival curves have non-zero initial slopes. Thus, it did not provide a good fit to cell
survival or tissue dose response data for the low radiation doses that are relevant to
fractionated radiation therapy.
9. Which of the following statements about staging and prognostic factors for primary
malignant bone tumors is true?

A. MRI is best for evaluating metastatic disease in the thorax.


B. MRI is best for staging of intraosseous or extraosseous tumors.
C. A tumor >8 cm in greatest dimension is classified as stage T3.
D. A worse prognosis is associated with lung metastases than with bone or hepatic
metastases.

Correct answer is B. RATIONALE: Option A: CT is the exam of choice for evaluating the
thorax for metastatic disease. CT is preferred over MRI for flat bones (such as the pelvis,
scapula, or posterior elements of the vertebrae) where characterization of a lesion by
radiography may be incomplete or difficult because of an inadequately visualized lesion. The
role of CT in these circumstances is to characterize the lesion and determine whether it is
potentially malignant or not. CT images may suffice for local staging. Option B: MRI provides the
most accurate depiction of intraosseous or extraosseous tumors. To improve imaging in
locations such as the pelvis or vertebrae, special sequences could be done. Bone scan is the
exam of choice to evaluate for multiple skeletal lesions. Option C: Stage T1 = tumor 8 cm or
less); stage T2 = tumor greater than 8 cm; stage T3 =discontinuous tumors in the primary bone
site. Option D: Bone and hepatic metastases have a worse prognosis than lung metastases.
REFERENCE: Greene FL, Page DL, Fleming ID, Fritz AG, eds. AJCC Cancer Staging Manual,
6th edition. 2002; pp. 187-190.

10. The fluence maps for an IMRT plan are:

A. generated by an optimization routine that minimizes the cost function.


B. correlated with the gantry speed function to obtain the dose volume histogram.
C. slid into the dose kernels for a sliding-window delivery.
D. flat to provide a uniform target dose.

Correct answer is A. RATIONALE: In inverse IMRT treatment planning, the planner defines the
orientation and energies of all beams, and dose constraints for both normal structures and the
target. A computer optimization algorithm is employed that attempts to meet these objectives by
minimizing the difference between the current dose distribution and the desired dose distribution
(cost function). Once the cost function is minimized, the optimization algorithm generates an
intensity fluence for each beam.

11. Which of the following trials demonstrated an improvement in survival for patients
receiving preoperative radiation therapy for rectal cancer?

A. German Rectal Cancer Study Group


B. Swedish Rectal Cancer
C. Dutch CKVO
D. NSABP R-01

Correct answer is B. RATIONALE: The only randomized trial that has shown a survival benefit
for preoperative radiotherapy was the Swedish Trial. The German and Dutch trials did not show
a survival benefit. NSABP R-01 evaluated postoperative treatment.
12. Which of the following two chemotherapy agents would be most appropriate to
administer concurrently with thoracic radiation therapy for patients with limited-stage
SCLC?

A. Cisplatin and etoposide


B. Cisplatin and irinotecan
C. Carboplatin and paclitaxel
D. Pemetrexed and carboplatin

Correct answer is A. RATIONALE: Cisplatin and etoposide remain the optimal regimen used
with thoracic radiation therapy in limited-stage SCLC. Other agents tested or adding a third
agent did not prove beneficial.

13. What is the most common presentation of bladder cancer in the United States?

A. Bone pain
B. Flank pain
C. Urinary obstruction
D. Painless hematuria

Correct answer is D. RATIONALE: Painless hematuria is the most common presentation of


bladder cancer.

14. The power of a statistical test is the probability that the null hypothesis is:

A. true.
B. false.
C. rejected when it is true.
D. rejected when it is false.

Correct answer is D. RATIONALE: This is the definition of statistical power.

15. Which of the following statements about radiation-induced locally multiply damaged sites
(LMDS) in DNA is FALSE?

A. Most DNA-damaging agents produce LMDS.


B. LMDS can consist of one or more closely spaced types of DNA damage.
C. LMDS can be inherently more difficult to repair.
D. High-LET radiation exposure causes an increase in both the number and complexity
of LMDS per unit dose.

Correct answer is A. RATIONALE: The random and discrete nature of energy deposition and
the resulting production of clustered lesions in DNA in some cases are largely unique to ionizing
radiation, compared to most other DNA damaging agents. REFERENCE: Hall and Giaccia.
Radiobiology for the Radiologist. 6th edition, Chapter 1 and (the beginning of) Chapter 2.
16. What is the FIGO stage for an adenocarcinoma of the uterus with 1.3-cm-depth invasion
of a 2.7-cm-thick myometrium and endocervical gland involvement but negative
peritoneal washings?

A. IB
B. IC
C. IIA
D. IIB

Correct answer is C. RATIONALE: According to FIGO staging criteria, <50% myometrial


invasion (1.3/2.7 = 48%) indicates stage IB, but endocervical gland involvement indicates stage
IIA.

17. What is the best time after surgery to initiate a 7-week course of conventional
fractionated radiation therapy for high-risk patients with advanced head and neck
cancer?

A. 04 weeks
B. 08 weeks
C. 10 weeks
D. 12 weeks

Correct answer is A. RATIONALE: This item is based on Ang's 2004 International Journal of
Radiation Oncology, Biology, Physics (IJROBP) paper showing that the optimal treatment
duration is 11 weeks (ie, 7-week radiation course delivered 4 weeks after surgery = 11 weeks)
for high-risk patients with advanced head and neck cancer.

18. Which of the following findings is most characteristic of Langerhans cell histiocytosis?

A. Psammoma body
B. Physaliphorous cell
C. Homer-Wright rosette
D. Birbeck granule

Correct answer is D. RATIONALE: Birbeck granules are characteristic of Langerhans cell


histiocytosis (LCH) and can be seen on electron microscopy. Psammoma bodies are collections
of calcium seen in a variety of different neoplasms; physaliphorous cells are seen with
chordoma; Homer-Wright rosettes are typically seen with neuroblastoma.

19. What happens to the angle in which isodose lines are tilted due to the presence of a
wedge in a photon beam as a function of depth?

A. Increases
B. Decreases
C. Can increase or decrease
D. Remains unchanged

Correct answer is B. RATIONALE: The presence of scattered radiation decreases with


increasing depth in a phantom, thus causing the angle in which isodose lines are tilted to
decrease.
20. Which of the following statements about adenocarcinoma of the anus is FALSE?

A. It typically arises within the anal mucosa or anal fistulas.


B. It represents approximately 5% to 10% of all cases of anal cancer.
C. It presents with more advanced disease than epidermoid carcinomas of the
anal canal.
D. Definitive chemoradiation results in poor local control and high distant failure rates.

Correct answer: ABCD. REFERENCES: Papagikos M, Crane CH, Skibber J, Janjan NA, Feig
B, Rodriguez-Bigas MA, Hung A, Wolff RA, Delclos M, Lin E, Cleary K. Chemoradiation for
adenocarcinoma of the anus. International Journal of Radiation Oncology, Biology, Physics (Int
J Radiat Oncol Biol Phys). Mar 1, 2003;55(3):669-78. Basik M, Rodriguez-Bigas MA, Penetrante
R, Petrelli NJ. Prognosis and recurrence patterns of anal adenocarcinoma. American Journal of
Surgery. Feb 1995;169(2):233-7.

21. Which of the following organs is believed to contain serially arranged FSUs?

A. Spinal cord
B. Kidney
C. Liver
D. Lung

Correct answer is A. RATIONALE: Withers' functional subunit model proposes that the
irradiation volume dependence of normal tissue tolerance depends in part on the structural and
functional organization of tissues into functional subunits (FSUs). For some tissues, these FSUs
are believed to be arranged "in series," such that the inactivation of a single unit could
compromise the entire organ's function. The spinal cord is thought to contain serially arranged
FSUs. For other tissues, the FSUs operate essentially independent of each other and are said
to be arranged "in parallel." Such tissues, thought to include the liver, lung, and kidney, have
large functional reserves and can tolerate very high radiation doses to small/medium treatment
volumes. REFERENCE: Hall and Giaccia. Radiobiology for the Radiologist. 6th edition, Chapter
19.

22. Which of the following locations of male urethral carcinoma has the worst prognosis?

A. Distal
B. Prostatic
C. Bulbomembranous
D. Location is not prognostic.

Correct answer: ABCD. RATIONALE: Understanding basic epidemiologic information and


common presentation characteristics will help the clinician determine the appropriate course of
action in this relatively infrequent diagnosis. Patients with bulbomembranous involvement
usually have advanced disease at presentation.
23. Which of the following treatments should be used as first-line therapy for gastric MALT
lymphoma to achieve complete tumor remission in the majority of patients?

A. Focal radiation field therapy after initial biopsy


B. Combined chemotherapy and radiation therapy
C. Antibiotic therapy for Helicobacter pylori
D. Surgical resection combined with chemotherapy

Correct answer is C. RATIONALE: Gastric mucosa-associated lymphoid tissue (MALT)


lymphoma is associated with Helicobacter pylori. More than two thirds of patients with this
disease respond to antibiotic therapy, which is considered to be first-line therapy.

24. Which of the following statements about pure bronchioloalveolar carcinoma is true?

A. It is not associated with the expression of TTF-1 and CK7.


B. The EGFR gene mutation in patients with lung cancer is linked to bronchioloalveolar
differentiation.
C. It requires the presence of lymphatic space invasion.
D. Stromal invasion is commonly associated.

Correct answer is B. RATIONALE: Pure bronchioloalveolar carcinoma (BAC) requires absence


of invasion of stroma, pleura, and lymphatic spaces. Nonmucinous BAC expresses TTF-1, CK7,
and lacks CK20.

25. Which of the following sites is most commonly associated with HPV-induced squamous
cell carcinoma?

A. Hypopharynx
B. Oropharynx
C. Nasopharynx
D. Larynx

Correct answer is B.
Site of cancer % of +HPV-16 DNA
Hypopharynx 0%
Oropharynx 50%
Nasopharynx 14%
Larynx 3%

REFERENCE: Jon Mork, et al. New England Journal of Medicine (NEJM). 2001;344,15:1125-
31.
26. According to AAPM TG 51, a thin lead sheet should be used in the calibration of an
x-ray beam to determine the depth dose curve for all photon energies greater than:

A. 04 MV.
B. 06 MV.
C. 10 MV.
D. 15 MV.

Correct answer is C. RATIONALE: According to AAPM TG 51, for photon beams about 10 MV
and above, electron contamination from the accelerator head may significantly affect the dose at
dmax. To reduce the effects of these contaminant electrons to negligible levels, this task group
recommends the use of a thin lead sheet when measuring the percent depth dose curve for
these higher-energy photon beams.

27. In mammalian cells, which of the following groups of proteins is involved in


nonhomologous end-joining repair of DNA double-strand breaks?

A. Ku80, Ligase III, RPA


B. XPC, XPE, CSA
C. Artemis, Ku70, Ligase IV
D. DNA PKcs, Ku70, RPA

Correct answer is C. RATIONALE: It is important to know which proteins participate in


nonhomologous end-joining (NHEJ), since, when defective, one or more of these proteins could
increase the cellular radiosensitivity of patients treated with radiation therapy, possibly
culminating in an increased risk of normal tissue complications. Artemis (a complex of Rad50,
MRE11 and NBS1, responsible for the processing of breaks), Ku70, and Ligase IV are all
involved in NHEJ. Ligase III, XPC, XPE, and RPA are not involved. REFERENCE: Tannock,
Hill, Bristow, and Harrington. The Basic Science of Oncology. 4th edition, Chapter 5.

28. Which of the following statements does NOT correctly describe a characteristic of
HNPCC?

A. It is also known as Lynch syndrome.


B. It is the most common hereditary colorectal cancer syndrome.
C. Current diagnostic criteria include the Amsterdam criteria.
D. Promoter hypomethylation accounts for some cases of microsatellite instability.

Correct answer is D. REFERENCES: Hendriks YM, de Jong AE, Morreau H, Tops CM, Vasen
HF, Wijnen JT, Breuning MH, Bröcker-Vriends AH. Diagnostic approach and management of
Lynch syndrome (hereditary nonpolyposis colorectal carcinoma): a guide for clinicians. CA: A
Cancer Journal for Clinicians (CA: Cancer J Clin). July-August 2006;56(4):213-25.
29. Which of the following FIGO stages is most commonly associated with a lower one-third
vaginal cancer that has invaded the subvaginal and parametrial tissues but not the
pelvic side wall?

A. I
B. II
C. III
D. IV

Correct answer: ABCD. RATIONALE: Stage II is the most common FIGO presentation of
vaginal cancer (in the pre-MRI era), but up to 50% of patients may have III+ disease in the MRI
era. The proposed FIGO subdivision of IIA (subvaginal) and IIB (parametrial) is often used by
Perez. REFERENCE: Taylor. Clinical Radiology. 2007;62:549-555.

30. What is the expected 5-year overall survival rate in a patient with muscle-invasive
bladder cancer treated with TURBT, followed by combined chemotherapy and radiation
therapy?

A. 80%
B. 65%
C. 50%
D. 25%

Correct answer is C. RATIONALE: According to most trials, the expected 5-year overall
survival rate is approximately 50% for patients with muscle-invasive bladder cancer treated with
TURBT, followed by combined chemotherapy and radiation therapy. This is similar to that of a
radical cystectomy.

31. Which of the following stages of oropharyngeal cancer can be adequately treated with
unilateral therapy?

A. T1N1 of the tonsil


B. T1N2a of the tonsil
C. Lateralized T2N0 of the base of tongue
D. T3N0 of the tonsil

Correct answer is A. RATIONALE: Based on Princess Margaret Hospital experience reported


by O' Sullivan. REFERENCE: International Journal of Radiation Oncology, Biology, Physics (Int.
J. Radiation Oncology Biol. Phys.). 2001;51(2):332–343.
32. Which of the following Ann Arbor stages would be most appropriate for a 15-year-old
boy who has Hodgkin lymphoma with clinical and radiographic evidence of left neck and
left supraclavicular fossa lymphadenopathy, generalized pruritus, and a 7-lb weight loss
over the past 6 months?

A. Stage IA
B. Stage IB
C. Stage IIA
D. Stage IIB

Correct answer is A. RATIONALE: The neck and supraclavicular fossa constitute one region
for lymphoma staging; therefore, this patient would have stage I disease. B symptoms include
the following: 1) Loss of >10% weight loss over 6 months, 2) fever, 3) night sweats. Pruritus is
not considered a B symptom.

33. Compared to 3D conformal radiation therapy, an image-guided radiotherapy (IGRT)


program requires:

A. a higher dose rate.


B. less immobilization.
C. more quality assurance.
D. daily CT imaging.

Correct answer is C. RATIONALE: Image-guided radiotherapy (IGRT) requires more quality


assurance (QA). Imaging systems, as well as integration between imaging and treatment
systems, need to undergo more QA in addition to performing the standard QA.

34. Which of the following secondary cancers is most likely to be induced by prior radiation
therapy?

A. Ovarian cancer 11 years after breast-conserving therapy


B. Angiosarcoma of the left breast 12 years after breast-conserving therapy
C. Leukemia 2 years after chemoradiotherapy for lymphoma
D. Adenocarcinoma of the lung in a patient who smokes and was treated for SCLC
4 years earlier

Correct answer is B. RATIONALE: Radiation-induced solid tumors typically occur in or near


the previously irradiated treatment field(s) between 10 and 50 years after radiation therapy for
the previous cancer.
35. Which of the following recursive partitioning analysis (RPA) classifications and primary
disease sites had an overall survival benefit based on the phase III RTOG 9508
(Andrews) trial that evaluated WBRT with and without stereotactic radiosurgery?

RPA Primary Disease Site


A. Class I Lung cancer
B. Class III Lung cancer
C. Class I Breast cancer
D. Class III Breast cancer

Correct answer is A. RATIONALE: Based on the subgroup analysis of RPA classification and
primary disease site, patients who had lung cancer and were in the RPA class I group
experienced an overall survival benefit with the addition of stereotactic radiosurgery (SRS) to
WBRT. Otherwise, no detectable overall survival advantage was demonstrated with the addition
of SRS to WBRT.

36. In the Norwegian (Aalders) adjuvant endometrial trial, which subset of patients may have
improved overall survival for pelvic radiation after vaginal brachytherapy?

A. IA, grade 2
B. IB, grade 3
C. IC, grade 3
D. IIA, grade 3

Correct answer is C. RATIONALE: The Norwegian (Aalders, et al) trial only included patients
with stage I disease. For all patients taken together, there was no overall survival benefit seen
with pelvic irradiation. Patients with stage IC, grade 3 had a better overall survival (82% vs 72%)
and local control (95% vs 80%) and the same distant metastasis (15%) on subset analysis.
Though the number of patients with stage IC, grade 3 (approximately 10%) was too few for
statistical significance for overall survival, the analysis suggests a benefit for this subgroup.
REFERENCE: Aalders J, Abeler V, Kolstad P, et al. Postoperative external irradiation and
prognostic parameters in stage I endometrial carcinoma: Clinical and histologic study of 540
patients. Obstetrics & Gynecology. 1980;56:419-427.

37. Which of the following imaging studies is most useful in the local evaluation of solitary
plasmacytoma of the bone?

A. MRI
B. CT scan
C. Bone scan
D. PET scan

Correct answer is A. RATIONALE: It is important to evaluate the extent of the lesion


appropriately. MRI provides the most complete anatomic information.
38. Which of the following statements about the profile of a 20-MeV electron beam is true?

A. The 20% and 50% isodose lines laterally constrict, whereas the 80% and 90%
isodose lines bulge out near the field edge.
B. The 20% and 50% isodose lines bulge out near the field edge, whereas the 80%
and 90% isodose lines laterally constrict.
C. All isodose lines demonstrate a lateral constriction near the field edge.
D. All isodose lines bulge out near the field edge.

Correct answer is B. RATIONALE: For high-energy electrons, the low isodose levels bulge out,
whereas the high isodose levels laterally constrict. This effect is accentuated with decreasing
field size.

39. Which of the following statements about the extent of lymph node dissection in patients
with gastric cancer is true?

A. A D2 dissection includes a splenectomy.


B. A D2 dissection did not improve survival when compared to a D1 dissection,
according to a Dutch study.
C. The majority of patients received a D1 dissection in the INT-0116 randomized study
evaluating postoperative chemoradiation.
D. Chemoradiation should not be given to patients after a D2 dissection.

Correct answer is B. RATIONALE: A splenectomy is associated with higher complications.


The Dutch study did not show a survival benefit when comparing D1 vs. D2 dissections. The
majority of patients in the INT-0116 study had a D0 dissection. One large study from Korea
suggests a 5% overall survival benefit with chemoradiation, even with a D2 dissection.
REFERENCE: Journal of Clinical Oncology (JCO) 22:2069-2077. International Journal of
Radiation Oncology, Biology, Physics (Int. J. Radiation Oncology Biol. Phys.). 2005;63(5):1279–
1285.

40. Which of the following types of mutation presents the LEAST genetic risk to the
first-generation offspring of parents who were irradiated?

A. Autosomal-recessive
B. Autosomal-dominant
C. Mitochondrial
D. Y-linked

Correct answer is A. RATIONALE: Since a recessive mutation must be inherited from both
parents, the chances of such an induced mutation affecting the first generation of offspring
would be exceedingly small.
41. Which of the following treatment plans is best for a patient with stage T1 squamous cell
carcinoma of the glottic larynx?

A. 79.2 Gy in 66 fractions of 1.2 Gy administered twice daily


B. 63 Gy in 28 fractions of 2.25 Gy
C. 63 Gy in 35 fractions of 1.8 Gy
D. 60 Gy in 30 fractions of 2.0 Gy, plus cisplatin

Correct answer is B. RATIONALE: Choice B is the current "gold standard." Choice A has been
shown to be inferior. Choice C is acceptable for stage T2, but not stage T1. Choice D is much
too aggressive.

42. What is the most common subtype of classical Hodgkin lymphoma?

A. Large B-cell
B. Mixed cellularity
C. Nodular sclerosing
D. Lymphocyte depleted

Correct answer is C. RATIONALE: Nodular-sclerosing Hodgkin lymphoma is the most


common subtype of classic Hodgkin lymphoma, accounting for more than two thirds of all
cases. Mixed-cellularity Hodgkin lymphoma is the next most common subtype at 30%, followed
by lymphocyte-rich and lymphocyte-depleted Hodgkin lymphoma.

43. The null hypothesis that the hazard ratio equals 1 can be tested using which of the
following regression analysis methods?

A. Cox
B. Logistic
C. Polynomial
D. Multiple linear

Correct answer is A. RATIONALE: Cox regression models are focused on hazard ratios,
whereas the other regression model options provide information about odds ratios or mean
values.
44. Which of the following radiobiological processes explains the increased radiosensitivity
of exponentially growing HeLa cells in culture as the dose rate is lowered from 154 to
37 cGy/h?

A. The cells become less hypoxic.


B. The cells repair DNA damage more efficiently.
C. The cells redistribute into the G2 phase of the cell cycle.
D. The chromatin becomes more condensed, enhancing DNA damage.

Correct answer is C. RATIONALE: The inverse dose rate effect is manifest as an increase in
cell killing with decreasing dose rate (of low-LET radiation), the opposite of what would
otherwise be expected (i.e., a decrease in cell killing with decreasing dose rate). This
phenomenon has been observed when exponentially growing cells in culture are irradiated with
a series of decreasing dose rates. There appears to be a cell line-specific, “critical” dose rate at
which the change from decreasing radiosensitivity to increasing radiosensitivity occurs. The
inverse dose rate occurs because, at or near the critical dose rate, further cell division is
inhibited and instead, cells begin to accumulate at the G2 phase cell cycle checkpoint. Because
G2 phase cells are more radiosensitive that G1 and S phases cells, this cell cycle redistribution
effect results in a net sensitization of the population as a whole. REFERENCE: Hall and
Giaccia. Radiobiology for the Radiologist. 6th edition, Chapter 5.

45. The internal target volume (ITV) is defined as the:

A. gross tumor volume plus a margin for microscopic growth.


B. clinical target volume plus a margin for internal motion.
C. planning target volume plus a margin for setup error.
D. organ at risk volume plus a margin for setup error and internal motion.

Correct answer is B. RATIONALE: ICRU 50 and 62 have established the definitions for a
series of volumes that are used in radiation therapy. Option A is the clinical target volume
(CTV), and option D is the planning organ at risk volume (PRV). The ITV is defined by option B.

46. The most common cancer in the renal pelvis is:

A. transitional cell carcinoma.


B. squamous cell carcinoma.
C. clear cell carcinoma.
D. adenocarcinoma.

Correct answer is A. RATIONALE: Most (90%) upper urinary tract (including renal pelvis)
tumors are transitional cell in origin.

47. Which of the following tests would most reliably confirm the cause of MRI enhancement
in the treatment field 6 months after a patient with glioblastoma has completed
chemoradiation?

A. 18FDG PET scan


B. MR spectroscopy
C. Diffusion-weighted MRI
D. Biopsy

Correct answer is D. RATIONALE: Although all of the tests may suggest tumor progression
versus treatment effect, none replace a stereotactic or open biopsy.
48. Which of the following sites should be included in radiation therapy for a stage T3
squamous cell carcinoma of the hypopharynx?

A. Superior retropharyngeal lymph node (Rouvière node)


B. Pterygomandibular raphe
C. Level VI lymph nodes
D. Meckel's cave

Correct answer is A. RATIONALE: Carcinomas of the hypopharynx drain into the


retropharyngeal nodes, and they should be covered to the base of the skull. Meckel's cave
should be covered when treating cancer with perineural invasion of the trigeminal nerve.

49. What is the cumulative incidence of secondary cancers in patients who receive EBRT for
hereditary retinoblastoma?

A. 00% to 5%
B. 10% to 15%
C. 35% to 40%
D. 70% to 75%

Correct answer is C. RATIONALE: Patients with hereditary retinoblastoma who received


external-beam radiation therapy (EBRT) have a 35% cumulative incidence of secondary
cancers. A child with unilateral retinoblastoma who presents with advanced intraocular disease
and undergoes enucleation has a cure rate of >95%. REFERENCE: Chintagumpala, et al. The
Oncologist. October 2007;12(10):1237-1246.

50. A 5-year-old boy who has pure germinoma with normal blood and CSF β−hCG and
α−fetoprotein levels has a complete response to subtotal resection and four cycles of
systemic chemotherapy. Which of the following treatment recommendations would be
most appropriate?

A. Craniospinal radiation therapy with a boost dose to the primary site


B. Local radiation therapy
C. Whole-brain radiation therapy
D. Stereotactic radiosurgery

Correct answer is B. RATIONALE: Local radiation therapy is recommended for a 5-year-old


patient who has had a good response to chemotherapy. The appropriate radiation treatment in
patients with CNS germ cell tumors has evolved from craniospinal radiation therapy to local
fields in the era of chemotherapy. Withholding radiation therapy is associated with an
unacceptable risk of recurrence.

51. Homologous recombination involves repair of which of the following DNA lesions?

A. Base damage
B. Single-strand breaks
C. Double-strand breaks
D. DNA-protein crosslinks

Correct answer is C. RATIONALE: Homologous recombination (HR) and nonhomologous end-


joining (NHEJ) are involved in DNA double-strand break repair, with each type predominating in
different parts of the cell cycle. (NHEJ in G1 phase; HR in S/G2 phases.) REFERENCE: Hall and
Giaccia. Radiobiology for the Radiologist. 6th edition, Chapter 5.
52. Disregarding the activity excreted via the urine, what is the approximate activity of
555 MBq (15 mCi) of 18FDG (T1/2 = 110 minutes) 5.5 hours after administration to a
patient who is to undergo PET imaging?

A. 017.3 MBq (0.47 mCi)


B. 034.7 MBq (0.94 mCi)
C. 069.4 MBq (1.88 mCi)
D. 138.8 MBq (3.75 mCi)

Correct answer is C. RATIONALE: This item requires knowledge that T1/2 represents a decay
of one half of the original activity: 5.5 h (330 m) is 3 half-lives (e.g., 0.5 x 0.5 x 0.5 = 0.125;
hence, 555 MBq x 0.125 = 69.4 MBq).

53. What is the expected progression-free survival rate after hypofractionated stereotactic
body radiation therapy (SBRT) for stage IA NSCLC?

A. 30%
B. 50%
C. 70%
D. 90%

Correct answer is D. RATIONALE: Several studies, some of which were prospective


(nonrandomized), have shown the local control rate to be 90% or higher for stage IA NSCLC
treated with hypofractionated SBRT. REFERENCE: Fowler JF, Tome WA, Welsh JS. Estimation
of the required doses in stereotactic body radiation therapy. Kavanagh BD and Timmerman RD,
eds. Stereotactic Body Radiation Therapy. Lippincott, Williams & Wilkins; 2005.

54. What percent of children with acute lymphoblastic leukemia have CNS involvement at
diagnosis?

A. 01% to 2%
B. 03% to 5%
C. 07% to 10%
D. 12% to 15%

Correct answer: ABCD. RATIONALE: CNS leukemia is present at diagnosis in 3% to 5% of


children with acute lymphoblastic leukemia (ALL). REFERENCE: Pediatric Radiation Oncology.
3rd edition. Lippincott, Williams & Wilkins.

55. A whole-organ x-ray dose of 8 Gy delivered in 2-Gy fractions is most likely to cause
permanent dysfunction in the:

A. liver.
B. brain.
C. spinal cord.
D. testes.

Correct answer is D. RATIONALE: A total dose of 6 to 8 Gy will cause permanent


azoospermia in almost all males. That same dose would cause little or no toxicity in the other
organs listed. REFERENCE: Mettler and Upton. Medical Effects of Ionizing Radiation. 3rd
edition, Chapter 1 (Table 1-8) and Chapter 6.
56. What is the maximum dose to an organ located 2 cm outside of a 6-MV treatment field
relative to the central axis dose at the same depth?

A. 00.2%
B. 02.0%
C. 10%
D. 15%

Correct answer is B. RATIONALE: Within 2 cm from the edge of the field, internal scatter
contributes dose outside of the field, though the values are still fairly low.

57. Which of the following radiation therapy methods for dose escalation has NOT been
tested in a randomized phase III trial for treatment of malignant glioma?

A. Radiosurgery
B. Brachytherapy
C. Fast neutron
D. Proton

Correct answer is D. RATIONALE: Proton radiation therapy for dose escalation has not been
tested in a randomized trial. Multi-institutional randomized trials have been performed for
radiosurgery, brachytherapy, and fast neutrons without showing a survival benefit.

58. What is the mechanism of action of irinotecan?

A. Topoisomerase I inhibition
B. Topoisomerase II inhibition
C. DNA binding and intercalation
D. DNA crosslinking

Correct answer is A. RATIONALE: Irinotecan is a camptothecin analogue that functions as a


topoisomerase-I inhibitor. Irinotecan is used to treat numerous malignancies, including
colorectal cancer and lung cancer, and is being investigated to treat other diseases, both alone
and in combination with other agents. REFERENCE: CPT-11. Camptosar. Pfizer Inc; New York,
NY (PACKAGE INSERT).

59. Which of the following statements about brain metastases secondary to breast cancer is
true?

A. The overall incidence is 15%.


B. The most common symptoms are acute onset of headaches, nausea, and vomiting.
C. Lung, liver, or lymph node metastases predict a higher risk for development of brain
metastases.
D. Age does not affect the risk for development of brain metastases.

Correct answer is C. RATIONALE: The incidence of brain metastases is 5%. The most
common finding associated with the onset of brain metastases is a slowly progressive
neurological deficit. Seizures and acute stroke or TIA-like symptoms also can be seen. A higher
risk of brain metastases is seen in patients who are under 40 years of age; have HER-2-positive
disease; and have ER-negative disease. REFERENCE: Kaal ECA and Vecht C. CNS
complications of breast cancer: current and emerging treatment options. CNS Drugs.
2007;21(7):559-579.
60. Which of the following chromosomal translocations is most commonly associated with
chronic myelogenous leukemia (CML)?

A. t(8;21)
B. t(9;22)
C. t(12;15)
D. t(15;17)

Correct answer is B. RATIONALE: It is important to recognize the proper diagnosis of chronic


myelogenous leukemia (CML). The relationship of CML to chromosomal translocation t(9;22) is
well documented.

61. Overexpression of p16 in HPV-associated oropharyngeal squamous cell carcinoma is an


independent prognostic factor for:

A. lower disease-free survival.


B. higher overall survival.
C. higher local recurrence.
D. higher response rate to EGFR-targeted therapy.

Correct answer is B. RATIONALE: In patients with oropharyngeal squamous cell carcinoma,


overexpression of p16 (a subtype of HPV) has been shown to be an independent prognostic
factor for local recurrence, disease-free survival, and overall survival. REFERENCE:
Weinberger, et al. Clinical Cancer Research. September 1, 2004;10:5684-5691.

62. Plateau-phase cells in culture received 4-Gy x-ray irradiation and then were exposed to
either saline alone (control) or saline plus a nontoxic new drug for 2 hours postirradiation
before being subcultured and assayed for clonogenic survival. Which of the following
would explain the observation of a higher surviving fraction in the control cells than in the
drug-treated cells at the end of the experiment?

A. Fixation of PLD in the drug-treated cells


B. Fixation of SLD in the drug-treated cells
C. Fixation of PLD in the control cells
D. Fixation of SLD in the control cells

Correct answer is A. RATIONALE: Any postirradiation treatment that has the net effect of
reducing plateau phase cell survival (in the absence of any other direct toxicity) is said to have
“fixed” (i.e., made permanent) potentially lethal radiation damage. Based on the operational
definitions of both sublethal damage (SLD) and potentially lethal damage (PLD), SLD and its
repair cannot be demonstrated unless at least two radiation doses are given with a radiation-
free interval between them. REFERENCE: Hall and Giaccia. Radiobiology for the Radiologist.
6th edition, Chapter 5.
63. According to the NRC, personnel dosimeters are required for:

A. staff members of a department who use radiation-producing devices.


B. staff members of a department who could be in rooms with radiation-producing
devices.
C. radiation workers likely to receive a whole-body dose in excess of 5 mSv/year.
D. radiation workers likely to receive a whole-body dose in excess of 50 mSv/year.

Correct answer is C. RATIONALE: According to 10 CFR 20.1502, personnel monitoring are


required for all employees likely to receive a dose in excess of 10% of the annual NRC limit (50
mSv to the whole body).

64. Which of the following treatments would be most appropriate for an 8-year-old boy with
nondisseminated unifocal pure germinoma of the pineal area?

A. Four cycles of etoposide and cisplatin alone


B. Radiation therapy to the primary tumor alone
C. Whole-ventricular radiation therapy, followed by a boost dose to the primary tumor
D. Complete surgical resection, followed by radiation therapy to the primary tumor

Correct answer is C. RATIONALE: Involved-field radiation therapy alone is regarded as


inadequate therapy for focal CNS germinoma with or without complete surgical resection.
Omission of consolidative radiation therapy (RT) after chemotherapy will result in a high risk of
disease recurrence. Although the treatment volume for focal CNS germinoma has been
vigorously debated, a recent systematic review from Lancet Oncology (2005) by Rogers, et al.
showed that whole-brain or whole-ventricular RT followed by a boost yielded identical rates of
local control, neuraxis control, and survival compared to craniospinal RT followed by a boost in
adequately staged patients. REFERENCE: Rogers SJ, Mosleh-Shirazi MA, and Saran FH.
Radiotherapy of localized intracranial germinoma: time to sever historical ties? Lancet
Oncology. 2005;6(7):509-519.

65. A false-positive occurs when test results are:

A. positive for the presence of disease but no disease is present.


B. negative for the presence of disease but the disease is present.
C. positive for the presence of disease and the disease is present.
D. negative for the presence of disease and no disease is present.

Correct answer is A. RATIONALE: False-positive results occur when the results are positive
for the presence of disease, but the patient does not have the disease.
66. Which of the following statements about patients who have breast cancer with ipsilateral
supraclavicular metastases and no other more distant disease at diagnosis is true?

A. The AJCC stage is IIIB.


B. They are not eligible for breast-conserving therapy.
C. The radiation dose to the supraclavicular lymph nodes is 50 to 66 Gy.
D. The supraclavicular fossa is managed by neoadjuvant chemotherapy,
supraclavicular lymph node dissection, and radiation therapy.

Correct answer is C. RATIONALE: The AJCC stage for ipsilateral supraclavicular lymph nodes
is IIIC. The study did not exclude patients from breast conservation, gave 50 to 66 Gy (median
58 Gy) to the supraclavicular fossa, and did not perform supraclavicular dissections.
REFERENCES: AJCC Cancer Staging Manual, 6th edition. Huang, et al. International Journal of
Radiation Oncology, Biology, Physics (Int J Radiat Oncol Biol Phys). 2007;67:490-6.

67. Bleomycin is more toxic to oxygenated cells than to hypoxic cells due to:

A. rapid proliferation of the oxygenated cells.


B. poor penetration of the drug into hypoxic cells.
C. increased repair of drug damage in hypoxic cells.
D. increased damage fixation in the presence of oxygen.

Correct answer is D. RATIONALE: Bleomycin has a unique mechanism of action different from
many other chemotherapy agents, and one that makes the drug more "radiation-like." The
bleomycin molecule contains a group of five nitrogen atoms arranged in a square-pyramidal
conformation that binds divalent iron as the active ligand. Molecular oxygen, bound by the iron,
produces highly reactive, highly localized hydroxyl radicals that cause DNA damage (not unlike
a radiation-induced locally multiply damaged site or LMDS). Because of this, bleomycin is more
toxic to aerobic cells than to hypoxic ones. REFERENCE: Pratt, Ruddon, Ensminger, and
Maybaum. Chapter 7: Noncovalent DNA-binding drugs. The Anticancer Drugs. 2nd edition.
1994.

68. Which of the following characteristics is associated with the majority of patients who
have follicular lymphoma?

A. It predominantly affects young adults.


B. The Bcl-2 gene rearrangement is typically present.
C. Bone marrow involvement is uncommon.
D. Without therapy, the median survival of patients is less than 5 years.

Correct answer is B. RATIONALE: The majority of patients with follicular lymphoma present
with widespread disease and bone marrow involvement. The disease most commonly affects
elderly patients. Due to its very indolent nature, even with observation, the median survival of
these patients exceeds 5 years. The t(14;18) and Bcl-2 gene rearrangement is present in the
majority of patients with follicular lymphoma.
69. An accelerating waveguide in a megavoltage linear accelerator is typically made of:

A. aluminum.
B. copper.
C. plastic.
D. acrylic.

Correct answer is B. RATIONALE: An accelerating waveguide in a megavoltage linear


accelerator is typically made of a material that is a good conductor.

70. Which of the following factors is associated with the highest risk for the development of
breast cancer?

A. Estrogen replacement therapy


B. Estrogen and progestin replacement therapy
C. Age of greater than 40 years
D. Postmenopausal obesity

Correct answer is C. RATIONALE: Age is the strongest risk factor for the development of
breast cancer. Breast cancer is rarely seen before 20 years of age and rises sharply between
40 and 80 years of age. REFERENCE: Harris, Lippmann, Morrow, and Osborne. Diseases of
the Breast. 4th edition. 2004;223-276.

71. The administration of concurrent cisplatin-based chemotherapy with radiation therapy for
head and neck cancer has been shown to decrease:

A. distant metastases.
B. locoregional recurrence.
C. acute toxicity.
D. late toxicity.

Correct answer is B. RATIONALE: All trials supporting chemoradiotherapy show an


improvement in locoregional control. Very few also show an effect on distant disease. Toxicity is
greater with concurrent chemotherapy and radiation therapy, and there is no logic to decreasing
the dose of radiation therapy since it is the most effective "drug" we have.

72. Which of the following sets of unfavorable prognostic criteria is used by the EORTC for
patients with early-stage Hodgkin lymphoma?

A. "B" symptoms, age, pathology, extranodal sites, and number of involved sites
B. "B" symptoms, age, erythrocyte sedimentation rate (ESR), mediastinal bulk, and
number of involved sites
C. "B" symptoms, pathology, ESR, mediastinal bulk, and number of lymph node sites
D. Gender, ESR, mediastinal bulk, number of sites, and extranodal sites

Correct answer is B. RATIONALE: The EORTC used the following criteria for assigning
patients with early-stage Hodgkin lymphoma to the favorable vs. unfavorable prognostic group:
"B" symptoms, age, erythrocyte sedimentation rate (ESR), mediastinal bulk, and the number of
nodal sites.
73. Which of the following is a kinase responsible for phosphorylating downstream proteins
in response to DNA damage?

A. BRCA1
B. BRCA2
C. p53
D. ATM

Correct answer is D. RATIONALE: Although ATM, BRCA1, BRCA2, and p53 are all important
proteins that participate in the DNA damage response after irradiation, ATM is the only one that
also has kinase activity essential for the phosphorylation of downstream proteins. ATM contains
a highly conserved C-terminal kinase domain related to phosphatidylinositol 3-kinase (PI-3K).
REFERENCE: Tannock, Hill, Bristow, and Harrington. The Basic Science of Oncology. 4th
edition, Chapter 5.

74. A magnetron in a megavoltage linear accelerator is used to:

A. steer the electron beam.


B. stabilize the electron bunch current.
C. transfer heat from the waveguide.
D. produce microwave power.

Correct answer is D. RATIONALE: A magnetron is a microwave power source used in


megavoltage linear accelerator, radar, and microwave ovens.

75. Which of the following statements about the diagnosis and treatment of gynecomastia
related to the use of antiandrogen therapy for prostate cancer is true?

A. The rate of gynecomastia in the Scandinavian SPCG-7/SFUO trial was 30% for
patients given prophylactic breast irradiation.
B. Gynecomastia is associated with diffuse enlargement of the breast.
C. Gynecomastia is associated with a mass adjacent to the nipple.
D. The radiation treatment schedule should be 20 Gy (2 Gy x 10 fractions).

Correct answer: ABCD. RATIONALE: The rate of gynecomastia was 30% for patients given
prophylactic breast irradiation vs. 70% rate in the control group. Option B/C: Gynecomastia is
associated with a firm, rubbery mass immediately deep to the nipple – areolar complex. An
adjacent mass is worrisome for breast cancer and should be evaluated with
mammography/ultrasound. Diffuse breast enlargement is pseudogynecomastia. Option D:
There are variable fractions, but this schedule is protracted and not typical of what is used. A
common treatment schedule is 3 Gy/fraction x 4–5 fractions. REFERENCES: Gikas P, Mokbel
K. Management of gynecomastia: an update. International Journal of Clinical Practice.
2007;61(7):1209-1215. Gunderson & Tepper, eds. Clinical Radiation Oncology. 2nd edition.
2007;1561.
76. Which of the following outcomes was demonstrated in the INT-0123 (Minsky) study
comparing the use of 50.4 and 64.8 Gy of radiation therapy with chemotherapy for
esophageal cancer?

A. High-dose radiation therapy resulted in a significant survival benefit in the as-treated


analysis, but outcomes were not improved in the high-dose group by intent-to-treat.
B. The benefit of high-dose radiation therapy was obscured due to patient deaths
before a total dose of 50.4 Gy was reached.
C. There was no suggestion of a survival benefit with high-dose radiation therapy either
in intent-to-treat or as-treated analysis.
D. Local control was significantly better in the high-dose radiation therapy arm.

Correct answer is C. RATIONALE: In the report of INT-0123, an as-treated analysis was


performed due to the high incidence of early grade 5 toxicity in the high-dose radiation therapy
arm. However, there remained no survival difference in the two arms. Local failure was not
significantly different.

77. According to a recently published (Slotman) randomized trial, PCI improves survival in
patients who have SCLC with:

A. limited-stage disease irrespective of response to chemotherapy.


B. extensive-stage disease irrespective of response to chemotherapy.
C. limited-stage disease with no progression during chemotherapy.
D. extensive-stage disease with no progression during chemotherapy.

Correct answer is D. RATIONALE: Based on the PCI meta-analysis, patients who had limited-
stage SCLC and achieved a complete response to chemotherapy experienced improved
survival rates with PCI. In the European randomized trial (Slotman, et al), patients who had
extensive-stage small cell lung cancer and achieved any response to chemotherapy were
eligible and experienced improved survival rates.

78. Which of the following statements about the therapeutic ratio in patients receiving
curative radiation therapy for cancer is true?

A. For approximately the same total dose, shortening the overall treatment time
increases the chances of local tumor control without increasing acute side effects.
B. The goal of curative radiation therapy is to achieve a complete tumor response by
the last day of treatment while minimizing acute side effects.
C. The goal of curative radiation therapy is to kill all clonogenic tumor cells that could
give rise to a recurrence while minimizing the risk for serious complications.
D. The risk of serious late complications should be limited to 10% or less 5 years after
treatment.

Correct answer is C. RATIONALE: The goal of curative radiation therapy is to inactivate all
clonogenic tumor cells that could give rise to a recurrence while simultaneously minimizing the
risk for serious late normal tissue complications. Shortening the overall treatment time may
increase the chance of local tumor control in some cases (i.e., especially for tumors containing a
subset of rapidly growing cells), but it runs the risk of exacerbating acute normal tissue reactions
as well. Achieving a complete tumor response – reduction of tumor burden to below the limits of
clinical detection – by the last day of radiation therapy is a desirable goal, but it in no way
guarantees that all clonogenic tumor cells have been eradicated. Typically, the risk of serious
late complications should be limited to 5% or less (not 10%) 5 years after treatment.
79. What is an advantage of using a "field in a field" treatment technique for breast
treatment versus using a wedge?

A. Avoids the need for a skin bolus


B. Avoids the need for collimator rotation for field matching with a supraclavicular field
C. Decreases the dose to the contralateral breast
D. Decreases the magnitude of the hot spot

Correct answer is C. RATIONALE: The addition of a wedge, especially when added to the
medial treatment field, will increase the dose to the contralateral breast due to an increase in
scatter radiation. By using a "field in a field" technique, this increase in scatter dose may be
avoided.

80. A patient with Hodgkin lymphoma has a 12-cm anterior mediastinal mass with bilateral
involvement of the hilar lymph nodes and invasion of the anterior chest wall detected on
CT scan. No disease is found below the diaphragm, and the patient has no "B"
symptoms. Which of the following stages is associated with this patient's disease?

A. IAEX
B. IIAEX
C. IIIAEX
D. IVAEX

Correct answer is B. RATIONALE: Involvement of two lymph node regions (both hilum) on the
same side of the diaphragm with localized contiguous involvement of a single extranodal site
(chest wall), with >10-cm mass, and absence of "B" symptoms constitute stage IIAEX disease.

81. What is the AJCC stage for a patient with a tumor invading into, but not through, the
muscularis propria with four positive lymph nodes after preoperative chemoradiation
followed by resection?

A. pT2N2
B. pT3N2
C. ypT2N2
D. ypT3N2

Correct answer is C. RATIONALE: The "y" prefix in staging criteria is used for patients
undergoing pretreatment. A tumor invading into but not through muscularis propria is classified
as stage T2. Four or more lymph nodes would be classified as stage N2.
82. Based on the updated results of the phase III (D'Amico) trial, which of the following
outcomes was demonstrated in patients who had localized, unfavorable-risk prostate
cancer and received radiation therapy with or without androgen deprivation?

A. With longer follow-up of patients, there was a loss of significance in overall survival
with the addition of androgen deprivation.
B. Based on multivariate analysis, the duration of androgen deprivation had no impact
on overall survival.
C. Only patients with a low comorbidity score realized a benefit from androgen
deprivation.
D. Cardiac events were not a common cause of death in patients treated with androgen
deprivation.

Correct answer is C. RATIONALE: This important update from this phase III trial helps to
individualize the use of androgen deprivation for unfavorable-risk patients with prostate cancer.
Comorbidity plays an important role in patients who receive androgen deprivation for prostate
cancer. REFERENCE: D'Amico, et al. Journal of American Medical Association (JAMA).
2008;23:289-295.

83. Which of the following statements about the management of DCIS is true?

A. In the NSABP B24 trial, the addition of tamoxifen to irradiation of the breast
decreased the rate of IBTR.
B. In the NSABP B24 trial, the addition of tamoxifen to radiation therapy did not affect
the rate of invasive breast cancer recurrence.
C. Margin status predicts the risk for noninvasive IBTR but does not predict the risk for
invasive IBTR.
D. The addition of tamoxifen to radiation therapy results in a greater reduction of IBTR
in postmenopausal women than in premenopausal women.

Correct answer is A. RATIONALE: The addition of tamoxifen reduced the IBTR of all
recurrences from 11% to 7.7%. Tamoxifen reduced the rate of invasive cancer from 5.3% to
2.6%. It did not affect the rate of noninvasive recurrence (5.8% with radiation therapy alone vs.
5.0% with radiation therapy and tamoxifen). Margin status predicts the risk for IBTR, regardless
of whether the recurrence has an invasive or a noninvasive histology. Premenopausal women
experience a greater benefit from tamoxifen and radiation therapy than postmenopausal women
because premenopausal women have higher rates of IBTR overall. REFERENCE: Harris,
Lippman, Morrow, and Osborne. Diseases of the Breast. 4th edition. 2004;521-537. Halpern,
Perez, and Brady. Principles and Practice of Radiation Oncology. 5th edition. 2008;1162-1174.

84. Which of the following factors is most important in determining the prognosis of patients
with intracranial ependymoma?

A. Sex of the patient


B. Extent of resection
C. Location of the tumor
D. Use of chemotherapy

Correct answer is B. RATIONALE: Among the factors listed above, the extent of resection is
the most important prognostic factor. The use of chemotherapy has not been consistently
shown to impact survival or tumor control. REFERENCE: Merchant TE. Current management of
childhood ependymoma. Oncology. Williston Park; May 2002;16(5):629-42, 644.
A tumor consisting of 109 cells is characterized by a survival curve with α = 0.3 Gy 1 and

85.
β = 0.015 Gy 2. If the tumor is treated with repeated, once-daily 2-Gy fractions, which of

the following total doses will yield a 90% tumor control probability?

A. 50 Gy
B. 60 Gy
C. 70 Gy
D. 80 Gy

Correct answer is C. RATIONALE: This item incorporates knowledge of both of the following
cell survival models, the multi-fraction survival response and Poisson statistics. With a starting
population of 109 tumor cells (all assumed to be clonogenic in this example), a TCD90 (90%
tumor control probability) would correspond to that total radiation dose that reduces the initial
population to an average of 10-1 cells or 10 logs of cell killing (SF = 10-10). Because a multi-
fraction survival curve is roughly exponential, the following relatively simple calculations are
possible:
β 2
= e–( D+ D )
α
1) SF2 (the surviving fraction after the single dose of 2 Gy)
= e–[(0.3x2) + (0.015x4)]
= e–0.66 ≈ 0.52
2) Then:
Final SF = (SF2)N, where N = number of fractions
Final SF = 10-10 = (0.52)N
Taking the natural log of both sides of the equation and solving for N: N ≈ 35 fractions –
of 2 Gy each – corresponding to a total dose of 70 Gy.

86. Which of the following constitutes a serious error (potentially 20% or greater) in
treatment delivery to a point within an irradiated volume?

A. Incorrect wedge angle (45° versus 15°)


B. Incorrect wedge direction for a 45° wedge
C. Incorrect photon energy (6 instead of 18 MV) when prescribed to a 10 cm depth
D. SAD treatment using SSD monitor units for an 11 cm depth calculation

Correct answer is B. RATIONALE: The dose difference in the use of a 45° wedge vs. a 15°
wedge is approximately 30%. An incorrect energy represents a difference of ~1.5% per cm, and
SAD vs. SSD depends on inverse square (2% per cm). An incorrect wedge angle would flip the
'toe' hot spot with 'heel' cold spot, leading to a potentially worst-case dose change of ~70%.

87. Which of the following types of benign meningioma should NOT be treated with
stereotactic radiosurgery?

A. Optic nerve sheath


B. Skull base
C. Convexity
D. Parasagittal

Correct answer is A. RATIONALE: Radiosurgery should not be performed for optic nerve
sheath tumors given the risk for optic neuropathy.
88. Which of the following clinical descriptions is associated with grade III heterotrophic
bone formation?

A. Bony islands within the soft tissues around the hip


B. Bony ankylosis between the proximal femur and pelvis
C. Exophytes of the pelvis or proximal femur and a distance of at least 1 cm or more
D. Exophytes of the pelvis or proximal femur and a distance of less than 1 cm

Correct answer is D. RATIONALE: Option A: Grade II. Option B: Grade IV. Option C: Grade II.
Option D: Grade III. REFERENCES: Booker AF, Bowerman JW, Robinson RA, Riley CH:
Ectopic ossification following total hip replacement. Journal of Bone Joint Surgery.
1973;55:1629-1632. Gunderson & Tepper, eds. Clinical Radiation Oncology. 2nd edition.
2007;1561-1562.

89. Which of the following statements about the use of radioactive iodine for patients with
thyroid malignancies is true?

A. Iodine imaging should be performed before ablation of the thyroid remnant after
surgery.
B. Typically, iodine imaging should be performed within 1 month after CT scans with
IV contrast.
C. A high level of thyroxin is necessary before iodine imaging.
D. The typical dose of radioactive iodine therapy is between 50 and 100 mCi.

Correct answer is A. RATIONALE: It is important to understand how and when to use iodine
imaging. Prior to iodine imaging, a high level of TSH is necessary for an optimal uptake of
radiolabeled iodine.

90. What is the probability that solitary plasmacytoma of the bone will progress to multiple
myeloma within 10 years?

A. 10% to 20%
B. 30% to 40%
C. 60% to 70%
D. 80% to 90%

Correct answer is C. RATIONALE: It is important to know this fact for counseling patients. The
majority of patients will progress to multiple myeloma, and surveillance/follow-up is important
after primary therapy.
91. What is the purpose of randomization using a probability model in a clinical trial?

A. Identify unknown prognostic factors


B. Select a representative sample of patients with the disease of interest
C. Provide an unbiased estimate of the treatment effect in each arm of the study
D. Assign equal numbers of patients to each arm of the study

Correct answer is C. RATIONALE: A random sample helps to provide a fair, unbiased


comparison of the treatments within the population of patients from which participants are
recruited, although this population may not be representative of all patients with the disease.
Randomization may not result in equal, or nearly equal, numbers of patients with a particular
characteristic in each treatment group. It does distribute biasing characteristics according to a
known probability distribution so that they do not obscure identification of treatment effects, if
any, by statistical tests. The number of patients in each study arm may differ, either by design or
as a product of the recruitment process.

92. Which of the following components of a signaling network help to prevent genomic
instability and carcinogenesis?

A. Executioners
B. Promoters
C. Initiators
D. Sensors

Correct answer is D. RATIONALE: Optimal DNA damage responses are regulated by sensors,
transducers, and effectors. Sensors, typically chromatin-bound proteins, can detect DNA
damage and recruit signal transducers. Transducers are protein kinases that can amplify the
signal by phosphorylation of numerous downstream proteins. Effectors are the proteins modified
by phosphorylation that can negatively regulate the cell cycle progression.

93. According to the NCI Canada randomized trial, which of the following statements about
the use of preoperative versus postoperative radiation therapy for patients with sarcoma
of the extremities is true?

A. Field sizes were significantly larger.


B. It resulted in a higher incidence of postoperative wound complications.
C. It increased the incidence of late fibrosis.
D. It improved local control.

Correct answer is B. RATIONALE: The NCI Canada study was terminated after interim
analysis showed a higher rate of wound complications in the preoperative radiation therapy (RT)
arm. The preoperative arm also used smaller fields sizes. This is expected since the full
operative bed/scars do not have to be included in the preoperative setting. The updated report
of this trial demonstrated a trend towards less late fibrosis with preoperative RT compared to
postoperative RT. No local control difference was seen between the two arms of the study.
REFERENCES: O'Sullivan, et al. Lancet. 2002. Davis, et al. Radiotherapy & Oncology. 2005.
94. The most important prognostic factor for patients with inflammatory breast cancer is:

A. tumor response to chemotherapy.


B. presence or absence of dermal lymphatic invasion.
C. HER-2/neu amplification.
D. patient age.

Correct answer is A. RATIONALE: Many studies have confirmed that a pathologic complete
response to chemotherapy is the most important prognostic factor in inflammatory breast
cancer. Dermal lymphatic invasion and HER-2/neu positivity may or may not be associated with
inflammatory breast cancer but have not been reported to be more important than response to
chemotherapy. REFERENCE: Harris, et al. International Journal of Radiation Oncology,
Biology, Physics (Int J Radiat Oncol Biol Phys). 2003;55:1200-8.

95. According to current U.S. regulatory standards, which of the following radiation dose
units is most appropriately used to express the dose limit for individual members of the
public?

A. Exposure (in Roentgen)


B. Equivalent dose (in Gray)
C. Effective dose (in Sievert)
D. Total effective dose equivalent (in Sievert)

Correct answer is D. RATIONALE: Total effective dose equivalent, in Sievert, is the current
term. Residents need to know that older generic terms (dose, exposure, etc.) no longer apply to
regulatory standards. Moreover, it requires knowledge that Sievert is the unit for regulatory
standards.

96. What is the most common histologic subtype of cholangiocarcinoma?

A. Adenocarcinoma
B. Neuroendocrine carcinoma
C. Transitional cell carcinoma
D. Squamous cell carcinoma

Correct answer is A. RATIONALE: More than 90% of cholangiocarcinomas are


adenocarcinomas.

97. Which of the following treatments should be performed for patients with bilateral Wilms
tumor?

A. Bilateral renal biopsy, followed by preoperative chemotherapy


B. Bilateral renal biopsy, followed by immediate radiation therapy and chemotherapy
C. Bilateral nephrectomy and consideration for kidney transplant
D. Nephrectomy of the worse side, followed by chemotherapy and radiation therapy

Correct answer is A. RATIONALE: The goal in patients with bilateral Wilms tumor is to obtain
local control while sparing renal parenchyma. Therefore, upfront cytoreduction with
chemotherapy is usually offered before a second-look surgery. The 10-year overall survival for
patients with synchronous tumors is 70%, according to NWTS-2 and -3 and SIOP 1, 2, and 5
studies. Bilateral Wilms tumor is regarded as stage V disease.
98. Low-dose-rate prostate brachytherapy as monotherapy is NOT appropriate for patients
with:

A. stage T2a prostate cancer.


B. a prostate volume of 40 cc.
C. a Gleason score of 3+3.
D. an AUA score of 20.

Correct answer is D. RATIONALE: Good candidates for prostate brachytherapy alone are low-
risk patients with stage T1-T2a disease, Gleason score of 6 or below, PSA less than 10, no
enlarged prostate or high AUA score.

99. Which of the following chromosomal translocations is most closely associated with
Burkitt's lymphoma?

A. t(8;14)
B. t(11;14)
C. t(11;18)
D. t(14;18)

Correct answer is A. RATIONALE: The (8;14) chromosomal translocation is most closely


associated with Burkitt's lymphoma. The rest of the translocations are for mantle cell lymphoma
(11;14), marginal cell lymphoma (11;18), and follicular cell (14;18) lymphoma.

100. Which of the following agents is a radiation protector?

A. Misonidazole
B. Amifostine
C. Tirapazamine
D. Iododeoxyuridine

Correct answer is B. RATIONALE: Amifostine is a "true" radiation protector that has no effect
in and of itself, but in combination with radiation acts within milliseconds to scavenge radiation-
induced free radicals that might otherwise interact with cellular components, especially DNA,
and lead to irreparable damage. (Several other classes of agents are "apparent" radioprotectors
in that they likewise afford protection of normal tissues, although by mechanisms other than
rapid, free radical scavenging, e.g., a cytokine that stimulates bone marrow to increase
production of new white blood cells, thereby "heading off" a potentially dangerous drop in
peripheral white cell counts.) Misonidazole is a hypoxic cell radiosensitizer, tirapazamine is a
bioreductive drug selectively toxic to hypoxic cells, and iododeoxyuridine is a radiosensitizer of
rapidly proliferating cells.
101. Which of the following responses of silver halide film used for quality assurance of an
IMRT fluence pattern is most accurate?

A. It tends to overrespond due to the lower energy of the large amount of scatter dose.
B. It is highly dependent on the temperature of the solid water phantom in which it is
measured.
C. It will be less than the response in tissue because of the lower Z of silver.
D. It will not change significantly with different processing techniques.

Correct answer is A. RATIONALE: A significant issue with film is its over-response to low-
energy x-rays due to the high atomic number of its active material, silver (Z = 47). As a result,
film is much more sensitive to the interaction of low-energy photons, such as the influence of
scattered photons during IMRT QA.

102. In a prospective study comparing patients with prostate cancer treated with 3D
conformal radiation therapy with or without an endorectal balloon (ERB), the ERB group
had:

A. reduced late rectal toxicity.


B. better biochemical outcome.
C. more high-grade telangiectasia.
D. increased rectal stricture.

Correct answer is A. RATIONALE: ERB reduced the rectal wall volume exposed to doses >40
Gy, resulting in reduction of late rectal mucosal changes and reduced late rectal toxicity.
REFERENCE: van Lin EN, et al. Reduced late rectal mucosal changes after prostate 3D
conformal radiation therapy (3D-CRT) with endorectal balloon as observed in repeated
endoscopy. International Journal of Radiation Oncology, Biology, Physics (Int J Radiat Oncol
Biol Phys). 2007;67(3):799-811.

103. Which of the following factors is NOT prognostic for patients with medulloblastoma?

A. Stage
B. Extent of resection
C. Age younger than 3 years
D. Location of tumor

Correct answer is D. RATIONALE: The specific location of the primary tumor does not appear
to have prognostic implication, though tumors in the cerebellar hemisphere tend to occur in
older patients. The other factors have a clear effect on prognosis.
104. Which of the following FIGO stages is associated with ovarian carcinoma with implants
confined to the pelvis and malignant cells in the peritoneal washings?

A. IIC
B. IIIA
C. IIIB
D. IIIC

Correct answer is A. RATIONALE: FIGO stage IIC signifies pelvic extension and/or implants
with malignant cells in ascites or peritoneal washings. FIGO stage IIIA = microscopic peritoneal
metastasis beyond pelvis; stage IIIB = macroscopic peritoneal metastasis beyond pelvis 2 cm or
less in dimension; stage IIIC = peritoneal metastasis beyond pelvis more than 2 cm in greatest
dimension and/or regional lymph node metastasis.

105. Which of the following organs is composed of anatomically defined FSUs?

A. Skin
B. Heart
C. Spinal cord
D. Liver

Correct answer is D. RATIONALE: Withers' functional subunit model proposes that the
irradiation volume dependence of normal tissue tolerance depends in part on the structural and
functional organization of tissues into functional subunits (FSUs). For some organs, such as the
liver or kidney, an FSU likely corresponds to the anatomically defined structure that would be
minimally required to perform the function of that tissue (i.e., a liver lobule and nephron,
respectively). For other organs, such as the skin, heart, central nervous system, and others,
what constitutes an FSU is either unknown or not clearly defined in either an anatomic or
histological sense. REFERENCE: Hall and Giaccia. Radiobiology for the Radiologist. 6th
edition, Chapter 19.

106. Extramedullary plasmacytoma is most commonly located in the:

A. lower extremities.
B. head and neck.
C. lungs.
D. spine.

Correct answer is B. RATIONALE: It is important to know where extramedullary plasmacytoma


is located in making a differential diagnosis. Extramedullary plasmacytoma may involve the
nasal cavity, sinuses, nasopharynx, tonsils, and larynx.

107. What is the approximate depth of the 90% dose for a 16-MeV electron beam?

A. 02 cm
B. 04 cm
C. 08 cm
D. 16 cm

Correct answer is B. RATIONALE: As a rule of thumb, the depth of the 90% depth dose can
be approximated as E(MeV)/4.
108. Which of the following is used to decide when a diagnostic test should be ordered?

A. Central limit theorem


B. Inferential statistics
C. Threshold model
D. Level of significance

Correct answer is C. RATIONALE: The threshold model is used to decide when a diagnostic
test should be ordered, as opposed to doing nothing or treating the patient without ordering the
test. Central limit theorem states that the distribution of means is approximately normal if the
sample size is large enough. Inference (statistical) is the process of drawing conclusions about
a population of observations from a sample of observations. Level of significance applies to the
probability of incorrectly rejecting the null hypothesis in a test of hypothesis.

109. According to a prospective randomized (Donovan) trial, which of the following outcomes
is associated with IMRT when compared to standard 2D radiation therapy for patients
with early-stage breast cancer?

A. Palpable induration is reduced after 5 years.


B. Cardiac events are reduced after 10 years.
C. Pulmonary events are reduced after 5 years.
D. Quality of life is improved at 5 years.

Correct answer is A. RATIONALE: A randomized trial comparing breast IMRT to standard 2D


radiotherapy was published. Results comparing dose homogeneity, breast appearance,
induration and quality of life were reported. No data on cardiac or pulmonary events was
reported. REFERENCE: Donavan, et al. Radiotherapy and Oncology. 2007;254-64.

110. Which of the following therapeutic options would be best after resection with negative
surgical margins for a desmoid tumor?

A. Radiation therapy to 50 to 55 Gy
B. Radiation therapy to 60 to 66 Gy
C. Systemic chemotherapy
D. Observation

Correct answer is D. RATIONALE: Desmoid tumors have a low recurrence rate following
complete surgical resection.
111. Which of the following statements about the epidemiology and initial presentation of
penile cancer is true?

A. Circumcision does not reduce the risk of penile cancer.


B. Approximately 50% of palpable inguinal lymph nodes are pathologically positive at
presentation.
C. Clinically negative but pathologically positive inguinal lymph nodes are rare.
D. Distant metastases are common at diagnosis.

Correct answer is B. RATIONALE: It is important to highlight that a palpable inguinal lymph


node is not necessarily pathologically positive. Circumcision reduces the risk of penile cancer.
Clinically negative but pathologically positive inguinal lymph nodes are common. Distant
metastases are not common at diagnosis.

112. Which of the following transcription factors is stabilized under hypoxic conditions?

A. AP-1
B. HIF-1α
C. HIF-1β
D. NFκB

Correct answer is B. RATIONALE: Hypoxia-inducible factor-1α (HIF-1α), upon stabilization in


the relative absence of oxygen, trans-activates many downstream target genes that help cells
adapt to the stresses associated with hypoxic conditions. In the presence of oxygen, HIF-1α
becomes ubiquitinated, targeting it for degradation in the proteasome. REFERENCE: Hall and
Giaccia. Radiobiology for the Radiologist. 6th edition, Chapter 6 (Figure 6.17 in particular).

113. Which of the following statements about BRCA2 germline mutations is true?

A. A similar histology of breast cancer is associated with patients who have BRCA1 or
BRCA2 germline mutations.
B. There is a higher risk for developing ovarian cancer in patients with a BRCA2
germline mutation than in patients with a BRCA1 germline mutation.
C. Men with a BRCA2 germline mutation have a 15% to 20% risk of developing
prostate cancer.
D. Patients with a BRCA2 germline mutation have a 50% risk of developing breast
cancer by 70 years of age.

Correct answer is C. RATIONALE: The histology of breast cancers occurring in patients with
BRCA1 and BRCA2 germline mutations differs from one another and from patients who are
sporadic carriers. Tumors in patients with BRCA1 germline mutations are ER-negative in up to
90% of cases. RCA2-related tumors are more frequently ER-positive than BRCA1-related
tumors. Medullary cancers are seen more commonly in patients with BRCA1 mutations than in
patients with either BRCA2 mutations or in controls. Tumors related to either BRCA1 or BRCA2
mutations tend to have higher grades when compared to sporadic tumors. The estimated risk
for ovarian cancer in patients with BRCA1 mutations is 30% to 40%; the estimated relative risk
(RR) is 18-29. The estimated risk for ovarian cancer in patients with BRCA2 mutations is 27%;
the estimated RR is 16-19. The lifetime risk for developing breast cancer in patients with either
BRCA1 or BRCA2 mutations is 60% to 80%. REFERENCES: Harris, Lippman, Morrow, and
Osborne. Diseases of the Breast. 4th edition. 2004;283-284 (option B). Halpern, Perez, and
Brady: Principals and Practice of Radiation Oncology. 5th edition. 2008;1630 (option C). Harris,
Lippman, Morrow, and Osborne. Diseases of the Breast. 4th edition. 2004;283 (option D). Harris,
Lippman, Morrow, and Osborne. Diseases of the Breast. 4th edition. 2004;283.
114. Which of the following reportable errors most commonly occurs during radiation
therapy?

A. Incorrect treatment site


B. Miscalculated exposure time or dose
C. Inadequate review of patient's chart
D. Failure of treatment unit

Correct answer is B. RATIONALE: Calculation errors of exposure time or dose are almost
twice as likely as the inadequate review of patient chart or error in an anatomical area to be
treated and more than ten times as likely as the treatment unit mechanical failure (IAEA 2000
report).

115. Which of the following statements about radiation therapy for urethral malignancies is
true?

A. Preoperative radiation therapy is the standard of care.


B. Nodal treatment field design depends on the tumor location.
C. Concurrent chemoradiotherapy is not recommended.
D. The entire urethra should be included in the treatment fields.

Correct answer is B. RATIONALE: Understanding basic concepts in the management of


urethral malignancies is important, including patterns of drainage and the indications for
combined modality therapy. Preoperative radiation therapy is not the standard of care.
Combined chemoradiotherapy is recommended, especially for unresectable cases. Nodal portal
treatment depends on the location of the tumor.

116. Which of the following treatments is NOT commonly recommended for patients who
have hematopoietic syndrome?

A. Administration of antibiotics
B. Administration of hematopoietic colony stimulating factors
C. Administration of transforming growth factor-β (TGF-β)
D. Bone marrow transplantation

Correct answer is C. RATIONALE: Use of hematopoietic cytokines, bone marrow


transplantation, and antibiotics are common treatments for hematopoietic syndrome. TGF-β
possesses both tumor-inhibiting and tumor-promoting activities (depending on context) and is
being studied as a possible therapeutic target for new drug development, but if anything, is an
inhibitor of hematopoiesis, not a stimulator. REFERENCES: Goustin A, et al. Perspective in
cancer research: Growth factors and cancer. Cancer Research. 1986;46:1015-1029. Shetty V,
et al. Measurements of apoptosis, proliferation and three cytokines in 46 patients with
myelodysplastic syndromes. Leukemia Research. 1996;20:891-900.
117. Which of the following findings best indicates the need for an axillary boost dose after
modified radical mastectomy and adjuvant chemotherapy for stage IIIA (T3N1) breast
cancer?

A. There is microscopic extracapsular nodal extension.


B. A single metastatic lymph node measures 2.5 cm.
C. Twenty percent of the lymph nodes are positive.
D. Fewer than six axillary lymph nodes are dissected.

Correct answer is D. RATIONALE: In a series, recurrence in the low-mid axilla was rare and
not significantly higher for patients with a higher percentage of positive lymph nodes, larger
nodal size, or extranodal extension. In contrast, an inadequate axillary dissection is a common
indication for a boost dose of irradiation to the axillary lymph nodes. REFERENCE: Strom, et al.
International Journal of Radiation Oncology, Biology, Physics (Int J Radiat Oncol Biol Phys).
2005;63:1508-13.

118. Which of the following statements about the response rate to radionuclide therapy for
painful metastatic bone disease is true?

A. The partial response rate for strontium is 50% to 60%.


B. The complete response rate for strontium is 30% to 50%.
C. The partial response rate for samarium is 40% to 50%.
D. The complete response rate for samarium is 50% to 55%.

Correct answer is D. RATIONALE:


Radionuclide Partial Rate Complete Rate Response Duration
89
Sr 65 – 80% 10 – 30% ~6 mos.
Samarium 75 – 80% 54% 2 – 4 ½ mos.

REFERENCE: Gunderson & Tepper, eds. Clinical Radiation Oncology. 2nd edition. 2007;448.

119. How does GafChromic film differ from radiographic film?

A. It is more accurate.
B. It is used to measure lower radiation doses.
C. It does not require chemical processing.
D. It has a more linear dose response.

Correct answer is C. RATIONALE: GafChromic film is self-developing and does not require
any chemical processing.
120. Which of the following statements best describes an intent-to-treat analysis?

A. A randomized patient who is later found to be ineligible for the study may be
disregarded at analysis.
B. Patients are included in the analysis of the study arm to which they were
randomized.
C. Patients who withdraw from the study before completion should be analyzed
separately.
D. Patients who are noncompliant with treatment should be dropped from analysis.

Correct answer is B. RATIONALE: Omitting patients who withdraw without completing the
study is a serious source of bias, as is dropping patients for noncompliance with treatment
protocols. These may be patients for whom the treatment does not work or for whom the
treatment has serious adverse consequences. There is no sound way to identify and eliminate
this bias in the context of the trial, so the intention-to-treat analysis includes all consenting,
randomized patients in the study arm to which they were randomized.

121. Postoperative radiation therapy may be indicated for a patient who has what stage of
NSCLC and is receiving what type of therapy?

A. T1N1 disease and is receiving adjuvant chemotherapy


B. T1N1 disease and is unable to receive adjuvant chemotherapy
C. T2N0 disease and is receiving adjuvant chemotherapy
D. T2N0 disease and is unable to receive adjuvant chemotherapy

Correct answer is B. RATIONALE: The recently published update of the Adjuvant Navelbine
International Trialist Association (ANITA) randomized trial demonstrated that postoperative
radiation therapy provided a survival benefit in patients with both pN1 and pN2 NSCLC disease
when these patients were unable to receive adjuvant chemotherapy. REFERENCE:
International Journal of Radiation Oncology, Biology, Physics (Int J Radiat Oncol Biol Phys).
72(3):695–701.

122. According to the CONKO (Oettle) trial, which of the following outcomes was associated
with adjuvant therapy for patients with resected pancreatic cancer?

A. Local control was improved in patients receiving gemcitabine.


B. Local control was improved in patients receiving radiation therapy.
C. Disease-free survival was improved in patients receiving gemcitabine.
D. Disease-free survival was not affected by radiation therapy.

Correct answer is C. RATIONALE: This is an important trial showing the efficacy of


chemotherapy in the adjuvant setting. Local control was not substantially different. Radiation
was not used in this study.
123. Oxygen's "k-value" of approximately 0.5% corresponds to the O2 concentration at which:

A. angiogenesis is stimulated.
B. the HIF-1α transcription factor is stabilized.
C. the radiosensitivity of the cell is halfway between that of the fully aerobic and fully
hypoxic response.
D. the fraction of hypoxic and aerobic cells in a tumor is approximately equal.

Correct answer is C. RATIONALE: Oxygen’s “k-value” corresponds to that concentration of


oxygen that yields cellular radiosensitivity halfway between that of the fully oxic and fully hypoxic
response. REFERENCE: Hall and Giaccia. Radiobiology for the Radiologist. 6th edition,
Chapter 6.

124. Which of the following types of carcinoma most commonly involves the fallopian tube?

A. Serous
B. Mucinous
C. Clear cell
D. Metastatic

Correct answer is D. RATIONALE: The most common carcinoma that involves the fallopian
tube is metastatic, generally from another site in the genital tract.

125. Which of the following mixtures of 1 Ci of each radionuclide will have the highest activity
after 5.26 years? Note: 169Yb (T1/2 = 32 days), 192Ir (T1/2 = 73.8 days), 60Co (T1/2 = 5.26
years), and 137Cs (T1/2 = 30 years)
169
A. Yb and 60Co
169
B. Yb and 137Cs
192
C. Ir and 60Co
192
D. Ir and 137Cs

Correct answer is D. RATIONALE: This item requires an understanding of the concept of half-
life. The mixture with the two longest-lived components (192Ir and 137Cs) will have the highest
activity.

126. Postoperative chemoradiation therapy is most likely to improve survival in patients with
which of the following stages of NSCLC?

A. T1N1
B. T1N2
C. T2N1
D. T3N0

Correct answer is B. RATIONALE: A recent assessment of postoperative radiation therapy


(RT) in 7,465 patients with resected stage II or III non-small cell lung cancer (NSCLC) found that
postoperative RT increased survival in patients with N2 disease but not in those with N1 or N0
disease. Only chemotherapy, not chemoradiation, should be given for N1 disease based on
currently available data. The recently published update of the Adjuvant Navelbine International
Trialist Association (ANITA) randomized trial demonstrated that postoperative RT provided a
survival benefit in patients with both pN1 and pN2 NSCLC disease when these patients were
unable to receive adjuvant chemotherapy. REFERENCES: Lally, et al. Journal of Clinical
Oncology. 2006;24(19):2998-3006. International Journal of Radiation Oncology, Biology,
Physics (Int J Radiat Oncol Biol Phys). 72(3):695–701.
127. Which of the following management steps would be most appropriate for a patient who
is to undergo breast-conserving therapy with neoadjuvant chemotherapy for a 6-cm
breast mass?

A. Placement of a clip in the tumor before chemotherapy to mark the primary location
for the surgeon in case the tumor has a complete clinical response to chemotherapy
B. Biopsy of mammographic abnormalities in other quadrants of the breast after
chemotherapy to rule out multicentric disease before irradiation
C. No radiation therapy required if the patient has a complete pathologic response to
neoadjuvant chemotherapy
D. Irradiation of the supraclavicular and axillary lymph nodes even if the nodes are
found to be negative after resection and administration of chemotherapy

Correct answer is A. RATIONALE: A clip needs to be placed to mark the primary tumor
location for the surgeon in case of a complete clinical response to neoadjuvant chemotherapy.
Multicentric disease needs to be ruled out prior to chemotherapy as well as after chemotherapy.
All patients should receive postoperative irradiation, regardless of their response to
chemotherapy. Regional lymph node irradiation is controversial, but was not required in the
randomized NSABP B-18 and B-27 trials. REFERENCES: Buchholz, et al. Journal of Clinical
Oncology. 2008;26:791-7. Rastogi, et al. Journal of Clinical Oncology. 2008;26:778-85.

128. Which of the following best describes the shape of the radiation dose response curve for
dicentric chromosomal aberrations in peripheral blood lymphocytes?

A. A linear-quadratic dose response following low-LET irradiation at a low dose rate


B. A linear-quadratic dose response following low-LET irradiation at a high dose rate
C. A quadratic dose response following low-LET irradiation at a high dose rate
D. A linear-quadratic dose response following high-LET irradiation at a high dose rate

Correct answer is B. RATIONALE: Dicentrics require two chromosomal breaks to form. For
low-LET radiation, a small fraction of these form from single particle traversals and the rest from
the traversal of two separate particles, leading to a linear-quadratic dose response overall. At
low dose rates, these single particle events dominate, causing the quadratic term to approach
zero and the dose response to become essentially linear. On the other hand, for high-LET
radiation, two breaks caused by single particle traversals dominate, likewise yielding a linear
dose response. However, the linear dose response curve for dicentrics is steep for high-LET
irradiation (regardless of dose rate) and shallow for low-LET irradiation at a low dose rate.
REFERENCE: Mettler and Upton. Medical Effects of Ionizing Radiation. 3rd edition, Chapter 3
(Figure 3-9 in particular).

129. In a linear regression model, what is the meaning of R2?

A. It is the mean square error.


B. It is the square of the significance level.
C. It is the percent of variance explained by the model.
D. It is a measure of the degree to which the data are skewed.

Correct answer is C. RATIONALE: R2, the square of the correlation, can be shown to be the
variance explained by the regression model divided by the total variance. That is, it is the
percent of variance explained by the model.
130. Which of the following pathologic factors is an absolute indication for concurrent
adjuvant chemoradiation for high-risk patients with advanced head and neck cancer?

A. Multiple positive lymph nodes


B. Extracapsular extension
C. Lymphvascular invasion
D. Perineural invasion

Correct answer is B. RATIONALE: This item is based on the EORTC and RTOG high-risk
postoperative trials (i.e., the results of RTOG 95-01 and EORTC 22931). The initial results of
RTOG 95-01 would support options A or B, but EORTC 22931 would not support option A, and
the joint analysis supports only the correct answer (option B).

131. If it takes 400 seconds to deliver 7.5 Gy to the surface of an HDR cylindrical applicator
for the first treatment, how many seconds will it take to deliver the same dose 2 weeks
later? (Assume the same source is used, and T1/2 of the source is 74 days.)

A. 351
B. 400
C. 456
D. 500

Correct answer is C. RATIONALE: Over the course of 2 weeks, the source will exponentially
decay. To deliver the same dose, the treatment time will need to be exponentially increased as
follows: T = To x exp(0.693*t/t1/2); T = 400 s x exp(0.693*14d/74d) = 456 seconds.

132. Which of the following terms applies to the proportion of time that a diagnostic test is
negative in patients who do not have the disease?

A. Sensitivity
B. Specificity
C. Standard error
D. Subjective probability

Correct answer is B. RATIONALE: Specificity reflects the proportion of time that a diagnostic
test is negative in patients who do not have the disease.

133. Which of the following statements about halogenated pyrimidine radiosensitizers is true?

A. Misonidazole produces less radiosensitization than etanidazole because it does not


cross the blood-brain barrier.
B. IdUrd is used preferentially to BrdUrd as a radiosensitizer because it produces less
photosensitivity.
C. Radiosensitization by halogenated pyrimidines is selective for slowly proliferating
cells.
D. Halogenated pyrimidine radiosensitizers are selectively toxic to tumor cells.

Correct answer is B. RATIONALE: The halogenated pyrimidine IdUrd is preferred to BrdUrd


because it produces comparable radiosensitization, but significantly less photosensitization, a
troublesome side effect in patients receiving BrdUrd. Misonidazole and etanidazole are not
halogenated pyrimidines. Halogenated pyrimidines are selective radiosensitizers of rapidly
proliferating cells, whether they derive from tumors or normal tissues. REFERENCE: Hall and
Giaccia. Radiobiology for the Radiologist. 6th edition, Chapter 25.
134. Which of the following statements about integrating paclitaxel into a regimen of
doxorubicin/cyclophosphamide (AC) chemotherapy, surgery, and radiation therapy for
treatment of breast cancer is true?

A. Radiation therapy is best sequenced between AC chemotherapy and paclitaxel


because of increased radiation toxicity after paclitaxel.
B. In patients undergoing breast-conserving therapy, the local recurrence rate is
unchanged in patients receiving AC chemotherapy compared with AC
chemotherapy plus paclitaxel.
C. Concurrent weekly paclitaxel during radiation therapy is associated with a higher risk
of pneumonitis.
D. Concurrent paclitaxel and dose-reduced radiation therapy result in improved local
control rates when compared to AC chemotherapy → paclitaxel → radiation therapy
outcomes.

Correct answer is C. RATIONALE: Radiation is usually sequenced after AC chemotherapy and


paclitaxel. In a randomized study (CALGB 9344), AC chemotherapy and paclitaxel in patients
treated with breast-conserving surgery and radiation therapy was associated with a lower local
recurrence rate at 5 years compared to AC chemotherapy alone. There was no detriment in
radiation therapy delivery or radiation therapy tolerance after paclitaxel. Twenty-five percent of
patients had dose-limiting toxicity, including pneumonitis, with concurrent radiation therapy and
paclitaxel in a study. REFERENCES: Journal of Clinical Oncology. 2005;23:30-40. Burstein, et
al. International Journal of Radiation Oncology, Biology, Physics (Int J Radiat Oncol Biol Phys).
2006;64:496-504.

135. According to the INT-0116 (Macdonald) study, which of the following was associated
with postoperative chemoradiation and chemotherapy (3 cycles) versus surgery alone
for resected gastric cancer?

A. Control of metastatic disease was improved with adjuvant therapy, leading to


improved overall survival.
B. Locoregional control was improved with adjuvant therapy, leading to improved
overall survival.
C. An increase in deaths was associated with adjuvant therapy, eliminating any survival
benefit.
D. The study used cisplatin/5-fluorouracil (5-FU) chemotherapy.

Correct answer is B. RATIONALE: The INT-0116 study showed a survival benefit directly
related to locoregional control. No difference in metastatic rate was seen. The study was
positive by intent-to-treat. The backbone of chemotherapy was 5-fluorouracil/leucovorin
(5-FU/LV). REFERENCE: New England Journal of Medicine (NEJM). 2001.
136. According to NRC regulations, acceptance tests of a new HDR treatment planning
system do NOT require verification of the:

A. effectiveness of the optimization algorithm to achieve the best treatment plan.


B. accuracy of dose, dwell time, and treatment time calculations at representative
points.
C. accuracy of the software used to determine sealed source positions from
radiographic images.
D. accuracy of isodose plots and graphic displays.

Correct answer is A. RATIONALE: According to 10 CFR Part 35.657, the effectiveness of the
optimization algorithm to achieve the best treatment plan is not a minimum requirement of the
acceptance test procedure to verify in the therapy-related treatment planning system.

137. Which of the following AJCC staging criteria for NSCLC is correct?

A. Tumor invasion of the carina is T3.


B. Tumor invasion of the phrenic nerve is T4.
C. Involvement of the ipsilateral supraclavicular lymph node is N3.
D. Involvement of the contralateral mediastinal lymph node is N2.

Correct answer is C. REFERENCE: AJCC lung cancer staging. AJCC Cancer Staging
Manual, 6th edition.

138. What is the probability of occult metastasis to the inguinofemoral lymph nodes of a
2.5-cm squamous cell carcinoma of the vulva classified as clinical stage T2N0M0?

A. 10%
B. 20%
C. 30%
D. 40%

Correct answer is C. RATIONALE: Clinical tumor size correlates with occult metastases in
patients with clinically negative lymph node involvement as follows: 0 to 1 cm = 7.7%; 1.1 to 2
cm = 23.9%; 2.1 to 3 cm = 31%; 3.1 to 5 cm = 36.4%. REFERENCE: Adapted from Gonzalez,
et al. International Journal of Radiation Oncology, Biology, Physics (IJROBP). 2003;57:419-424.

139. Spheroids are used as a model system for:

A. small preangiogenic tumors.


B. large angiogenic tumors.
C. tumors suppressed by the immune system.
D. apoptosis.

Correct answer is A. RATIONALE: Multicellular spheroids are a model system for studying
radiation effects that combine the convenience and control of a cellular assay with the three
dimensional structure, cell-cell contact, and nutrient and oxygen gradients more reminiscent of
tumors in vivo. As large spheroids can reach sizes of 1-2 mm in diameter, they can be
considered good models for small, pre-angiogenic tumors.
140. A Kaplan-Meier survival curve yields a 1-year survival probability of 0.45 and an
associated standard error of 0.07. The 95% confidence interval for the 1-year survival
probability is approximately:

A. 0.40 to 0.50.
B. 0.38 to 0.52.
C. 0.31 to 0.59.
D. 0.29 to 0.61.

Correct answer is C. RATIONALE: The 95% confidence bounds are computed as the value of
interest (approximately +2X the standard error). In this case, the 95% confidence interval is
computed as follows: 0.45 +2(0.07), which equals approximately 0.31 to 0.59.

141. Which of the following statements about heterotrophic ossification is true?

A. Its incidence is 10% in patients with moderate or severe osteophytes at the femoral
head and socket before hip replacement.
B. Radiographically detected calcifications are not seen before 3 months
postoperatively.
C. The radiation field is 8 x 8 cm, is approximately 3 cm above the acetabulum, and
excludes two thirds of the implant shaft.
D. Preoperative radiation treatment with 7 to 8 Gy in 1 fraction is equally effective as
the same treatment given postoperatively.

Correct answer is D. RATIONALE: Option A: The incidence of heterotrophic ossification is


more than 50% in these patients. Option B: Calcified structures are seen as early as 3 to 6
weeks postoperatively, and patients may report increased pain from heterotrophic bone
formation within several days after their surgery. Option C: The irradiation field that covers the
entire periarticular risk region will be approximately 14 x 14 cm. The cranial border is 3 cm
above the acetabulum, and the field includes two thirds of the implant shaft. Option D: In three
randomized trials, there was no significant difference in effectiveness of high and low doses or
difference between fractionated and single-dose therapy. REFERENCE: Gunderson & Tepper,
eds. Clinical Radiation Oncology. 2nd edition. 2007;1561-1562.

142. The beam stop on a linear accelerator should transmit only 0.1% of the incident radiation
beam. What additional shielding expressed in half value layers (HVL) is required to
reduce the transmission from 1.6% to 0.1%?

A. 2 HVL
B. 3 HVL
C. 4 HVL
D. 5 HVL

Correct answer is C. RATIONALE: This item tests knowledge of the concept of HVL; 1.6% x
0.5 x 0.5 x 0.5 x 0.5 = 0.1%. Hence, 4 HVL are needed.
143. Which of the following statements about the use of chemotherapy in addition to radiation
therapy for squamous cell carcinoma of the head and neck is true?

A. Neoadjuvant chemotherapy is superior to concomitant chemotherapy.


B. Polychemotherapy is more effective than monochemotherapy.
C. The effect of chemotherapy on patient survival significantly decreases with
increasing age.
D. The absolute benefit of concurrent chemotherapy is less than 5%.

Correct answer is C. RATIONALE: Based on the recently updated meta-analysis of


chemotherapy in head and neck cancer (MACH0NC), concomitant chemotherapy was most
effective when compared to neoadjuvant and adjuvant chemotherapy with an absolute survival
benefit of 8% at 5 years. No significant difference was seen between monochemotherapy and
polychemotherapy. For the effect of chemotherapy on survival by covariate values (sex, age,
performance status, tumor stage, nodal stage, overall stage, and tumor site), the only significant
interaction was a decreasing effect of chemotherapy with increasing age (test for trend,
p=0.003). REFERENCE: International Journal of Radiation Oncology, Biology, Physics (Int J
Radiat Oncol Biol Phys). 2007;69:112-114.

144. Which of the following sites of pathologic bone fracture occurs most commonly
secondary to metastatic cancer, regardless of the primary tumor site?

A. Humerus
B. Acetabulum
C. Femoral shaft
D. Subtrochanteric femur

Correct answer is D. RATIONALE: Option A: 17%; option B: 8.5%; option C: 10%; option D:
21%. REFERENCES: Mirels H. Metastatic disease in long bones: a proposed scoring system.
Clinical Orthopedics. 1989;249-256. Gunderson & Tepper, eds. Clinical Radiation Oncology. 2nd
edition. 2007;437- 449.

145. Which of the following statements about bone metastases secondary to breast cancer is
true?

A. Approximately 15% of patients have osteoblastic lesions.


B. Samarium is equally effective for osteolytic and osteoblastic lesions.
C. The 5-year survival rate in patients with isolated bone metastases is approximately
65%.
D. Pamidronate (Aredia) administered for lytic bone metastases does not affect the
subsequent risk of pathologic fractures.

Correct answer is A. RATIONALE: Options A, B, and C: Five-year survival with metastatic


disease to the bone only is 40%. Samarium is effective only with blastic lesions. Approximately
15% of patients with breast cancer have osteoblastic metastatic disease. Patients with lytic
bone metastases were given pamidronate IV q. 4 weeks x 24 cycles had a skeletal complication
rate of 56% compared with 67% in the control group. The most common skeletal complication
was pathologic fracture, and this occurred in 45% of patients receiving pamidronate and in 55%
of the placebo patients (p.054). REFERENCES: Harris, Lippman, Morrow, and Osborne.
Diseases of the Breast. 3rdedition. 2004;1001-1159. Theriault, et al. Pamidronate reduces
skeletal morbidity on women with advanced breast cancer and lytic bone lesions: a randomized
placebo-controlled trial. Journal of Clinical Oncology (JCO). 1999;17(3):846-854.
146. Which of the following treatment modalities should be recommended for a patient with a
stage II, centrally located NSCLC that is medically inoperable?

A. Conventional fractionated radiation therapy (66 Gy in 33 fractions)


B. Stereotactic body radiation therapy (60 Gy in 3 fractions)
C. Radiofrequency ablation
D. Cryotherapy

Correct answer is A. RATIONALE: The central location of the cancer makes this patient a poor
candidate for stereotactic body radiation therapy (SBRT), which would cause a higher rate of
airway stenosis. Radiofrequency ablation has not been established as definitive therapy.
Cryotherapy can be useful for palliation of endobronchial disease.

147. Which of the following statements about the behavior of tumor cells after exposure to
ionizing radiation is true?

A. The presence of chromosomal aberrations within 24 hours of irradiation is a


predictor of apoptotic cell death.
B. Apoptosis within 3 days after irradiation is a reliable surrogate marker for clonogenic
survival.
C. Clonogenic cell survival is defined as the ability of cells to form colonies of at least
50 cells.
D. The "gold standard" for measuring cellular radiation sensitivity is the ability of cells to
proliferate for 5 days following irradiation.

Correct answer is C. RATIONALE: Clonogenic survival assays are considered the "gold
standard" for measuring cellular radiation sensitivity, but they take at least a week to complete,
and often, twice that or more. Shorter-term survival assays, however, although informative in
and of themselves and more convenient in some situations, typically do not correlate well with
clonogenic survival.

148. What percent of pituitary adenomas are secretory tumors?

A. 10%
B. 30%
C. 50%
D. 70%

Correct answer is D. RATIONALE: The majority of pituitary adenomas (70%) are secretory.

149. A poor overall prognosis is associated with women who have urethral carcinoma
involving the:

A. meatus.
B. distal urethra.
C. proximal urethra.
D. periurethral glands of Skene.

Correct answer is C. RATIONALE: Tumors of the distal urethra and meatus are typically early
stage and superficially invasive in women with urethral carcinoma. In contrast, tumors arising in
the proximal urethral typically are more deeply invasive and associated with a poorer outcome.
150. Before thermoluminescent materials can be read, they must be:

A. cooled.
B. heated.
C. charged.
D. discharged.

Correct answer is B. RATIONALE: Thermoluminescent materials use heat to stimulate the


emission of light from a molecular or crystal matrix following its exposure to ionizing radiation as
a means of quantifying dose. Therefore, in order to read a TLD, the material must be heated.

151. Hypoxic cells in tumors are:

A. resistant to neutron irradiation.


B. resistant to photon irradiation.
C. sensitive to proton irradiation.
D. sensitive to photon irradiation.

Correct answer is B. RATIONALE: Relative to aerobic cells, hypoxic cells are more resistant to
low-LET types of ionizing radiation (X-rays and protons), but equally sensitive to high-LET types
of ionizing radiation (neutrons). REFERENCE: Hall and Giaccia. Radiobiology for the
Radiologist. 6th edition, Chapter 7.

152. Loss of heterozygosity of chromosomes 1p/19q in patients with oligodendroglioma does


NOT predict which of the following outcomes?

A. Radiographic response to chemotherapy


B. Overall survival after radiation therapy alone
C. Overall survival after chemotherapy alone
D. Tumor resectability

Correct answer is D. RATIONALE: 1p/19q LOH predicts better response and survival after
both chemotherapy and radiation therapy. The ability to completely resect a tumor is not
predicted by 1p/19q status.

153. Which of the following statements about the use of cetuximab for squamous cell
carcinoma of the head and neck is true?

A. It should be given after completion of radiation therapy.


B. It should be initiated simultaneously with radiation therapy.
C. A loading dose is required 1 week before beginning radiation therapy.
D. It is more effective when given every 3 weeks rather than weekly.

Correct answer is C. RATIONALE: A loading dose of 400 mg/M2 should be given 1 week
before beginning radiation therapy to saturate the EGFR, and then given weekly during the
course of radiation therapy. REFERENCE: Bonner, et al. New England Journal of Medicine
(NEJM). 354(6):567-8.
154. Which of the following advantages is associated with proton therapy versus photon
therapy?

A. Proton beams have a sharper penumbra.


B. Proton beams have a higher LET.
C. Proton therapy has a lower integral dose.
D. Proton therapy uses higher energy beams.

Correct answer is C. RATIONALE: Proton beams typically deliver approximately less than half
of the integral dose compared to photon beams for the same target dose.

155. Which of the following outcomes was demonstrated by the GETUC-01 phase III trial,
which compared radiation therapy to the prostate alone versus whole-pelvis radiation
therapy for patients with prostate cancer?

A. Whole-pelvic radiation therapy improved progression-free survival for high-risk


patients.
B. Whole-pelvis irradiation decreased lymph node failure.
C. Whole-pelvis radiation therapy resulted in greater chronic GU and GI toxicity.
D. No improvement in progression-free survival was observed.

Correct answer is D. RATIONALE: The result of this phase III trial published in the Journal of
Clinical Oncology helps to define the role of pelvic irradiation for patients who have clinically
localized prostate cancer with clinically negative lymph nodes. This trial showed no survival
benefit with whole-pelvic irradiation when compared to prostate irradiation alone.

156. According to the Lung Intergroup 0139 trial, which of the following outcomes was
demonstrated in the trimodality arm for patients with stage IIIA NSCLC?

A. There was a statistically significant improvement in disease-free and overall survival.


B. There was a statistically significant improvement in local control.
C. A complete resection was achieved in only 50% of patients.
D. The trimodality treatment approach offered the best survival rate when a
pneumonectomy was incorporated.

Correct answer is B. RATIONALE: Seventy-one percent of patients had a complete resection.


The local relapse rate after trimodality treatment was significantly improved (10% vs. 22%), and
the disease-free survival rate also was significantly improved. There was a trend for increased
5-year overall survival associated with the trimodality approach; however, that is not optimal
when a pneumonectomy is required due to the increase in mortality rates.

157. According to the GOG (Creasman) trial, what is the risk for pelvic lymph node
metastasis in patients with stage I, grade 2 endometrial adenocarcinoma invading the
middle one third of the myometrium?

A. <5%
B. 08% to 15%
C. 18% to 25%
D. 28% to 35%

Correct answer is B. REFERENCE: Creasman. GOG trial. Cancer. 1987;60:2035-2041.


These tables provide a rationale for which patients need pelvic radiation for coverage of pelvic
lymph nodes if not dissected.
158. The highest risk of mental retardation occurs when an embryo or fetus is irradiated
during which stage of gestation?

A. 00 to 3 weeks
B. 04 to 7 weeks
C. 08 to 15 weeks
D. 16 to 25 weeks

Correct answer is C. RATIONALE: There is a high risk of mental retardation, approaching 40%
risk per Sievert, if the fetus is irradiated between 8 and 15 weeks of gestation. The risk is 4
times lower – although still elevated – if irradiation occurs between 15 and 25 weeks.
REFERENCE: Hall and Giaccia. Radiobiology for the Radiologist. 6th edition, Chapter 12.

159. Based on the results of the United Kingdom START A and START B trials, which of the
following whole-breast radiation fractionation schedules was used?

A. 41.6 Gy delivered in 13 fractions, 3.2 Gy per fraction 4 days each week for 3 weeks
B. 40 Gy delivered in 15 fractions, 2.67 Gy per fraction 5 days each week for 3 weeks
C. 40 Gy delivered in 10 fractions, 4 Gy per fraction 5 days every 2 weeks for 4 weeks
D. 39 Gy delivered in 13 fractions, 3 Gy per fraction 5 days each week for 2½ weeks

Correct answer is B. RATIONALE: The START A and B trials compared different whole-breast
hypofractionation schedules to 50 Gy in 2 Gy fractions over 5 weeks. The START A trial used
41.6 Gy or 39 Gy in 13 fractions but used 5 fractions every 2 weeks to maintain an elapsed time
of 5 weeks. The START B trial used 40 Gy in 15 fractions 5 days a week over an elapsed time
of 3 weeks. REFERENCES: Lancet Oncology. 2008;9:331-41. Lancet. 2008;371:1098-107.

160. Which of the following isotopes is currently used for permanent seed prostate implants?
103
A. Pd
192
B. Ir
60
C. Co
131
D. I

Correct answer is A. RATIONALE: 103Pd is a commonly used radioactive source for permanent
prostate seed implant. Its relatively short half-life (17 days) and low average energy (21 keV)
make it a good candidate for permanent seed implants. The energies of the emitted photons for
192
Ir and 60Co are much higher, and as a result, are more penetrating, making them impractical
for permanent implants. P-32 is a pure beta emitter and has a short range of penetration.
161. Which of the following statements about chondrosarcoma is true?

A. Thirty percent of chondrosarcomas are high-grade lesions.


B. Primary lesions can develop from a preexisting chondroid lesion.
C. Primary chondrosarcomas are typically peripheral, arising from the surface of the
bone.
D. A favorable prognosis is strongly correlated with histological grade and adequate
surgical excision.

Correct answer is D. RATIONALE: Option A: 5% to 10% of chondrosarcomas are high-grade


lesions. These lesions have a high potential to metastasize. Option B: Primary lesions are not
associated with preexisting lesions. Secondary lesions develop from preexisting chondroid
lesions, such as enchondroma or osteochondroma. Option C: Chondrosarcomas are classified
as central when they arise from the medullary cavity and peripheral when they arise from the
surface of bone. Primary chondrosarcomas are nearly always central; secondary
chondrosarcomas can be central or peripheral. Option D: Prognosis is generally favorable and
is strongly correlated histologic grade and adequacy of surgery. REFERENCE: Chow W.A.
Update on chondrosarcomas. Current Opinion in Oncology. 2007;19:371-376.

162. According to the phase III GOG 92 (Sedlis) randomized trial, adjuvant radiation therapy
for patients with high-risk, node-negative cervical cancer consisted of:

A. pelvic irradiation alone.


B. vaginal brachytherapy alone.
C. pelvic irradiation plus vaginal brachytherapy.
D. pelvic plus paraaortic lymph node irradiation.

Correct answer is A. RATIONALE: GOG Trial 92 randomized patients with high-risk node-
negative cervical cancer following radical surgery to either pelvic irradiation alone or no further
therapy. All patients had at least two of the following risk factors: >1/3 stromal invasion, capillary
space involvement, and large clinical tumor diameter. REFERENCE: Sedlis, et al.
Gynecological Oncology. 1999;73:177.

163. Which of the following radiation fractionation schedules used to treat bone metastases
provides the highest biological effective dose (BED) for symptomatic relief?

A. 08 Gy in 1 fraction
B. 20 Gy in 5 fractions
C. 30 Gy in 10 fractions
D. 40.5 Gy in 15 fractions

Correct answer is D. RATIONALE: Summary of prospective trials for radiation therapy used for
treatment of painful osseous metastases suggests schedules with higher biological effective
doses (BED) resulted in better pain relief and reduced the need for retreatment. REFERENCE:
Gunderson & Tepper, eds. Clinical Radiation Oncology. 2nd edition, 2007;443-445.
164. Classic Kaposi sarcoma (occurring in an elderly male of Mediterranean ancestry) most
commonly is observed first on the skin of the:

A. lower legs.
B. palms.
C. trunk.
D. face.

Correct answer is A. RATIONALE: The initial distribution of classic Kaposi sarcoma (KS)
virtually always is on the lower legs (followed occasionally by the palms and rarely the mucosa
of the oral cavity); whereas there is far less predictability for other types of KS.

165. The extrinsic death receptor and intrinsic mitochondrial apoptotic pathways converge on:

A. caspase-2.
B. caspase-3.
C. caspase-8.
D. caspase-9.

Correct answer is B. RATIONALE: Caspase-3 is an effector caspase whose activation by


either the extrinsic or intrinsic apoptotic signaling pathways commits the cell to programmed
suicide.

166. Which of the following genetic alterations is present in the majority of patients with
pancreatic ductal adenocarcinomas?

A. MDM2 amplification
B. K-ras gene mutation
C. Platelet-derived growth factor receptor mutation
D. Deletion of the PTEN tumor suppressor gene

Correct answer is B. RATIONALE: Targeted-based therapies are becoming a critical


component of oncology therapy, and in some sites, such as the colon, the mutation status of K-
ras is predictive of a therapeutic response.

167. The anisotropy function in the brachytherapy dose calculation for a line source accounts
for the:

A. difference in the dose around the source due to self-absorption.


B. decay of the source activity between the date of calibration and the date of implant.
C. falloff of dose as the inverse of the distance squared.
D. radial dependence of photon scattering and attenuation in a medium.

Correct answer is A. RATIONALE: According to AAPM TG report 43, the anisotropy function,
F(r, θ), accounts for the variation in dose as a function of polar angle relative to the transverse
plane of the source.
168. What is the AJCC stage for a patient who has prostate cancer with invasion of the
ipsilateral seminal vesicles?

A. T2b
B. T3a
C. T3b
D. T4

Correct answer is C. RATIONALE: Straight-forward question on AJCC staging of prostate


cancer. REFERENCE: AJCC Cancer Staging Manual, 6th edition.

169. Which of the following radiation sensitizers combined with postoperative radiation
therapy has been shown to improve the outcome of high-risk patients with advanced
head and neck cancer?

A. Cisplatin every 3 weeks


B. Mitomycin and cisplatin every 3 weeks
C. Cetuximab weekly
D. Cisplatin weekly

Correct answer is A. RATIONALE: This item is based on the EORTC and RTOG high-risk
postoperative trials (ie, RTOG 95-01 and EORTC 22931).

170. Which of the following proteins is a stimulator of angiogenesis?

A. Thrombospondin-1 (TSP-1)
B. Tissue inhibitor of matrix metalloproteinases-3 (TIMP-3)
C. Basic fibroblast growth factor (bFGF)
D. Angiopoeitin-1 (Ang-1)

Correct answer is C. RATIONALE: Basic fibroblast growth factor (bFGF) is one of several
known promoters of angiogenesis (the best known is VEGF, vascular endothelial growth factor),
whereas the other proteins listed are inhibitors of angiogenesis.

171. What is the expected incidence of inguinal lymph node metastasis in a patient with a
2-cm squamous cell carcinoma of the vulva with a depth of invasion of 5.5 mm?

A. 06%
B. 08%
C. 22%
D. 37%

Correct answer is D. RATIONALE: Pooled data from vulvar series notes the following
correlation of incidence of groin lymph node metastases in patients with primary tumors of 2 cm
or less and the following depths of invasion: 1 mm or less = 0%; 1.1 to 2 mm = 6.6%; 2.1 to 3
mm = 8.2%; 3.1 to 4 mm = 22.0%; 4.1 to 5 mm = 25.0; and >5 mm = 37.5%.
172. After electrons exit a linear accelerator, which of the following is the dominant process
by which a clinical photon beam is produced?

A. Photoelectric effect
B. Compton scattering
C. Electron scattering
D. Bremsstrahlung

Correct answer is D. RATIONALE: Bremsstrahlung ("braking radiation") in the linear-


accelerating (linac) target is the dominant effect producing photons from electrons.

173. Which of the following agents is most effective when combined with cisplatin for
treatment of mesothelioma?

A. Pemetrexed
B. Gemcitabine
C. Navelbine
D. Docetaxel

Correct answer is A. RATIONALE: In a randomized phase III trial, pemetrexed plus cisplatin
was associated with improved survival compared with cisplatin alone for patients with malignant
pleural mesothelioma (MPM). Median time to progression was significantly longer in the
pemetrexed/cisplatin arm: 5.7 months versus 3.9 months (P =.001). Response rates were
41.3% in the pemetrexed/cisplatin arm versus 16.7% in the control arm (P <.0001).
REFERENCE: Journal of Clinical Oncology. July 15, 2003;21(14):2636-44.

174. According to the German Rectal Cancer Study Group (Sauer) trial, what was the
pathologic complete response rate to preoperative chemoradiation for patients who have
rectal cancer?

A. 08%
B. 18%
C. 28%
D. 38%

Correct answer is A. RATIONALE: The German trial is one of the most important recent
studies on rectal cancer. The pathologic complete response (CR) rate was reported.
REFERENCE: Sauer, et al. New England Journal of Medicine (NEJM). 2004;351:1731-40.

175. Temsirolimus (Torisel) is a recently approved targeted therapy for metastatic renal cell
carcinoma that inhibits which of the following types of protein?

A. Vascular endothelial growth factor (VEGF)


B. Epidermal growth factor receptor (EGFR)
C. Mammalian target of rapamycin (mTOR)
D. Platelet-derived growth factor (PDGF)

Correct answer is C. RATIONALE: Temsirolimus is a mammalian target of rapamycin (mTOR)


inhibitor, which has been shown to be efficacious for metastatic renal cell carcinoma.
176. Which of the following signaling pathways is thought to be responsible for EGFR
overexpression in human cancer cells?

A. IKκB → NFκB
B. Ras → JNK
C. Ras → Raf → MAPK
D. PI3K → AKT → mTOR

Correct answer is C. RATIONALE: The epidermal growth factor receptor (EGFR) is one of a
family of four closely related transmembrane receptors that use tyrosine kinase activity as the
signal transduction trigger. EGFR is frequently overexpressed in human epithelial cancers, and
as such, constitutes an attractive target for cancer prevention and treatment. Once the tyrosine
kinase is triggered, the signal is transduced through the Ras → Raf → MAPK pathway, which
regulates several processes involved in tumor cell survival and proliferation, including the
inhibition of apoptosis, increased radiation and chemotherapy resistance, stimulation of
angiogenesis, promotion of metastasis and, ironically, further overexpression and activation of
EGFR.

177. FIGO stage IIIA is associated with cervical cancer that extends to which of the following
anatomic sites?

A. Pelvic sidewall
B. Lower uterine segment
C. Upper one third of the vagina
D. Lower one third of the vagina

Correct answer is D. RATIONALE: The FIGO staging system in cervical cancer divides stage
III into IIIA lesions (i.e., tumor involves the lower one third of the vagina with no extension to the
pelvic sidewall) and IIIB (i.e., tumor extends to the pelvic wall and/or causes hydronephrosis or
nonfunctioning kidney. Tumors extending into the upper two thirds of the vagina are stage IIA.
Extension into the lower uterine segment is not included in the FIGO staging of cervical cancer.

178. Based on a pooled analysis, which of the following statements about the addition of
high-dose cisplatin with radiation therapy in postoperative patients with high-risk head
and neck cancer is FALSE?

A. It improves overall survival.


B. It improves locoregional control.
C. It increases disease-free survival.
D. It decreases distant metastasis.

Correct answer is D. RATIONALE: Based on high-risk EORTC and RTOG postoperative trials,
there is no difference in distant metastasis between the two arms.
179. For quality assurance of a linear accelerator, which of the following variations in
x-ray/electron beam output constancy would require action to be taken?

A. 5% daily variation
B. 3% daily variation
C. 1% daily variation
D. 1% monthly variation

Correct answer: ABCD. RATIONALE: According to AAPM TG40, the suggested action level
for x-ray/electron beam output constancy is 3% for a daily check and 2% for a monthly variation
of 2% for a monthly output check.

180. Based on the Masaoka staging system for thymoma, which of the following stages is
associated with macroscopic invasion into adjacent organs?

A. IIA
B. IIB
C. III
D. IV

Correct answer is C. RATIONALE: This item refers to the Masaoka staging system.
Macroscopic invasion into adjacent organs is classified as stage III disease.

181. What would be the most appropriate adjuvant therapy after nephroureterectomy for a
patient with multifocal, high-grade ureteral carcinoma with one positive lymph node and
negative margins?

A. Chemoradiation
B. External-beam radiation therapy
C. Intraoperative radiation therapy
D. Observation

Correct answer is D. RATIONALE: There are no prospective randomized trials that support
adjuvant radiation therapy in this setting, especially in the setting of a negative surgical margin.

182. Which of the following syndromes is associated with a high risk for the development of
cancer and is caused by biallelic mutations in the BRCA2 gene?

A. Fanconi anemia
B. Bloom's syndrome
C. Nijmegen breakage syndrome
D. Ataxia telangiectasia

Correct answer is A. RATIONALE: Complementation analysis of cell lines from different


patients with Fanconi anemia has led to the description of at least 12 groups, named FA-A, B,
C, D1, D2, E, F, G, I, J, L, and M, with the corresponding genes named as FANCA-FANCM.
FANCD1 is BRCA2, and monoallelic mutations in BRCA2 cause susceptibility to breast and
other cancers. Ataxia telangiectasia, Bloom's syndrome, and Nijmegen breakage syndrome are
caused by mutations of ATM, BLM, and NBS1, respectively. REFERENCE: Green AM and
Kupfer GM. Fanconi anemia. Hematology/Oncology Clinics of North America. 2009;23:193-214.
183. Which of the following terms applies to a clinical trial in which each group of subjects
receives two or more treatments but in different sequences?

A. Cross-over study
B. Cross-sectional study
C. Cross-validation procedure
D. Cross-product ratio

Correct answer is A. RATIONALE: A cross-over study is applied to a clinical trial in which each
group of subjects receives two or more treatments, but in different sequences. A cross-sectional
study is an observational study that examines a characteristic in a set of subjects at one point in
time. A cross-validation is a procedure applying the results of an analysis from one sample of
subjects to a new sample of subjects to evaluate how well they generalize. A cross-product ratio
is similar to relative risk.

184. Which of the following radioisotopes is most commonly used in HDR brachytherapy for
patients who have cervical cancer?
125
A. I
137
B. Cs
192
C. Ir
198
D. Au

Correct answer is C. RATIONALE: Several isotopes are currently available for brachytherapy
in patients with cervical cancer. Iridium-192 is the most popular HDR source; cesium-137 is the
most popular source used in LDR brachytherapy.

185. Which of the following factors would increase the percent depth dose the most due to an
increase in the SSD?

A. Change in lateral scatter


B. Change in field size
C. Inverse square law
D. Beam hardening

Correct answer is C. RATIONALE: Although the PDD will be affected due to an increase in
SSD by a change in scatter, field size, and inverse square law, the largest influence will be the
due to the increased distance from the source (option C). In this scenario, the PDD will increase
by the inverse of the distance squared. Beam hardening will not be influenced if the depth in
tissue remains constant and we consider the attenuation in air to be negligible.

186. Which of the following forms of leukemia is most appropriately treated with imatinib
(STI571)?

A. Acute lymphocytic
B. Acute myelogenous
C. Chronic lymphocytic
D. Chronic myelogenous

Correct answer is D. RATIONALE: Imatinib is a tyrosine kinase inhibitor that is indicated for GI
stromal tumors (GIST) and chronic myelogenous leukemia (CML). REFERENCE: FDA package
insert.
187. A clinical trial must have which of the following characteristics?

A. A control group
B. A prospective plan that involves human subjects
C. A predetermined, fixed sample size
D. Treating physicians or patients blinded to treatment received

Correct answer is B. RATIONALE: Some clinical trials have only a single arm, with data from
an already completed study as the (historical) control. The control arm, if there is one, may
receive no treatment, a sham treatment, or an alternative, active treatment. Sample size may be
altered during the course of a trial of any phase based on outcomes observed during the trial.
Blinding can help to reduce treatment-related bias, but it is not essential.

188. According to Michalowski's tissue classification system, bone marrow would be


classified as which of the following types of tissue?

A. Consolidated
B. Flexible
C. Hierarchical
D. Mature

Correct answer is C. RATIONALE: Michalowski's system classifies normal tissues according to


whether they contain stem cells, transit-amplifying cells and differentiated, functioning cells, or
whether they appear to only (or mostly) contain mature, functioning cells. Type H or
"hierarchical" tissues contain both stem, transit, and mature cells, with each type discretely
located in its own anatomically or histologically distinct "compartment" within the tissue.
Examples include the bone marrow, the intestinal epithelium, and the epidermis of the skin.
Type F or "flexible" tissues show no strict hierarchy or histologically identifiable compartments,
but only appear to be composed of differentiated, functioning cells (or in a few cases, of mature
cells that can revert to stem cells and back in response to an injury). Examples include the
central nervous system, muscle, heart, and liver. REFERENCE: Hall and Giaccia. Radiobiology
for the Radiologist. 6th edition, Chapter 19.

189. Which of the following gene mutations has been shown to predict a patient's response to
cetuximab therapy when used as second-line treatment of metastatic colorectal cancer?

A. Akt
B. K-ras
C. EGFR
D. VEGF

Correct answer is B. REFERENCE: Lièvre A, Bachet JB, Le Corre D, Boige V, Landi B, Emile
JF, Côté JF, Tomasic G, Penna C, Ducreux M, Rougier P, Penault-Llorca F, Laurent-Puig P. K-
ras mutation status is predictive of response to cetuximab therapy in colorectal cancer. Cancer
Research. Apr 15, 2006;66(8):3992-5.
190. According to the NRC, a female radiation worker is considered pregnant:

A. from the date of conception.


B. when she begins to display visible signs of pregnancy.
C. when she verbally declares her pregnancy to her employer.
D. when she declares her pregnancy in writing to her employer.

Correct answer is D. RATIONALE: According to 10 CFR 20.1003, a woman must declare her
pregnancy to her employer/licensee, in writing, along with the estimated date of conception.

191. Which of the following types of leukemia has the highest incidence?

A. Acute myelogenous
B. Acute lymphocytic
C. Chronic myelogenous
D. Chronic lymphocytic

Correct answer is D. RATIONALE: It's important to know which type of leukemia presents most
frequently. Chronic lymphocytic leukemia (CLL) is most common.

192. Bladder conservation therapy involving TURBT and chemoradiation would be most
appropriate to recommend for patients with:

A. T1 multifocal TCC.
B. T2a TCC of the lateral wall of the bladder.
C. T2b TCC with concurrent carcinoma in-situ.
D. T3b TCC and hydronephrosis.

Correct answer is B. RATIONALE: Option B depicts the most appropriate patient to consider
for TURBT and chemoradiation. The other patients have relative contraindications or factors
that have been shown to have a negative impact on outcome in prospective trials.

193. Which of the following characteristics is associated with a diagnosis of retinoblastoma?

A. Eighty percent of children are diagnosed in the first year of life.


B. Approximately 80% of children with bilateral disease have a family history of the
disease.
C. Children who present with bilateral disease are usually 2 years older than those who
present with unilateral involvement.
D. It is rare in children older than 6 years of age.

Correct answer is D. RATIONALE: Eighty percent of children with retinoblastoma are


diagnosed before 3 years of age. There is a family history of the disease in approximately 20%
of children diagnosed with bilateral retinoblastoma. Patients with bilateral disease usually
present at a younger age (14–16 months) than patients with unilateral disease (29–30 months).
The diagnosis of retinoblastoma in children 6 years or older is extremely rare. REFERENCE:
Chintagumpala, et al. The Oncologist. October 2007;12(10):1237-1246.
194. Which of the following statements about pleural mesothelioma is true?

A. The epithelial subtype has the worst prognosis.


B. It does not spread to the mediastinal lymph nodes.
C. Thirty percent of cases are associated with a history of asbestos exposure.
D. Combined modality therapy results in an overall median survival of 14 to 16 months.

Correct answer is D. RATIONALE: Patients with mixed or sarcomatous cell disease or


mediastinal lymph node involvement have a worse overall survival. Seventy percent to eighty
percent of mesothelioma cases were associated with a history of asbestos exposure. Patients
who receive combined modality treatment for mesothelioma have an overall median survival of
about 14 to 16 months.

195. In eukaryotic cells, chromosomes are duplicated during which stage of the cell cycle?

A. Anaphase
B. Interphase
C. Metaphase
D. Prophase

Correct answer is B. RATIONALE: Chromosomal material is replicated during the S phase of


the cell cycle, a part of interphase that includes the G1, S, and G2 phases. The other options are
sub-phases of mitosis (M phase), that portion of the cell cycle when the already-duplicated
chromosomes are evenly segregated into daughter cells.

196. Which of the following statements about patients who have locoregional recurrence of
breast cancer after modified radical mastectomy with or without systemic and/or
endocrine therapy and no radiation therapy is true?

A. Median time to local recurrence is 3 years.


B. The majority of recurrences present as a skin rash over the chest wall.
C. Regional recurrence in the axillary, supraclavicular, and/or internal mammary lymph
nodes occurs in about 10% of cases.
D. The axillary lymph nodes are the most common site of regional lymph node
recurrence.

Correct answer is A. RATIONALE: Median time to postmastectomy recurrence is 3 years. The


majority of recurrences present as skin or subcutaneous nodules in the flaps around the
mastectomy scar. Regional recurrence in the axillary, supraclavicular, or internal mammary
lymph nodes occurs in 30% to 40% of cases with a local recurrence. The supraclavicular lymph
nodes are the most common site of regional recurrence and occur in 11% to 35% of patients.
REFERENCE: Harris, Lippman, Morrow, and Osborne. Diseases of the Breast. 3rd edition.
2004;1067.
197. The dose chamber is located between which of the following two objects in a
megavoltage linear accelerator?

A. Wedge tray and the block tray


B. X-ray production target and the patient
C. Electron gun and the accelerating waveguide
D. Klystron and the accelerating waveguide

Correct answer is B. RATIONALE: The dose chamber is located between the x-ray production
target and the patient in a megavoltage linear accelerator. It is used to monitor x-rays, and
therefore, the dose delivered to the patient.

198. Approximately what percent of patients with NSCLC present with stage III disease?

A. 10%
B. 20%
C. 30%
D. 50%

Correct answer is C. RATIONALE: According to AJCC 2002, stage I = 25%, stage II = 7%,
stage III = 32%, and stage IV = 36%. Therefore, the majority of patients will have advanced-
stage lung cancer at diagnosis. REFERENCE: AJCC Cancer Staging Manual, 6th edition.

199. The concurrent addition of cetuximab to radiation therapy for squamous cell carcinoma
of the head and neck significantly increases:

A. oral mucositis.
B. overall survival.
C. disruption of radiation therapy.
D. treatment-related diarrhea.

Correct answer is B. RATIONALE: The concurrent use of cetuximab with radiation therapy
demonstrated significantly improved median survival, overall survival, and locoregional control.
It also resulted in acneiform rash, but not oral mucositis. REFERENCE: Bonner, et al. New
England Journal of Medicine (NEJM). 354:6.567-8.

200. Which of the following types of tumor is most often associated with EGFR amplification?

A. Anaplastic oligodendroglioma
B. Low-grade glioma
C. De novo glioblastoma
D. Ependymoma

Correct answer is C. RATIONALE: EGFR-based strategies are currently being employed in


clinical trials for glioblastoma. EGFR amplification is noted in de novo glioblastoma, but not in
those glioblastomas that arise from lower-grade neoplasms. These tumors appear to be
molecularly distinct and may respond to different treatment strategies.
201. Which of the following statements about the incidence of esophageal cancer in the
United States is true?

A. Squamous cell carcinomas are becoming more common.


B. There has been a six-fold increase in esophageal adenocarcinoma over the last
25 years.
C. The incidence of esophageal cancer is decreasing due to screening programs for
Barrett's disease.
D. The proximal esophagus is the most common location of esophageal cancer.

Correct answer is B. REFERENCE: Pohl H and Welch HG. The role of over diagnosis and
reclassification in the marked increase of esophageal adenocarcinoma incidence. Journal of
National Cancer Institute. January 19, 2005;97(2):142-6.

202. What is the maximum permissible dose equivalent to an embryo or fetus that is allowed
during a radiation worker's entire pregnancy?

A. 00.5 mSv
B. 01.0 mSv
C. 05.0 mSv
D. 10 mSv

Correct answer is C. RATIONALE: The NRC Title 10, which applies to organizations that
receive a license from the NRC to use radioactive materials or operate nuclear facilities,
stipulates that the dose equivalent to the embryo/fetus during an entire pregnancy, due to the
occupational exposure of a declared pregnant woman, should not exceed 5 mSv. (Similarly,
NCRP Report 116 recommends a monthly limit of 0.5 mSv to the embryo/fetus once pregnancy
is declared.) REFERENCES: NRC Title 10, Code of Federal Regulations (US NRC 10 CFR
§20.1208). NCRP Report 116.

203. What is the expected risk of cancer recurrence in a patient with a 10-cm stage IA
dysgerminoma of the ovary managed by unilateral salpingo-oophorectomy and no
further treatment?

A. 05% to 10%
B. 15% to 25%
C. 30% to 50%
D. 75% to 85%

Correct answer is B. RATIONALE: This point is emphasized since many clinicians feel that a
patient with a well-staged stage IA dysgerminoma can be observed after unilateral salpingo-
oophorectomy, regardless of the size of the primary tumor, if fertility preservation is desired.
Careful follow-up is required, because as many as 15% to 25% of patients will experience a
recurrence (pooled data). Traditionally, most of these women received radiation. Current
thinking is that because of the tumor's chemosensitivity, virtually all patients with a recurrence of
dysgerminoma can be successfully salvaged if adequate follow-up and early detection can be
accomplished.
204. A craniospinal axis treatment requires a compensator for the spinal field. The field width
is 8.0 cm on the patient's skin at 130-cm SSD, and the compensator tray is at 67 cm.
What should be the minimum field width for the compensator?

A. 04.1 cm
B. 05.4 cm
C. 06.2 cm
D. 10.4 cm

Correct answer is A. RATIONALE: By performing a similar triangles calculation, the width of 8


cm is reduced by the ratio of 67/130.
205. Which of the following environmental exposures is a risk factor for the development of
leukemia?

A. Chlorine
B. Benzene
C. Bromine
D. Radon

Correct answer is B. RATIONALE: It is important to be aware of risk factors for


screening/counseling family members.

206. Which of the following diagnoses is most appropriate for a 16-year-old patient who has
Parinaud's syndrome and an elevated serum α-fetoprotein level?

A. Hepatoblastoma
B. Pineal germ cell tumor
C. Hypothalamic germinoma
D. Choriocarcinoma

Correct answer is B. RATIONALE: Parinaud's syndrome is an upward gaze palsy that can be
caused by a pineal tumor. An elevated α-fetoprotein (AFP) level is suggestive of a germ cell
tumor, particularly a non-germinomatous germ cell tumor.

207. Based on the Massachusetts General Hospital (DeLaney) trial, what is the 5-year local
control rate after radiation therapy is administered to patients with soft tissue sarcoma
and positive margins after wide excision?

A. 76%
B. 54%
C. 33%
D. 12%

Correct answer is A. RATIONALE: This large retrospective study showed an overall 5-year
local control rate of 76% in patients with soft tissue sarcoma and positive margins treated with
excision and irradiation. REFERENCE: DeLaney, et al. International Journal of Radiation
Oncology, Biology, and Physics (IJROBP). 2007;67:1460.
208. The Laws of Bergonié and Tribondeau state that normal tissues should be radiosensitive
if they contain a significant proportion of cells that are:

A. undifferentiated, with a low mitotic rate and a short mitotic future.


B. undifferentiated, with a high mitotic rate and a long mitotic future.
C. well differentiated, with a low mitotic rate and a short mitotic future.
D. well differentiated, with a high mitotic rate and a long mitotic future.

Correct answer is B. RATIONALE: In the early days of radiation therapy (1906), two French
radiobiologists, Drs. Bergonié and Tribondeau, attempted to categorize tissues with respect to
their relative radiation sensitivities based on the proliferative and differentiation status of critical
cells within those tissues. Those tissues found to be enriched in relatively undifferentiated cells
with high proliferative potential and long proliferative life spans were considered to be the most
"radiosensitive" (preferred term by today's standards would be "radioresponsive").
REFERENCE: Mettler and Upton. Medical Effects of Ionizing Radiation. 3rd edition, Chapter 1
(section on “Radiation Effects on Cell and Organ Systems”).

209. What percent of patients diagnosed with primary ocular lymphoma will have CNS
involvement within 1 year of diagnosis?

A. 05% to 10%
B. 20% to 30%
C. 50% to 80%
D. >90%

Correct answer is C. RATIONALE: Fifty to eighty percent of patients with primary ocular
lymphoma will have CNS involvement at a median rate of 9 months. Thus, radiation therapy
must include the intracranial contents and both eyes to ensure coverage of high-risk areas.

210. Which of the following statements about proton radiation therapy is true?

A. Like photons, protons primarily interact via Compton interactions.


B. Like photons, protons may be produced with an electron linear accelerator.
C. Protons deliver their highest dose at the end of their range.
D. Protons have a significantly higher LET than photons.

Correct answer is C. RATIONALE: Unlike photons, protons have a finite range in a medium
(Braggs peak). As such, proton therapy may be delivered such that the maximum energy loss of
the protons is delivered at the depth of the tumor, minimizing the dose to healthy tissue. Option
A is false because protons primarily interact via ionization. Although protons may be accelerated
with a linear accelerator (linac), option B is incorrect because they would not be accelerated
with an electron linac, but rather with a proton or deuteron linear accelerator. Lastly, option D is
incorrect because the LET of protons is similar to that of photons.

211. Which of the following histologies of thyroid cancer is most likely to take up iodine?

A. Hürthle cell carcinoma


B. Tall cell variant of papillary carcinoma
C. Anaplastic carcinoma
D. Follicular carcinoma

Correct answer is D. RATIONALE: It is very important to determine which histologies are likely
to take up iodine so appropriate treatment can be initiated.
212. A patient has anaplastic ependymoma with negative findings on CSF cytology and MRI
of the spine. What is the risk of CNS dissemination after gross total resection, followed
by radiation therapy to the primary site only?

A. <5%
B. 10% to 15%
C. 30% to 40%
D. 50% to 60%

Correct answer is B. RATIONALE: Although anaplastic histology was once considered an


indication for elective craniospinal radiation therapy (RT), the true risk of dissemination is very
modest (~10-15%) if the primary site is treated locally.

213. For most biological endpoints, which of the following statements best describes the
relationship between RBE and LET?

A. RBE increases as LET increases.


B. RBE decreases as LET increases.
C. RBE reaches a maximum value near LET =100 keV/μm.
D. RBE reaches a minimum value near LET =100 keV/μm.

Correct answer is C. RATIONALE: A graph of the relationship between RBE and LET is
generally bell-shaped, with the RBE increasing to a maximum at an LET of approximately 100
keV/µm and then decreasing with further increase in LET. REFERENCE: Hall and Giaccia.
Radiobiology for the Radiologist. 6th edition, Chapter 7 (Figures 7.6 and 7.7).

214. Which of the following disease states is included in the intergroup rhabdomyosarcoma
study (IRS) clinical group III classification?

A. Localized tumor with resected positive lymph nodes with microscopic residual
disease
B. Localized or locally extensive disease with gross residual disease after a biopsy
C. Localized disease with infiltration from the organ of origin to an adjacent structure
with complete resection
D. Distant metastatic disease present at diagnosis

Correct answer is B. RATIONALE: The IRS clinical group staging system refers to the extent
of disease, degree of resection, and the presence or absence of involved lymph nodes and/or
metastasis; whereas the TNM staging classification relies on the primary anatomic site first,
then on size of the primary disease, invasiveness of the primary disease, and presence or
absence of involved lymph nodes and/or metastasis.

215. Which of the following treatments would be most appropriate for a 50-year-old patient
who has Graves' disease with significant bilateral ophthalmopathy and diplopia?

A. 131I therapy
B. 40 Gy of fractionated radiation therapy to the orbits
C. 20 Gy of fractionated radiation therapy to the orbits
D. Systemic iodine chelation with immunosuppressive therapy

Correct answer is C. RATIONALE: Graves' ophthalmopathy can be treated with surgical


decompression, steroid treatment, or low-dose fractionated radiation therapy (20 Gy in 10 to 20
fractions) with a reasonable symptomatic improvement. Four randomized trials with radiation
therapy (RT) vs sham RT showed consistent marginal benefit.
216. According to AAPM TG 40, an independent monitor unit calculation of a treatment plan
should differ by no more than:

A. 02%.
B. 05%.
C. 10%.
D. undefined.

Correct answer is B. RATIONALE: According to AAPM TG 40, the independent calculation


should not differ by more than 5% from the treatment plan.

217. Which of the following visual-field defects is typically found in patients with
craniopharyngioma?

A. Central scotoma
B. Homonymous hemianopsia
C. Bitemporal hemianopsia
D. "Pie-in-the-sky" defect

Correct answer is C. RATIONALE: Bitemporal hemianopsia is the most common visual-field


deficit noted because of compression of the optic chiasm.

218. According to the PORTEC (Creutzberg) adjuvant endometrial trial, what percent of local
failures was limited to the vagina?

A. 90% to 100%
B. 70% to 80%
C. 50% to 60%
D. 30% to 40%

Correct answer is B. RATIONALE: Seventy-three percent of local failures were in the vagina.
This pattern of failure supports the rationale for using vaginal brachytherapy as adjuvant
treatment. REFERENCE: Creutzberg CL, van Putten WL, Koper PC, et al. Surgery and
postoperative radiotherapy versus surgery alone for patients with stage I endometrial
carcinoma: Multicenter randomized trial. PORTEC Study Group.

219. Which of the following radiation doses is most appropriate for treatment of a solitary
plasmacytoma involving the femur?

A. 25 Gy
B. 36 Gy
C. 45 Gy
D. 60 Gy

Correct answer is C. RATIONALE: It is important to know that the radiation dose for solitary
plasmacytoma of the femur is higher than that for myeloma. The content of this item is
somewhat controversial, since some may chose to use 36 Gy for treatment based on a more
recent analysis. However, the weight of the evidence supports the dose chosen (i.e., >40 Gy).
220. Which of the following statements about radiation carcinogenesis is true?

A. Radiation carcinogenesis is a deterministic effect.


B. The probability of radiation carcinogenesis increases with increasing dose above a
practical threshold of 500 mSv.
C. IMRT has the potential to reduce the risk of secondary cancers due to less low-dose
scatter to normal tissues.
D. Proton radiation therapy has the potential to reduce the risk of secondary cancers
due to the lack of an exit dose.

Correct answer is D. RATIONALE: In theory, proton radiation therapy has the potential to
reduce the risk of second cancers because less normal tissue is irradiated due to the lack of an
exit dose. It is important to bear in mind that radiation carcinogenesis is considered a stochastic
effect (not a deterministic one); therefore, there is no radiation dose, no matter how small, that
does not have some probability of causing a new cancer in previously irradiated normal tissue.

221. Assuming that total white blood cell (WBC) counts follow a normal distribution, what
would be the range of WBC counts for 50% of the individuals if the mean total WBC
count is 7600 cells/mm3 with a standard deviation of 1400 cells/mm3?

A. Between 6200 and 9000


B. Between 7400 and 7800
C. <6200 and >9000
D. <7600

Correct answer is D. RATIONALE: Since the WBC counts follow a normal distribution, 50% of
the population would be at or below the mean.

222. What is the fundamental characteristic of a sliding-window IMRT?

A. The physical compensator is slid into position.


B. The beam is on while the multileaf collimator leaves are moving.
C. The x-ray target is sliding while the beam is on.
D. The beam is on as the collimator is rotating.

Correct answer is B. RATIONALE: According to its definition, multileaf collimator (MLC) leaves
move continuously while the beam is on for a given treatment field in a sliding-window or
dynamic MLC IMRT. Although option A is a valid IMRT technique, it is not characteristic of a
sliding-window IMRT. Neither options C or D are currently used techniques for delivering IMRT
treatment plans.
223. Which of the following immunohistochemical staining patterns differentiates primary
bronchogenic carcinoma from metastatic adenocarcinoma of gastrointestinal origin?

A. The expression of CDX2 is highly specific for pulmonary adenocarcinoma.


B. The expression of TTF-1 is important in distinguishing primary bronchogenic
carcinoma from metastatic adenocarcinoma.
C. The expression of CK7− and CK20+ usually is associated with pulmonary
adenocarcinoma.
D. The expression of CK7+ and CK20− usually is associated with metastatic
adenocarcinoma of the colorectum.

Correct answer is B. RATIONALE: It is important to know some principles of lung cancer


pathology. CDX2 is highly specific for gastrointestinal malignancies, which could help
distinguish from primary lung adenocarcinoma. The majority of primary lung carcinomas are
positive for TTF-1, whereas metastatic adenocarcinoma to the lung is virtually always negative.
Primary lung adenocarcinoma is usually CK7+ and CK20-, therefore distinguishable from CK7-
and CK20+ metastatic colorectal adenocarcinoma.

224. Which of the following childhood CNS tumors has the worst prognosis?

A. Medulloblastoma
B. Pilocytic astrocytoma
C. Atypical teratoid rhabdoid tumor
D. Pure germinoma

Correct answer is C. RATIONALE: Atypical teratoid rhabdoid tumor carries the worst prognosis
among the tumors listed with a 1-year survival rate of 50%. The other tumors have a 5-year
survival rate in the range of 80-90%.

225. Under normal conditions, what are the dominant modes of DNA damage for γ-rays
versus α-particles?

A. Direct action for both γ-rays and α-particles


B. Direct action for γ-rays but indirect action for α-particles
C. Indirect action for γ-rays but direct action for α-particles
D. Indirect action for both γ-rays and α-particles

Correct answer is C. RATIONALE: The higher the LET of a radiation beam, the greater the
fraction of cellular damage produced by direct action. For low-LET radiation, about 70% of the
damage occurs through the indirect effect, barring cellular hypoxia or an unusually high
concentration of radical scavengers.
226. Which of the following factors is associated with an increased risk for the development of
breast cancer?

A. Scattered fibroglandular breast tissue density on mammography


B. History of unilateral clear nipple discharge on physical examination
C. Twin pregnancy before 30 years of age
D. Daily consumption of about 34 to 45 g of alcohol

Correct answer is D. RATIONALE: Mammographic density is an independent risk factor for


breast cancer. B1-RADS reporting is related to mammographic sensitivity and is not an estimate
of risk per se. A higher risk of subsequent beast cancer is associated with benign breast
findings of proliferation without atypia (1.3–1.9 RR) and atypical ductal hyperplasia (3.9–13 RR).
Unilateral clear nipple discharge is associated with ductal papillomas. Darker or blood-tinged
nipple discharge is more commonly associated with breast cancer. Earlier age at first pregnancy
and higher parity are associated with lower long-term risks for developing breast cancer. There
is no association of an increased risk of breast cancer with twin pregnancies. Consumption of
~34–45 g of alcohol per day is associated with a 1.3 RR for breast cancer. REFERENCE:
Harris, Lippman, Morrow, and Osborne. Diseases of the Breast. 3rd edition; 2004.

227. Which of the following statements about tumors of the temporal bone is true?

A. Paraganglioma is most common.


B. Cranial nerve VIII is most commonly involved when skin cancer invades into the
temporal bone.
C. Level 2 lymph node involvement is most common when tumors involve the external
auditory canal.
D. The most frequent symptoms of temporal bone paragangliomas are headaches and
facial nerve palsy.

Correct answer is A. RATIONALE: It is important to understand the spread pattern and


common presentation of tumors in this area. The most frequent symptoms associated with
temporal bone paragangliomas are hearing loss and tinnitus.

228. Which of the following chromosomal translocations may identify patients with alveolar
rhabdomyosarcoma associated with a very poor prognosis?

A. t(X;18) with SYT-SSX1 fusion transcript


B. t(2;13) with PAX3-FKHR fusion transcript
C. t(11;22) with EWS-FL1 fusion transcript
D. t(9;22) with bcr-abl fusion transcript

Correct answer is B. RATIONALE: The t(2;13) with the PAX3-FKHR fusion transcript identified
a group of patients in the IRS-IV study population who had a 4-year overall survival rate of
<10%. The PAX7-FKHR transcript identified a group of patients who had alveolar
rhabdomyosarcoma (RMS) and a better prognosis (i.e., a 4-year overall survival rate of >70%).
These translocations and fusion transcripts are not identified in embryonal RMS. Option A is
seen in synovial sarcoma. Option C is seen in Ewing's sarcoma, and option D is seen in chronic
myelogenous leukemia (CML).
229. Which of the following correction factors would be most important to apply when a dose
calibration is performed?

A. Incomplete ion collection efficiency (Pion)


B. Temperature pressure (PTP)
C. Electrometer calibration (Pelec)
D. Polarity effect (Ppol)

Correct answer is B. RATIONALE: Temperature-pressure correction factor (PTP) is dominating,


and typically on the order of 3%. Other correction factors are typically about 1% or less.

230. A randomized trial from the MRC Testicular Study Group compared irradiation to the
paraaortic and pelvic lymph nodes versus to the paraaortic lymph nodes alone. Which
of the following statements about this trial is true?

A. There was a statistically significant difference in overall survival.


B. There was a statistically significant difference in relapse-free survival.
C. There was no difference in the rate of azoospermia between the two groups.
D. The pelvic failure rate was less than 5% in patients receiving radiation therapy to the
paraaortic lymph nodes only.

Correct answer is D. RATIONALE: The pelvic failure rate in patients treated with paraaortic
radiation only was less than 2%.

231. Which of the following radiation treatments is most appropriate for a well-demarcated,
round, 5-mm basal cell carcinoma of the lower eyelid that is 2 mm thick?

A. 6-MeV electrons, no bolus, 1.5-cm circular field size


B. 6-MeV electrons, intraorbital tungsten shield, 50 Gy in 25 fractions of 2 Gy over
5 weeks
C. 100-kV x-rays, 0.5-cm bolus, 50 Gy in 25 fractions of 2 Gy over 5 weeks
D. 100-kV x-rays, intraorbital tungsten shield, 54 Gy in 18 fractions of 3 Gy over
3.6 weeks

Correct answer is D. RATIONALE: Fewer and fewer residents are receiving experience
treating skin tumors with 100 kV. Choice D works very well for the majority of skin lesions;
choice A would need a bolus; choice B would need a ceramic (not tungsten) shield and is overly
fractionated; choice C should not have a bolus and is overly fractionated.

232. Which of the following pairs of DNA damaging agents and resulting DNA lesions are
most responsible for cell death?

DNA Damaging Agent DNA Lesion


A. Paclitaxel Interstrand crosslinks
B. Ionizing radiation Single-strand breaks
C. Ultraviolet radiation Pyrimidine dimers
D. Etoposide Bulky adducts

Correct answer is C. RATIONALE: DNA double-strand breaks represent the most dangerous
type of DNA damage caused by ionizing radiation (and many chemotherapeutic drugs). A
thorough understanding of the type of damage caused by radiation and different
chemotherapeutic agents is important for the rational application of existing combination
therapies, and would be essential when testing novel molecular therapies that target DNA repair
processes in cancer cells.
233. According to the Lung Intergroup 0139 trial, the mortality rate was highest after which of
the following surgical procedures for stage III NSCLC?

A. Right-sided pneumonectomy
B. Left-sided pneumonectomy
C. Left lower lobectomy
D. Right upper lobectomy

Correct answer is A. RATIONALE: The mortality rate in patients receiving trimodality treatment
was 7.9%, and the mortality rate in patients receiving chemoradiation was 2.1%. Complex
pneumonectomy had a much higher mortality rate than that of simple pneumonectomy. The
mortality rate after right-sided complex pneumonectomy was 50%; the mortality rate after left-
sided complex pneumonectomy was 16%. The mortality rate for lobectomy was 1%, which
increased to 26% after pneumonectomy.

234. As determined by the planning optimization, IMRT plans are significantly more conformal
than 3D plans because the:

A. number of gantry angles used has decreased.


B. collimator is rotated.
C. wedges are inverted for several of the fields in IMRT.
D. beam fluence is nonuniform.

Correct answer is D. RATIONALE: IMRT uses nonuniform beam intensities across a radiation
field to tightly conform to a target of interest. There are several techniques available to achieve
this end goal.

235. Which of the following dose fractionation schemes is most appropriate for patients with
stage I seminoma?

A. 25 Gy in 20 fractions
B. 30 Gy in 10 fractions
C. 36 Gy in 20 fractions
D. 45 Gy in 25 fractions

Correct answer is A. RATIONALE: As discussed in Gunderson's text, 25 Gy in 20 fractions is


one of fractionation schemes used at Princess Margaret Hospital. Other fractionation schemes
described here are incorrect.

236. Which of the following statements about the treatment of paraganglioma is true?

A. A radiation dose of 45 Gy should offer a local control rate of >80%.


B. A complete response is common following external-beam radiation therapy.
C. Stereotactic radiosurgery is contraindicated.
D. Regional lymph nodes should be electively irradiated.

Correct answer is A. RATIONALE: It is important to understand the effect of treatment, the


appropriate dose, and volume for this very curable benign tumor. Local control rates with
external-beam radiation were reported ranging from 65% to 100% with a mean of 90%.
However, a complete response is uncommon. Most patients will have stable disease after
radiation therapy. Stereotactic radiosurgery has been used in the primary treatment of
paragangliomas with a dose ranging from 12 to 18 Gy. Lymph node metastasis is extremely
rare for paragangliomas.
237. Which of the following findings on MR spectroscopy is most commonly associated with a
low-grade astrocytoma?

A. Low choline
B. Low N-acetylaspartate
C. High creatine
D. High lactate

Correct answer is B. RATIONALE: Patients with low-grade astrocytoma typically have low N-
acetylaspartate (NAA), choline/creatine>1, and no lactate.

238. Based on an understanding of thermal dose and the effects of timing between heat and
radiation, which of the following protocols would be expected to produce the greatest
thermal enhancement ratio?

A. Heat prior to radiation


B. Heat following radiation
C. Heat alternating with radiation daily
D. Heat concurrent with radiation

Correct answer is D. RATIONALE: For the combination of heat and radiation, the greatest
thermal enhancement ratio (TER) occurs when the two agents are given simultaneously. For
moderately elevated temperatures that do not in and of themselves cause much heat-induced
cell killing, hyperthermia has been shown to interfere with the repair of radiation-induced DNA
damage and results in a reduction in the shoulder on the radiation survival curve when the two
agents are administered either simultaneously or within minutes of each other. This has also
been demonstrated in vivo in rodent tumors, where TER’s of nearly 5.0 have been achieved for
the combination treatment relative to radiation alone. Although heat can also sensitize normal
tissues to radiation injury, generally there is still a net therapeutic gain because normal tissues
are better equipped physiologically to dissipate heat relative to tumors. Therefore, the sustained
elevated temperatures needed to result in radiosensitization are less likely to occur.
REFERENCE: Hall and Giaccia. Radiobiology for the Radiologist. 6th edition, Chapter 28.

239. An increased risk of hepatocellular carcinoma is associated with exposure to:

A. hepatitis A.
B. Helicobacter pylori.
C. human papillomavirus.
D. aflatoxin B1.

Correct answer is D. RATIONALE: Aflatoxin B1 has been associated with an increased risk of
hepatocellular carcinoma (HCC). Hepatitis B and C (not hepatitis A) are associated with an
increased risk of HCC.
240. Which of the following AJCC stage groups is associated with a 2.5-cm squamous cell
carcinoma of the vulva with bilateral inguinal lymph node metastases and no evidence of
metastatic disease?

A. IIIA
B. IIIB
C. IVA
D. IVB

Correct answer is C. RATIONALE: T2N2M0 is Stage IVA. IVB represents distant metastatic
disease. REFERENCE: AJCC Cancer Staging Manual, 6th edition.

241. Which of the following factors is associated with the international prognostic score (IPS)
for Hodgkin lymphoma?

A. Extranodal site
B. Performance status
C. Erythrocyte sedimentation rate
D. Serum albumin level

Correct answer is D. RATIONALE: A serum albumin level of <4 g/dL is the only factor
belonging to the international prognostic score for Hodgkin lymphoma. REFERENCE:
HasenClever, et al. New England Journal of Medicine (NEJM). 1998;1506-14.

242. Which of the following thoracic radiation therapy regimens administered concurrently
with chemotherapy improves overall survival for limited-stage SCLC?

A. 45 Gy in twice-daily fractions of 1.5 Gy each


B. 48 Gy in twice-daily fractions of 1.5 Gy each, with a 2-week break after 24 Gy
C. 61.2 Gy in once-daily fractions of 1.8 Gy each for 5 weeks and twice-daily fractions
of 1.8 Gy each for the last 9 days
D. 70 Gy in once-daily fractions of 2 Gy each

Correct answer is A. RATIONALE: The radiation therapy regimen of 45 Gy bid is the only one
showing a survival benefit in a randomized Phase III trial. REFERENCE: Turrisi. Intergroup trial.
New England Journal of Medicine (NEJM). January 28, 1999;340(4):265-71.

243. For a pediatric patient receiving craniospinal irradiation, the advantage of using
electrons versus photons to treat the posterior spinal fields is to:

A. provide improved coverage of the spine.


B. minimize hot spots between the abutting cranial and spinal fields.
C. minimize the exit dose.
D. minimize the surface dose.

Correct answer is C. RATIONALE: Compared to photons, electrons are less penetrating. As a


result, the dose to structures anterior to the spine is considerably lower when electrons are used
for spinal irradiation.
244. The cervix drains primarily into which of the following lymph nodes?

A. Paracervical
B. Internal iliac
C. External iliac
D. Inguinal

Correct answer is A. RATIONALE: The uterine cervix has abundant lymphatic drainage
primarily into the paracervical lymph nodes; from there, drainage is to the external and internal
iliac nodes and subsequently to the common iliac and paraortic lymph nodes. Drainage of the
uterine fundus, in contrast, is to the external and internal iliac nodes as well as directly into the
paraortic lymph nodes.

245. A person receiving 8 to 12 Sv of acute radiation exposure most likely will die of which of
the following syndromes?

A. Prodromal
B. Hematopoietic
C. Gastrointestinal
D. Cerebrovascular

Correct answer is C. RATIONALE: The mode of death most associated with an acute
exposure in the 8-12 Sv range is the gastrointestinal syndrome, which is precipitated by
radiation-induced killing of "crypt cells" (stem cells of the gut lining). Death typically occurs
within a week of irradiation. The irradiated individual likely exhibited the prodromal syndrome as
well (not fatal per se) and would also have developed the hematopoietic syndrome within a few
weeks of irradiation; however, death from the GI syndrome would occur sooner. The dose range
specified was below the threshold for the development of the cerebrovascular syndrome,
however. REFERENCE: Hall and Giaccia. Radiobiology for the Radiologist. 6th edition, Chapter
8.

246. Which of the following complications most commonly occurs in patients receiving
fractionated radiation therapy for pituitary adenomas?

A. Vascular injury
B. Optic neuropathy
C. Radiation necrosis
D. Endocrine dysfunction

Correct answer is D. RATIONALE: The majority of patients with pituitary adenomas develop
one or many endocrine deficiencies after undergoing radiation therapy.

247. The dose rate at the isocenter of a megavoltage linear accelerator is 4 Gy/min. The
dose rate 1 cm from a typical HDR 192Ir source is about:

A. 1000 times greater.


B. 1000 times less.
C. the same.
D. the square root of 192.

Correct answer is C. RATIONALE: The activity of a typical high-dose-rate (HDR) 192Ir source is
5 to 10 Ci, and the dose rate is about 3 to 8 Gy/min.
248. Which of the following statements about preinvasive breast cancer findings is true?

A. The histology of invasive cancer is usually lobular in patients with LCIS.


B. LCIS is not associated with any clinical or mammographic abnormalities.
C. In the NSABP P1 trial, the use of tamoxifen for patients with LCIS did not reduce the
incidence of breast cancer.
D. There is a slightly higher rate of ipsilateral breast tumor recurrence when LCIS is
associated with primary invasive cancer.

Correct answer is B. RATIONALE: In the tamoxifen prevention trial, 18/411 women in the
placebo group and 8/415 women in the tamoxifen group had breast recurrences in the setting of
LCIS only. This resulted in a 56% reduction in risk. However, because the number of events
was so small, it did not reach statistical significance. When invasive cancer occurs in the setting
of LCIS, the histology is usually ductal. There is no difference in IBTR when LCIS is associated
with the primary invasive cancer. REFERENCES: Fischer.Tamoxifen for prevention of breast
cancer: report of the NSABP P-1 study. Journal of the National Cancer Institute. 1998;90:1371-
1388. Harris, Lippman, Morrow, and Osborne, Diseases of the Breast. 3rd edition, 2004;497-
505.

249. Which of the following pairs of tumor markers should be obtained to evaluate a patient
with a germ cell tumor of the ovary?

A. CA-125 and CA-130


B. CA-19-9 and CA-125
C. CEA and CA-15-3
D. AFP and β-hCG

Correct answer is D. RATIONALE: AFP and β-hCG are tumor markers in germ cell tumors of
the ovary. AFP will be elevated in tumors with yolk sac elements, and β-hCG will be elevated in
tumors with chorionic elements. CA-125 is commonly used as a marker for epithelial ovarian
cancers; however, CA-130 may be elevated in more than 90% of women with epithelial ovarian
cancers. CA-19-9 is often elevated in mucinous cancers of the ovary. CA-19-9 has been
combined with CA-125 as markers for borderline tumors of the ovary. CA-15-3 is elevated in
about one half of women with ovarian cancer, but it is not useful on its own. When CA-15-3 is
used in conjunction with the CA-125 assay, it increases the specificity in the differential
diagnosis of adnexal masses.

250. A "reciprocal dose plot" type of isoeffect curve is used to calculate a tissue's:

A. Tpot.
B. α/β ratio.
C. repair half-time.
D. growth fraction.

Correct answer is B. RATIONALE: The reciprocal dose plot technique pioneered by Douglas
and Fowler (in which the reciprocal of the total radiation dose to achieve an isoeffective level of
tissue response is plotted as a function of the dose per fraction) is used to determine a tissue’s
α/β ratio. REFERENCE: Douglas BC and Fowler JF. Radiation Research Journal. 1976;66:401-
426.
251. Which of the following types of non-Hodgkin lymphoma is most aggressive?

A. Mycosis fungoides
B. Mantle cell
C. Extranodal marginal zone B-cell
D. Grades I and II follicular

Correct answer is B. RATIONALE: Mantle cell lymphoma is mostly diagnosed as stage IV


disease and is known to have a moderately aggressive course with median survival around 3
years. Follicular, extranodal marginal zone, and mycosis fungoides all have an indolent disease
course. REFERENCE: DeVita.

252. What was the impact on the 4-year actuarial survival in patients receiving chemotherapy,
according to the Phase III (SWOG 8797, Peters) trial comparing postoperative pelvic
irradiation alone versus chemoradiation in high-risk patients with cervical cancer?

A. Worse
B. Comparable
C. Better
D. Survival was not reported.

Correct answer is C. RATIONALE: SWOG Trial 8797 compared adjuvant pelvic irradiation
versus adjuvant chemoradiotherapy in high-risk cervical cancer patients following radical
hysterectomy. High-risk features included pelvic lymph node involvement, positive surgical
margins, and microscopic parametrial involvement. Patients receiving pelvic irradiation plus
chemotherapy had an improved 4-year actuarial survival (81% vs 71%) compared to those
treated with adjuvant irradiation alone. REFERENCE: Peters, et al. Journal of Clinical Oncology.
2000;18:1606-1613.

253. Which of the following outcomes is associated with patients who have breast cancer that
has metastasized to multiple tissue sites with or without bone involvement?

A. Median survival is 4.8 years.


B. Overall survival is improved with chemotherapy.
C. Patients with ER-negative and PR-positive tumors have a 10% to 20% response
rate to endocrine therapy alone.
D. Patients with ER-positive and PR-positive tumors have a 50% to 75% response rate
to endocrine therapy alone.

Correct answer is D. RATIONALE: Median survival for patients with metastatic breast cancer
is 2½ to 3 years.

TUMOR RESPONSE RATE TO


ER PR ENDOCRINE THERAPY
Negative Negative <10%
Positive Negative 20%–30%
Negative Positive 30%–50%
Positive Positive 50%–75%

REFERENCE: Harris JR, Lippman ME, Morrow M, and Osborne CK. Treatment of metastatic
breast cancer. Diseases of the Breast. 3rd edition. 2004;1101-1159.
254. A 10 x 10-cm2 10-MV photon beam is prescribed to a point at a depth of 10 cm in the
mediastinum with no heterogeneity. Using heterogeneity dose corrections in which the
same beam traverses 6 cm of the lung, the dose at this point becomes:

A. 12% higher.
B. 06% higher.
C. 06% lower.
D. 12% lower.

Correct answer is A. RATIONALE: A simple calculation would use the fact that the 6 cm of
lung is equivalent to ~2 cm of tissue (deff = density lung x distance = 0.3 x 6 cm = 1.8 cm),
thereby 4 cm of increased transmission. Based on 3% per cm, a 12% higher dose would be
received.

255. Which of the following dose ranges of EBRT is most appropriate for an adult patient who
has undergone resection with negative surgical margins for soft tissue sarcoma of the
extremity?

A. 68 to 70 Gy
B. 60 to 66 Gy
C. 55 to 59 Gy
D. 45 to 50 Gy

Correct answer is B. RATIONALE: The most appropriate postoperative radiation dose is in the
range of 60 to 66 Gy.

256. Which of the following tests should be used in the analysis of a nominal independent
variable with a nominal dependent variable?

A. Chi-square
B. One-way ANOVA
C. Regression
D. t test

Correct answer is A. RATIONALE: For questions involving two variables


(independent/dependent), the appropriate method is as follows: Chi-square: nominal/nominal.
One-way ANOVA: nominal (more than two values)/numerical. Regression: numerical/numerical.
t test: nominal (binary)/numerical. Actuarial: nominal/numerical (censored).
257. Which of the following is NOT implicated in the mechanism of action of tirapazamine?

A. Generation of an oxidizing free radical


B. Generation of hydrogen peroxide
C. Interference with DNA replication
D. Poisoning of topoisomerase II

Correct answer: ABCD. RATIONALE: Tirapazamine is a heteroaromatic N-oxide that


undergoes a one-electron reduction in the absence of oxygen to form a short-lived oxidizing free
radical intermediate. This toxic species is thought to be responsible, directly or indirectly, for the
production of DNA double-strand breaks, interference with DNA replication and poisoning of
topoisomerase II, one or more of which can result in cell death. Tirapazamine is a substrate for
classic one-electron reductases such as cytochrome P450 and P450R, and NOS, nitric oxide
synthase. REFERENCES: Hall and Giaccia. Radiobiology for the Radiologist. 6th edition,
Chapter 25. Wardman P. Chemical radiosensitizers for use in radiotherapy. Clinical Oncology.
2007;19:397-417. McKeown SR, Cowen RL, and Williams KJ. Bioreductive drugs: From
concept to clinic. Clinical Oncology. 2007;19:427-442.

258. The RTOG 91-11 trial showed that concurrent chemoradiation:

A. increased severe late toxicity compared to radiation therapy alone.


B. increased local control by approximately 20% compared to radiation therapy alone.
C. had an equal larynx preservation rate compared to sequential chemoradiation.
D. had the same rate of mucositis compared to sequential chemoradiation.

Correct answer is B. RATIONALE: Concurrent chemoradiation is more effective than radiation


therapy (RT) alone in preserving the larynx (approximately 78% vs 56% at 2 years). It is
important for radiation oncologists to appreciate the relative magnitude of this improvement.
Also, this trial is unique in showing an effect of concurrent chemoradiation on distant
metastases.

259. Considering the typical percent depth dose curves for electron beams, the surface dose
for a 9-MeV beam is in what percentage range of the maximum absorbed dose, dmax?

A. 20% to 39%
B. 40% to 59%
C. 60% to 79%
D. 80% to 100%

Correct answer is D. RATIONALE: Unlike photons, skin sparing is modest at best. The typical
surface dose for megavoltage electron beams ranges from 80% to 100% of the dose at dmax.

260. Which of the following histologies is most commonly associated with primary CNS
lymphoma?

A. Mantle cell
B. Large B-cell
C. Anaplastic
D. Follicular

Correct answer is B. RATIONALE: Large B-cell lymphoma is the most common histology for
CNS lymphoma. REFERENCE: DeVita VT, Hellman S, Rosenberg SA. Cancer: Principles &
Practice of Oncology. 6th edition. Lippincott, Philadelphia; 2001;2300.
261. Overall survival in patients with locally advanced prostate cancer has been shown to be
increased by EBRT:

A. combined with a brachytherapy boost.


B. combined with hormone therapy.
C. administered to the whole pelvis.
D. administered via dose escalation.

Correct answer is B. RATIONALE: Combining hormone therapy with external-beam radiation


therapy (EBRT) is the only choice that has been shown to increase overall survival.

262. The interfraction interval is typically no less than 6 hours for radiation therapy delivered
in multiple dose fractions per day. The radiobiological reason for this treatment protocol
is to:

A. allow reoxygenation to occur in tumors.


B. cause tumor cells to undergo cell cycle redistribution into more radiosensitive
phases.
C. take maximum advantage of cellular repopulation in normal tissues.
D. take maximum advantage of repair of SLD in late-responding normal tissues.

Correct answer is D. RATIONALE: In several seminal hyperfractionation trials, most notably


the CHART trial for head and neck cancer, multiple doses per day delivered in less than 6-hour
intervals led to some unanticipated cases of radiation myelitis. This has been interpreted as
being a manifestation of the “incomplete repair” of sublethal damage. Late-responding normal
tissues have somewhat longer half-times of repair than early-responding tissues; therefore, they
may require a longer interfraction interval before this repair is complete. In fact, it has been
suggested by some that even 6 hours between fractions may be inadequate to take maximum
advantage of the sparing effect of sublethal damage repair in the spinal cord and brain.
REFERENCE: Dische, et al. Radiotherapy and Oncology: Journal of the European Society for
Therapeutic Radiology and Oncology. 1997;44:123-136.

263. What is the expected rate of CNS metastasis after systemic chemotherapy and PCI for
patients with extensive-stage SCLC?

A. 05%
B. 15%
C. 30%
D. 50%

Correct answer is B. RATIONALE: The incidence of brain metastases in patients who had
extensive-stage SCLC and received PCI was 15%. The incidence of brain metastases in
patients who had limited-stage SCLC and received PCI was approximately 20%.
264. What is the most common histology associated with malignant tumors of the ureters in
the United States?

A. Squamous cell carcinoma


B. Transitional cell carcinoma
C. Small cell carcinoma
D. Adenocarcinoma

Correct answer is B. RATIONALE: Transitional cell carcinoma is the most common histology
for ureteral carcinomas.

265. Which of the following doses of definitive radiation therapy is most appropriate for a
patient with unresectable Ewing's sarcoma of the pelvis?

A. 70.0 Gy
B. 64.8 Gy
C. 55.8 Gy
D. 50.4 Gy

Correct answer is C. RATIONALE: The recommended radiation dose for gross disease in a
patient with Ewing's sarcoma of the pelvis is 55.8 Gy.

266. Which of the following stages would be most appropriate for a patient who has Wilms
tumor with diffuse tumor spillage into the abdominal cavity?

A. I
B. II
C. III
D. IV

Correct answer is C. RATIONALE: This is the definition of stage III disease: Residual
intraabdominal tumor (nonhematogenous) exists after the completion of surgery. Lymph node
findings are positive, or peritoneal implants are found. The resected specimen has histologically
positive margins, or the tumor has spilled into the abdominal cavity.

267. How many mrem is equal to 1 mSv?

A. 000.1
B. 001.0
C. 010
D. 100

Correct answer is D. RATIONALE: There is a direct conversion from conventional units (mrem)
to SI units (mSv), where 100 rem = 1 Sv.
268. What is the number of degrees of freedom in a Chi-square test for the equality of
proportions from three independent populations?

A. 2
B. 3
C. 4
D. 5

Correct answer is A. RATIONALE: The number of degrees of freedom is the number of groups
minus 1.

269. Which of the following is a cellular feature of necrosis?

A. Cytoplasmic shrinkage
B. Chromatin condensation
C. Mitochondrial swelling
D. Membrane blebbing

Correct answer is C. RATIONALE: Mitochondrial swelling (along with overall cellular swelling,
culminating in the release of intracellular material into the surrounding microenvironment) is a
defining characteristic of a cell undergoing necrosis. Cytoplasmic shrinkage, chromatin
condensation, and membrane blebbing are distinguishing features of apoptosis (not necrosis).

270. According to the 1999 Intergroup (Turrisi) trial, accelerated hyperfractionated thoracic
radiation therapy beginning with the first cycle of chemotherapy significantly improved
5-year survival as compared with concurrent once-daily radiation therapy by:

A. 30%.
B. 20%.
C. 10%.
D. 05%.

Correct answer is C. RATIONALE: The survival rates for patients receiving once-daily radiation
therapy were 41% at 2 years and 16% at 5 years. REFERENCE: New England Journal of
Medicine. Jan 28, 1999;340(4):265-71.
271. According to a 2008 study by the University of Pennsylvania, the use of breast MRI for
the initial diagnosis and pretreatment evaluation of patients with early-stage breast
cancer demonstrated which of the following conclusions?

A. It has a greater positive predictive value than mammogram or ultrasound alone in


the selection of patients for partial breast irradiation.
B. It has a greater positive predictive value for detecting multicentric disease in patients
with invasive breast cancer than in those with noninvasive breast cancer.
C. It is not associated with an improvement in outcome after breast-conserving surgery
and radiation therapy.
D. It should be used for initial staging of women who have breast cancer associated
with BRCA1 and BRCA2 germline mutations.

Correct answer is C. RATIONALE: This was a retrospective study of patients treated by


breast-conserving surgery and radiation therapy. The primary endpoint was comparison of
outcomes between women who either had a breast MRI as part of their initial pretreatment
evaluation or did not. There were no differences in local control or survival outcomes. The study
did not generate data or make conclusions about the appropriateness of breast MRI in different
subgroups for selecting patients for breast-conserving surgery or partial breast irradiation.
REFERENCE: Solin, et al. Journal of Clinical Oncology. 2008;26:386-91.

272. Which of the following radiation doses and fractionation schemes would be most
appropriate for a 3-year-old child who has orbital embryonal rhabdomyosarcoma with
gross disease?

A. 36 Gy delivered in 1.8-Gy fractionation daily


B. 45 Gy delivered in 1.8-Gy fractionation daily
C. 50.4 Gy delivered in 1.8-Gy fractionation daily
D. 59.4 Gy delivered in 1.1-Gy fractionation twice daily

Correct answer is B. RATIONALE: The current recommendation is 45 Gy given in 25 fractions


of 1.8 Gy with a 5-year overall survival (OS) rate of 96%, and a 5-year failure-free survival (FFS)
rate of 89% based on IRS III data.

273. According to the NRC, the annual total effective dose limit to an individual member of
the general public is:

A. 00.5 mSv.
B. 01 mSv.
C. 10 mSv.
D. 50 mSv.

Correct answer is B. RATIONALE: According to 10 CFR 20.1301, the annual maximum dose
limit to a member of the general public is 1 mSv. Exceptions can be made for caregivers and/or
family members caring for a patient following a radioactive procedure. However, under these
circumstances; the maximum annual dose is limited to 5 mSv.
274. Which of the following statements concerning the oxygen enhancement ratio (OER) is
true?

A. It is higher when one or a few large radiation doses are used versus many
smaller-sized dose fractions.
B. It increases with increasing LET of the type of radiation used.
C. For tissues characterized by a partial pressure of oxygen of 30 mm Hg, the OER is
typically 2.5 to 3.0.
D. The "paired survival curve" method is used to determine the OER.

Correct answer: ABCD. RATIONALE: Calculated values for the OER are typically higher (2.5-
3.0) when a “severe” endpoint that involves a great deal of cell killing is used (such as might be
achieved when a single, large radiation dose is given) than when many smaller dose fractions
are used that produce less cell killing and a “milder” endpoint (OER ≈ 1.5-2.0). REFERENCE:
Hall and Giaccia. Radiobiology for the Radiologist. 6th edition, Chapter 6.

275. Which of the following statements about the prognosis of patients who have IBTR after
breast-conserving therapy is true?

A. Overall survival is not affected by the location of tumor recurrence in the breast.
B. Axillary lymph node status of the primary tumor does not affect overall survival.
C. Patients with BRCA1 or BRCA2 germline mutations have a worse prognosis than
patients without these mutations.
D. The histologic grade of the primary tumor affects the patient's prognosis.

Correct answer is D. RATIONALE: Overall survival is affected by the quadrant of recurrence,


the histologic grade, and lymph node (LN) status of the primary tumor. Factors associated with
the risk of distant metastases are the location of the recurrence, extent of local recurrence,
lymph node status of the primary tumor, and histologic grade of the primary tumor. Excellent
outcomes have been reported for patients with BRCA1/BRCA2 germline mutations after salvage
treatment for ipsilateral breast tumor recurrence (IBTR). REFERENCES: Voogd AC. Local
recurrence after breast-conserving therapy (BCT) for early stage-breast cancer detection,
treatment, and outcome. Dutch Study Group on local recurrence after breast conservation.
Cancer. 1999;89:437-446. Turner BC. BRCA1/BRCA2 Germline mutations in locally recurrent
breast cancer patients after lumpectomy and radiation therapy; implications for breast-
conserving management in patients with BRCA1/BRCA2 mutations. Journal of Clinical
Oncology. 1999;17:3017-3024. Pierce LJ. Effect of radiotherapy after breast-conserving
treatment in women with breast cancer and germline BRCA1/2 mutations. Journal of Clinical
Oncology. 2000;18:3360-3369.
276. What is the expected 5-year overall survival rate for a 5-year-old patient who has
average-risk medulloblastoma treated with craniospinal irradiation and chemotherapy?

A. 67%
B. 70%
C. 85%
D. 93%

Correct answer is C. RATIONALE: Children's cancer group study A9961 for average-risk
medulloblastoma reported a 5-year-old child’s overall survival (OS) rate to be 86%, while a 5-
year-old child's event-free survival (EFS) rate was reported to be 81%.

277. Which of the following toxicities is most common in patients who are receiving
concurrent chemoradiation for NSCLC?

A. Pneumonitis
B. Esophagitis
C. Pericarditis
D. Myelitis

Correct answer is B. RATIONALE: Esophagitis can be observed in 40% to 60% of patients


receiving concurrent chemoradiation therapy. Pneumonitis is much less common. Pericarditis
and myelitis are rare in the modern era of radiation therapy.

278. Which of the following statements about the MAGIC (Cunningham) study comparing
perioperative ECF chemotherapy to surgery alone for resected gastric cancer is true?

A. A lower rate of curative surgery was demonstrated in patients who underwent


perioperative chemotherapy.
B. A higher rate of postoperative complications was found in patients who underwent
perioperative chemotherapy.
C. The overall and progression-free survival rate was improved in patients who
underwent perioperative chemotherapy.
D. Most patients completed all 6 cycles of chemotherapy.

Correct answer is C. RATIONALE: The MAGIC study showed a higher rate of curative surgery
with chemotherapy, a similar postoperative complication rate, and improved overall survival
(OS) rate and progression-free survival (PFS) rate. However, only 41% of patients completed all
chemotherapy. REFERENCE: Cunningham, et al. New England Journal of Medicine (NEJM).
2006.

279. How can the peripheral dose to a pregnant patient be eliminated?

A. Increasing secondary jaw thickness


B. Using multileaf collimators and blocks simultaneously
C. Placing special blocking immediately next to the patient
D. The peripheral dose cannot be eliminated.

Correct answer is D. RATIONALE: Although peripheral dose may be reduced by placing


special blocking immediately next to the patient, peripheral dose cannot be completely
eliminated due to internal scatter.
280. Which of the following types of biopsy is most commonly used to diagnose Hodgkin
lymphoma?

A. Excisional
B. Incisional
C. Fine-needle aspirate
D. Random

Correct answer is A. RATIONALE: Excisional biopsy is the preferred method of biopsy since it
maintains architecture and sufficient material for immunohistochemistry flow cytometry.

281. Which of the following statements about a radiation-induced stochastic effect is true?

A. The severity of the effect increases with dose.


B. There is a threshold dose below which the risk of the effect is zero.
C. Radiation-induced cataracts are an example of a stochastic effect.
D. The probability of occurrence of the effect increases with dose.

Correct answer is D. RATIONALE: The definition of a stochastic radiation effect is one that is
probabilistic in nature and whose incidence increases with increasing dose, is “all or nothing”
(i.e., does not show degrees of severity), and is not characterized by a threshold dose below
which the likelihood of the effect is zero. Non-stochastic, also called deterministic, effects show
dose thresholds and graded levels of severity depending on dose. REFERENCE: Hall and
Giaccia. Radiobiology for the Radiologist. 6th edition, Chapters 10, 14, and 15.

282. Which of the following syndromes is associated with neuroblastoma?

A. Sick sinus syndrome


B. Restless legs syndrome
C. Opsomyoclonus-ataxia syndrome
D. SIADH secretion

Correct answer is C. RATIONALE: Opsomyoclonus-ataxia syndrome is seen in 1.3% of


patients who have neuroblastoma.

283. Which of the following statements about the management of thymoma is true?

A. Chemotherapy is not effective.


B. A complete thymectomy is preferred for resectable disease.
C. Adjuvant radiation therapy is indicated for patients with Masaoka stage I disease.
D. Preoperative radiation therapy is indicated for patients with Masaoka stage I
disease.

Correct answer is B. RATIONALE: For resectable disease, a complete thymectomy is


preferred because recurrences have been observed in the residual thymus after a partial
thymectomy. Several reports showed that 17% to 28% of patients who have thymomas develop
a second malignancy after thymectomy. Thus, close follow-up is needed given that secondary
malignancy is common in patients with thymomas.
284. Which of the following statements about the diagnosis of inflammatory breast cancer is
true?

A. Dermal lymphatic invasion is classified as stage T4d.


B. Dermal lymphatic invasion may be absent on a biopsy of the skin.
C. Since inflammatory carcinoma is a clinical diagnosis, a biopsy is not required.
D. Dermal involvement over more than one third of the breast is classified as
stage T4d.

Correct answer is B. REFERENCE: American Joint Committee on Cancer (AJCC) staging, 6th
edition.

285. According to the EORTC 22911 (Bolla) trial, adjuvant radiation therapy after a radical
prostatectomy is most beneficial for patients with:

A. a positive margin in the pathology specimen.


B. a Gleason score of 4+5 in the pathology specimen.
C. seminal vesicle involvement in the pathology specimen.
D. a PSA level of 0.4 ng/mL immediately after the prostatectomy.

Correct answer is A. RATIONALE: In the randomized controlled European Organization for


Research and Treatment of Cancer (EORTC) trial 22911, surgical margin status assessed by
review of the pathology specimen was the strongest predictor of prolonged biochemical
disease-free survival with immediate postoperative radiation therapy.

286. Neutrons may interact within the medium they are traversing by which of the following
mechanisms?

A. Bremsstrahlung
B. Photoelectric effect
C. Pair production
D. Knockout reaction

Correct answer is D. RATIONALE: Unlike photons and charged particles, neutrons interact
almost exclusively with the nucleus of an atom. Of the options listed, the only interaction
possible for neutrons is the knockout reaction. Photons may interact via the photoelectric effect
and pair production, and electrons passing near a nucleus may lose energy via the radiative
emission of bremsstrahlung x-rays.

287. Which of the following statements about the role of hypoxia in tumor progression is
FALSE?

A. Cyclic hypoxia followed by reoxygenation can induce genomic instability.


B. Low oxygen conditions increase the metastatic potential of tumor cells.
C. Hypoxia can select for tumor cells that retain apoptotic pathways.
D. Clinical studies have shown that tumor hypoxia has a negative impact on patient
outcomes, even in the absence of radiation therapy.

Correct answer is C. RATIONALE: Hypoxia has been shown to select for genomically unstable
tumor cells with mutations that result in diminished apoptotic potential. REFERENCE: Hall and
Giaccia. Radiobiology for the Radiologist, 6th edition, Chapter 6.
288. Which of the following factors best justifies the use of preoperative radiation therapy for
a patient with inflammatory breast cancer?

A. Skin-sparing mastectomy
B. Dermal lymphatic involvement
C. Matted axillary lymph nodes at presentation
D. Stable or progressive disease during neoadjuvant chemotherapy

Correct answer is D. RATIONALE: Surgery should not be performed in the presence of


inflammatory signs. Thus, radiation may be needed if there is either an incomplete response,
stable disease, or disease progression to induction chemotherapy. Matted axillary nodes or
dermal lymphatic invasion at presentation are not contraindications to surgery after a good
response to chemotherapy. Skin-sparing mastectomy or breast conservation should not be used
in patients with inflammatory breast cancer.

289. Which of the following management options would be most appropriate for low-grade
chondrosarcoma?

A. Curettage for primary treatment of affected extremity sites


B. Curettage and cryotherapy for primary treatment of affected axial and pelvic sites
C. Postoperative neutron radiation therapy after complete surgical resection
D. Postoperative photon radiation therapy of >65 Gy

Correct answer is D. RATIONALE: Options A and B (see references 1 and 4 below): Tumors
treated with curettage and cryotherapy have lower or comparable recurrence rates (6%) to en-
block resection (25%) at extremity sites. En-block resection recurrences (17%) are lower for
axial-pelvic sites when compared to curettage and cryotherapy (43%). Option C (see reference
2 below): Neutrons have been used for treatment of unresectable sarcomas. Option D (see
reference 3 below): Optimal treatment of chondrosarcoma requires doses of >65 Gy.

REFERENCES:
(1) Chow, W.A. Update on chondrosarcomas. Current Opinion in Oncology. 2007;19:371-
376.
(2) Gunderson & Tepper eds. Clinical Radiation Oncology. 2nd edition, 2007;379.
(3) Gunderson & Tepper eds. Clinical Radiation Oncology. 2nd edition, 2007;549- 552.
(4) Gelderblom H, Hogendoorn PCW, Dijkstra SD, et al. The clinical approach towards
chondrosarcoma. The Oncologist. 2008;13:320-329.
290. Which of the following statements about thyroid malignancies is true?

A. Papillary carcinoma spreads most commonly to level 2 lymph nodes.


B. Older patients with papillary carcinoma have a better prognosis than younger
patients.
C. Tumor size is the most important prognostic factor for a resectable follicular cancer.
D. Hürthle cell carcinoma has a worse prognosis than other well-differentiated
malignancies.

Correct answer is D. RATIONALE: The rationale for this question is to demonstrate the
importance of age and certain pathologies in the prognosis of patients with thyroid
malignancies. Hürthle cell cancer has a decreased avidity for 131I; therefore, treatment with
radioactive iodide has a limited efficacy. Hürthle cell cancer reportedly behaves in a more
aggressive fashion than other well-differentiated thyroid cancers, with a tendency to have a
higher incidence of metastasis and a lower survival rate. This is truer for the lesions that are
clearly demonstrated to be malignant and in patients who are considered to be at high risk
based on such factors as age, tumor size, invasiveness, and the presence of metastasis. Widely
invasive tumors behave more aggressively. Recurrence among patients with Hürthle cell
carcinoma is considered to be incurable.

291. A 32-year-old woman has clinical stage IIA Hodgkin lymphoma. Which of the following
doses of radiation therapy would be most appropriate if the patient achieves a complete
response after 6 cycles of ABVD chemotherapy?

A. 44 Gy
B. 36 Gy
C. 30.6 Gy
D. 20 Gy

Correct answer is C. RATIONALE: National Comprehensive Cancer Network guidelines


specify a radiation dose of 30 Gy for treatment of non-bulky clinical stage IIA Hodgkin
lymphoma.

292. In treating a superficial bone lesion, a radiation oncologist prescribes 40 Gy in 2 Gy daily


fractions. This prescription is an example of:

A. radiation exposure.
B. kinetic energy released in a medium (KERMA).
C. absorbed dose in a medium.
D. absorbed dose in air.

Correct answer is C. RATIONALE: This item requires knowledge of the definition of absorbed
dose in a medium (in this instance, bone is the medium).
293. The figure below shows two cell survival curves, one exponential and one shouldered.
To calculate the RBE, the dose corresponding to Point "X" should be compared to the
dose corresponding to Point:

A. I.
B. II.
C. III.
D. IV.

Correct answer is C. RATIONALE: RBE is defined as the ratio of doses of a standard versus
“test” type of radiation to yield the same biological endpoint (i.e., an isosurvival of cells in this
example). Accordingly: RBE = Dose corresponding to Point III / Dose corresponding to Point X.

294. Which of the following conditions is a contraindication for performing a radical resection
on a patient with adenocarcinoma of the pancreas?

A. Tumor invasion of the splenic artery


B. Tumor encasement of the superior mesenteric artery
C. Involvement of the peripancreatic lymph nodes
D. Extensive pancreatic intraepithelial neoplasia

Correct answer is B. RATIONALE: Encasement of the superior mesenteric artery (SMA) is a


contraindication of a Whipple procedure and is of major importance in pretreatment planning
(i.e., definitive chemoradiation versus surgery).

295. Which of the following subtypes of ependymoma has the lowest risk of recurrence after
gross total resection alone?

A. Thoracic spine, low grade


B. Supratentorial, anaplastic
C. Fourth ventricular, tanycytic
D. Filum terminale, myxopapillary

Correct answer is D. RATIONALE: The myxopapillary subtype of ependymoma is generally


more indolent as often occurs in the lumbosacral region. Tumors of the filum terminale usually
can be more completely resected and have a better prognosis than tumors of other sites.
296. Which of the following radionuclides has the longest half-life, based on its decay
constant (λ)?
103
A. Pd (λ= 0.041/day)
125
B. I (λ= 0.0117/day)
131
C. Cs (λ= 0.0714/day)
198
D. Au (λ= 0.257/day)

Correct answer is B. RATIONALE: This item requires knowledge that T1/2 is inversely
proportional to the decay constant (λ), or that the half-life T1/2 = 0.693/λ. Hence, radionuclide
125
I, which has the smallest transformation (decay) per day, exhibits the longest half-life.

297. Low-grade oligodendrogliomas most commonly are located in the:

A. cerebellum.
B. parietal lobe.
C. frontal lobe.
D. hippocampus.

Correct answer is C. RATIONALE: Low-grade oligodendrogliomas most commonly occur in


the frontal lobe.

298. Which of the following statements best explains the conventional dose rate effect?

A. As the dose rate decreases, PLD repair is stimulated.


B. As the dose rate increases, PLD repair is inhibited.
C. As the dose rate decreases, SLD repair occurs during irradiation.
D. As the dose rate increases, SLD repair is inhibited.

Correct answer is C. RATIONALE: The explanation for the conventional dose rate effect, the
observation of decreasing biological effectiveness (both in vitro and in vivo) of irradiation as the
dose rate is lowered, is that sublethal damage (SLD) repair occurs during radiation exposure
(which is longer the lower the dose rate for a given total dose). Further, it should be noted that
DNA repair enzymes are, in most cases, constitutively present and active in normal cells and
need only be “mobilized” in response to DNA damage. Thus, stating that a particular damaging
agent like ionizing radiation “stimulates” or “inhibits” repair is a bit simplistic and misleading.
REFERENCE: Hall and Giaccia. Radiobiology for the Radiologist. 6th edition. Chapter 5.

299. The addition of early thoracic radiation therapy to chemotherapy for patients with
limited-stage SCLC is expected to improve overall survival at 3 years by:

A. 00%.
B. 05%.
C. 10%.
D. 15%.

Correct answer is B. RATIONALE: In the meta-analysis of the thoracic radiation therapy trials,
the overall survival rate was improved by 5.4% at 3 years in patients who received radiation
therapy in addition to chemotherapy for limited-stage SCLC.
300. Which of the following pathological features is characteristic of nodular
lymphocyte-predominant Hodgkin lymphoma?

A. CD15−, CD30+, CD20+


B. CD15+, CD20+, CD22+
C. CD15+, CD30+, CD20−
D. CD20+, CD15−, CD30−

Correct answer is D. RATIONALE: Nodular lymphocyte-predominant Hodgkin lymphoma has


distinct pathological features characterized by the presence of CD20+, CD15-, and CD30- cells.
These features are different from classic Hodgkin lymphoma, which does not express CD20.

301. Which of the following diagnostic imaging time points for tumor assessment should be
used for EBRT target volume definition in a 3-year-old child with stage 4, high-risk
neuroblastoma?

A. Preinduction chemotherapy
B. Postinduction chemotherapy but preoperative
C. Postoperative
D. Post-PBSCT

Correct answer is B. RATIONALE: The postinduction, preoperative tumor volume allows


coverage of the sites of risk and sparing of normal tissues if anatomy was distorted by mass
effect of the tumor.

302. Which of the following types of ionization chamber is best for measuring surface dose?

A. Farmer
B. Parallel plate
C. Well chamber
D. Transmission chamber

Correct answer is B. RATIONALE: Parallel-plate ionization chambers (or extrapolation


chambers) are the most suitable chambers for surface dosimetry because of their geometry and
ability to measure dose in thin volumes close to the surface.

303. Which of the following histologies is most commonly associated with small bowel
cancer?

A. Carcinoid
B. Sarcoma
C. Lymphoma
D. Adenocarcinoma

Correct answer is D. RATIONALE: Adenocarcinoma — 45%; carcinoid — 29%; lymphoma —


16%; sarcoma — 10%. REFERENCE: Journal of National Cancer Institute. April
1987;78(4):653-6.
304. Patients with breast cancer are routinely tested for the activity of which of the following
oncogenes?

A. ABL
B. ERB
C. MYC
D. RAS

Correct answer is B. RATIONALE: Screening for the activity of Her-2, a member of the ERB
oncogene family, is routinely carried out in patients with newly diagnosed breast cancer.
Treatment with the molecularly targeted drug Herceptin is indicated only in patients who test
positive for overexpression of Her-2.

305. Randomized trials involving dose escalation of EBRT for prostate cancer have
consistently shown a significant difference in which of the following endpoints?

A. Distant-metastases-free survival
B. PSA-failure-free survival
C. Grade 2-3 rectal toxicity
D. Grade 2-3 urinary toxicity

Correct answer is B. RATIONALE: The randomized dose-escalation trials primarily have seen
a benefit in PSA control rates and not a benefit in the other endpoints.

306. Which of the following outcomes is associated with cisplatin-based adjuvant


chemotherapy for NSCLC?

A. Reduction in the risk of death by about 10%


B. Improved 5-year disease-free survival by about 10%
C. Absolute benefit of 10% in 5-year disease-free survival
D. Absolute benefit of 10% in 5-year overall survival

Correct answer is A. RATIONALE: According to the Lung Adjuvant Cisplatin Evaluation


(LACE) and meta-analysis, cisplatin-based adjuvant chemotherapy improves survival among
patients who have completely resected NSCLC with approximately 10% reduction in the risk of
death, with absolute benefits in 5-year disease-free and overall survival rates of 5%.
REFERENCES: Journal of Clinical Oncology (JCO). 2008;(26)3552-9. New England Journal of
Medicine (NEJM). 2004;350(4):315-60.

307. Which of the following statements about cholangiocarcinomas is true?

A. Surgery provides curative results in the majority of patients.


B. The majority of patients present with metastatic disease.
C. Proximal lesions have a worse prognosis than distal lesions.
D. Regional lymph nodes are rarely involved.

Correct answer is C. RATIONALE: Surgery, although having poor outcomes, is the mainstay of
treatment for cholangiocarcinoma. Distal lesions have better outcomes than proximal tumors.
308. Which of the following statements about the overview analysis by the Early Breast
Cancer Clinical Trialists' Cooperative Group (EBCTCG) on the benefit of adjuvant
irradiation for patients with breast cancer is true?

A. The 5-year locoregional recurrence rate is 42% for patients undergoing breast-
conserving surgery (BCS) alone.
B. For every four locoregional recurrences avoided at 5 years, one breast-cancer-
related death is avoided at 15 years.
C. The 15-year gain in the breast cancer mortality rate is 18% for patients undergoing
BCS and radiation therapy versus BCS alone.
D. When the 5-year locoregional recurrence rate is reduced by >10%, an improvement
in the mortality rate will emerge at 10 years.

Correct answer is B. RATIONALE: Option A: The 5-year locoregional recurrence rate for
breast-conserving therapy (BCT) alone was 32%. Option C: The 15-year gain in breast cancer
mortality for BCS + radiation therapy (RT) vs BCS alone was 5.4%. Option D: When there is a
greater than 10% reduction in the 5-year locoregional recurrence, the mortality benefit will
emerge at 5 years. REFERENCES: Lancet. 2005;365:1687-1717. Overview analysis, Early
Breast Cancer Clinical Trialists' Cooperative Group (EBCTCG). 2005.

309. Which of the following statements about a static wedge used with a 6-MV linear
accelerator is FALSE?

A. The wedge causes a spectral change in the beam.


B. The wedge transmission factor varies with the field size.
C. The wedge transmission factor varies with the depth.
D. The percent depth dose is the same with or without the wedge.

Correct answer is D. RATIONALE: The percent depth dose will change since there is beam
hardening for 6-MV photons due to the metal wedge.

310. Which of the following organs is most likely to experience dysfunction after whole-organ
irradiation?

A. Liver
B. Brain
C. Lung
D. Esophagus

Correct answer is C. RATIONALE: For total organ irradiation using conventionally sized daily
dose fractions, the total dose to cause an approximately 5% probability of severe, irreversible
lung damage within 5 years of treatment is approximately 18 Gy, far lower than the dose for the
other organs listed. In fact, the lung is among the most sensitive of the late-responding normal
tissues, with the critical factors determining tolerance being the total dose delivered, the fraction
size, and the volume irradiated. REFERENCES: Hall and Giaccia. Radiobiology for the
Radiologist. 6th edition, Chapter 19 (Table 19.2). Mettler and Upton. Medical Effects of Ionizing
Radiation. 3rd edition, Chapter 1 (Table 1-8) and Chapter 6.
311. What is the 5-year pelvic failure rate (ie, vagina, paravagina, inguinal and iliac lymph
nodes) of stage I carcinoma of the vagina treated with radiation therapy to 80 Gy alone?

A. <5%
B. 15%
C. 25%
D. 35%

Correct answer is B. RATIONALE: The largest series (MDACC) of 193 patients reports a 10%
vaginal failure rate and a 15% pelvic local failure rate. This is consistent with Mallinkrodt and
Stanford. REFERENCES: Frank, et al. International Journal of Radiation Oncology, Biology, and
Physics (IJROBP). 2005;62:1. Perez. International Journal of Radiation Oncology, Biology, and
Physics (IJROBP). November/December 2005;15(6):1283-90. Tran, et al. Gynecological
Oncology. June 2007;105(3):641.

312. Which of the following tumor markers is commonly elevated in patients with
hepatocellular carcinoma, but is rarely elevated in patients with cholangiocarcinoma?

A. CEA
B. CA 19-9
C. AFP
D. β-hCG

Correct answer is C. RATIONALE: α-Fetoprotein (AFP) is rarely positive in


cholangiocarcinoma but is found in 35% to 75% of hepatomas. This may help distinguish
between the two conditions during the initial workup.

313. Which of the following treatments achieves the best disease control in patients with
primary CNS lymphoma?

A. Corticosteroids
B. Whole-brain radiation therapy
C. Debulking of the tumor mass before administration of chemoradiation
D. Postsurgical chemotherapy only

Correct answer is B. RATIONALE: It is important to avoid use of corticosteroids prior to


establishing a histological diagnosis since they can cause a change in tumor appearance.
Surgical resection will not affect disease outcome because of its multifocal nature. For the same
reason, whole-brain irradiation is warranted for achieving disease control. Compared to all the
modalities, radiation therapy is associated with the worse neurotoxicity, particularly in elderly
patients.

314. Which of the following types of tumor is most likely associated with the EGFR V3
mutant?

A. Glioblastoma
B. Meningioma
C. Squamous cell carcinoma
D. Nasopharyngeal carcinoma

Correct answer is A. RATIONALE: The V3 mutant of EGFR is associated with glioblastoma


(GBM). The presence of this mutation suggests that not all EGFR-based therapies will be active
in GBM. EGFR V3 specific immunotherapies are in phase III clinical trials.
315. Which of the following statements about IMRT is true?

A. Dynamic IMRT is more conformal than step-and-shoot IMRT.


B. Image guidance is required to perform IMRT.
C. IMRT requires inverse treatment planning.
D. The first form of IMRT was helical tomotherapy.

Correct answer is C. RATIONALE: Because of the large numbers of treatment beamlets that
need to be optimized, a computerized inverse treatment planning is required for IMRT.

316. For radiation-induced normal tissue toxicity, the term TD50/5 refers to the:

A. total radiation dose for which there is a 50% probability of developing toxicity within
5 years of irradiation.
B. total radiation dose for which there is a 5% probability of developing toxicity within
5 years of irradiation.
C. probability of developing toxicity 5 years after 50 Gy of irradiation.
D. probability of developing toxicity 5 months after 50 Gy of irradiation.

Correct answer is A. RATIONALE: The definition of the term "TD50/5" (or "50% Tolerance Dose
at 5 Years") is the total radiation dose that would yield an approximately 50% probability of
causing an unacceptable normal tissue complication within 5 years of the completion of
radiotherapy. REFERENCE: Mettler and Upton. Medical Effects of Ionizing Radiation. 3rd
edition, Chapter 1 (Table 1-8).

317. Which of the following paraneoplastic syndromes are associated with SCLC?

A. SIADH, hypercalcemia, and Cushing syndrome


B. SIADH, limbic encephalitis, and Lambert-Eaton syndrome
C. Lambert-Eaton syndrome and hypercalcemia
D. Cushing syndrome and thrombocytosis

Correct answer is B. RATIONALE: Hypercalcemia is classically encountered in squamous cell


carcinoma, but not in SCLC. Thrombocytosis has not been described, although
thrombocytopenia may occur.

318. Compared with CT images, MR imaging has which of the following disadvantages for
the purpose of treatment planning?

A. Reduced soft tissue contrast


B. Inability to acquire functional information
C. Limited usefulness for treatment sites other than the brain
D. Susceptibility to spatial distortions

Correct answer is D. Of all the answers provided, option D is the only one that is correct.
Compared to CT, MR imaging provides improved soft tissue contrast, the ability to acquire
functional imaging information, and has proven to be quite useful in treatment sites beyond the
brain.
319. Which of the following phenotypic characteristics would be increased in cells that are
defective in the FEN1 endonuclease, which is known to be involved in base excision
repair (BER)?

A. Mutation rate
B. Sensitivity to UV irradiation
C. Sensitivity to alkylating agents
D. Sensitivity to ionizing radiation

Correct answer: ABCD. RATIONALE: UV, alkylating agents, and ionizing radiation generally
do not produce the types of DNA damage repaired by base excision repair, such that a defect in
the FEN1 endonuclease should not produce a particular sensitivity-associated phenotype. A
BER defect will lead to an overall increase in genomic instability and mutation rate, however.
REFERENCE: Henneke G, Friedrich-Heineken E, and Hubscher U. Flap endonuclease 1: A
novel tumor suppresser protein. Trends in Biochemical Sciences. 2003;28:384-390.

320. Which of the following factors has been found to affect the outcome of salvage therapy
in patients with relapsed Hodgkin lymphoma?

A. Performance status
B. Extranodal site of relapse
C. Lactate dehydrogenase (LDH) levels
D. Erythrocyte sedimentation rate (ESR)

Correct answer is A. RATIONALE: The following factors are found to affect the outcome of
patients with relapsed disease: duration of remission after initial therapy for <1 year, presence
or absence of "B" symptoms, Karnofsky performance status <90, age >40 years.

321. Which of the following viruses has been identified as a causative agent in the
development of cervical cancer?

A. Epstein-Barr virus
B. Human papillomavirus
C. Human T-lymphotropic virus
D. Hepatitis C virus

Correct answer is B. RATIONALE: Several viruses have been identified as causative agents in
different human tumors. The role of human papilloma viruses (HPV), particularly subtypes 16
and 18, has been established in cervical cancer and a HPV recombinant vaccine for the
prevention of cervical cancer has been approved by the U.S. Food and Drug Administration.
Epstein Barr Virus (EBV), Hepatitis C Virus (HCV) and the Human T-Lymphotropic Virus (HTLV)
have been linked with Burkitt's lymphoma, hepatocellular carcinoma, and T-cell leukemia,
respectively.
322. When matching a tangential breast field to a supraclavicular field, the collimator for the
tangential field must be rotated. The rotation of the collimator is a function of which of
the following two treatment parameters?

A. SSD and field size


B. SSD and gantry angle
C. Field size and gantry angle
D. Isocenter depth and gantry angle

Correct answer: ABCD. RATIONALE: To avoid an overlap between the supraclavicular and
tangential fields, collimator rotation must be utilized to match the inferior edge of the
supraclavicular field with the superior edge of the tangential fields. Since the beam edges are
matched on the patient’s skin surface, the collimator rotation will be independent of the
isocenter depth, and SSD and will be dependent on the field size and the gantry angles used for
the treatment fields.

323. In response to radiation-induced DNA damage, p53 increases the expression of which of
the following gene products?

A. Cyclin A
B. Cyclin E
C. 53BP1
D. p21

Correct answer is D. RATIONALE: p53 is the protein product of the crucially important tumor
suppressor gene TP53 and functions as a transcriptional activator. DNA damage-induced G1
arrest is governed by p53-mediated transactivation of p21, which inhibits G1 cyclins and
therefore prevents movement of cells from G1 into S phase. REFERENCE: Tannock, Hill,
Bristow, and Harrington. The Basic Science of Oncology. 4th edition, Chapter 7.

324. A 54-year-old man who underwent TURBT and intravesicular BCG for superficial
bladder cancer 3 years ago now presents with painless hematuria. A repeat cystoscopy
is normal. What is the next most appropriate test?

A. Ultrasound of the bladder


B. Ultrasound of the prostate
C. Intravenous pyelogram
D. Serum PSA level

Correct answer is C. RATIONALE: Patients with a history of prior superficial bladder cancers
may be at increased risk for other tumors of the upper tracts. Given the choices, intravenous
pyelogram (IVP) is the most appropriate next step given a negative cystoscopy.

325. Given the same field size at the prescription depth, which of the following factors is
different for an isocentric setup and an SSD setup with SSD = 120 cm?

A. Linac calibration factor


B. Tissue-maximum ratio (TMR)
C. Phantom scatter factor (Sp)
D. Collimator scatter factor (Sc)

Correct answer is C. RATIONALE: The effective field size increase at the surface will affect the
scatter linearly. The other factors are independent.
326. Which of the following risk factors has NOT been shown to be associated with the
development of anal cancer?

A. HIV infection
B. HPV infection
C. HSV infection
D. Multiple sexual partners

Correct answer is C. REFERENCE: Ryan DP, Compton CC, Mayer RJ. Carcinoma of the anal
canal. New England Journal of Medicine (NEJM). March 16, 2000;342(11):792-800.
327. When inactivated, which of the following genes abolishes both the cytotoxic and
radiosensitizing effects of gemcitabine?

A. ATM
B. MLH1
C. BCL-2
D. TP53

Correct answer is B. RATIONALE: Gemcitabine is a pyrimidine analog that functions as an


antitumor agent and potent radiosensitizer through depletion of the deoxynucleoside
triphosphate pool and subsequent incorporation into DNA, resulting in inhibition of DNA
synthesis and repair. A functional mismatch repair (MMR) system is critical for gemcitabine to
be active. Defects in MMR, in particular the loss of MLH1, abolishes both the cytotoxicity and
radiosensitizing effect of gemcitabine.REFERENCE:

328. According to the GOG 99 (Keys) adjuvant endometrial trial, what percent of patients
undergoing pelvic irradiation had grade 3 and 4 gastrointestinal toxicities?

A. 00%
B. 04%
C. 08%
D. 16%

Correct answer is C. RATIONALE: This question demonstrates that pelvic radiation can have a
high morbidity rate (8% with severe GI toxicities) if not administered judiciously. REFERENCE:
Keys, et al. Gynecological Oncology. March 2004;92(3):744-51.

329. Which of the following findings is most common at diagnosis of patients with renal cell
cancer?

A. Approximately 75% of patients have metastases to the lung.


B. Approximately 40% of patients have lymph node involvement.
C. The bone is the most common site of metastasis.
D. The renal vein is invaded in 5% of cases.

Correct answer: ABCD. RATIONALE: Approximately 75% of patients with metastatic renal cell
cancer (RCC) have metastases to the lung, 36% to the soft tissues, 20% to the bone, 18% to
the liver, 8% to the cutaneous sites, and 8% to the central nervous system. Lymph node
metastases occur with an incidence of 9% to 27% and most often involve the renal hilar,
paraaortic, and paracaval lymph nodes. The renal vein is invaded in 21% of cases.
330. Which of the following conditions most commonly is found in patients with Langerhans
cell histiocytosis?

A. Diabetes insipidus
B. Hyperthyroidism
C. SIADH secretion
D. Precocious puberty

Correct answer is A. RATIONALE: Diabetes insipidus is seen in patients with Langerhans cell
histiocytosis (LCH) because of histiocytic infiltration of the hypothalamus and pituitary gland.

331. The Bcl-2 gene is a:

A. tumor suppressor.
B. tyrosine kinase inhibitor.
C. proto-oncogene.
D. initiator.

Correct answer is C. RATIONALE: Bcl-2 is a proto-oncogene that is responsible for


transcriptional deregulation. It negatively affects normal apoptosis in lymphoid malignancies,
especially follicular lymphoma. REFERENCE: Devita. p. 2219.

332. Which of the following prognostic factors is most important in determining the risk for
distant metastases and the overall survival rate for adults with soft tissue sarcoma?

A. Tumor size
B. Tumor grade
C. Histologic subtype
D. Performance status

Correct answer is B. RATIONALE: Tumor grade has the most important influence on outcome.

333. Which of the following functions is NOT served by cell cycle checkpoints?

A. Preventing cells from progressing through the cell cycle with residual DNA damage
B. Identifying cells with irreparable DNA damage and targeting them for elimination
C. Increasing DNA repair rates
D. Helping maintain genomic stability

Correct answer is C. RATIONALE: Cell cycle checkpoints induced by DNA-damaging agents,


such as ionizing radiation, temporarily halt cell cycle progression to allow DNA repair processes
to occur. The function of cell cycle checkpoints is to prevent genetic instability and increase cell
survival by preventing cell propagation in the presence of residual DNA damage. Proteins
involved in cell cycle checkpoint control are attractive targets for the development of novel
anticancer drugs.
334. Which of the following ionization chambers is used as the primary standard in the
measurement of Roentgens (R)?

A. Free-air
B. Thimble
C. Parallel plate
D. Extrapolation

Correct answer is A. RATIONALE: While all of the above are ionization chambers, the free-air
chamber is a very large chamber in comparison. According to the definition of roentgen, the
ionized electrons produced by photon interactions in the gas medium of the chamber volume
must be collected within the chamber volume. For small-volume chambers (options B-C), this is
not feasible.

335. What is the risk of recurrence in the paraaortic or pelvic lymph nodes after orchiectomy
alone for stage I pure seminoma?

A. <5%
B. 20%
C. 35%
D. 50%

Correct answer is B. RATIONALE: From multiple surveillance studies, the risk of recurrence
for patients with stage I seminoma is between 15% to 20%. REFERENCE: Princess Margaret,
Royal Marsden study.

336. What is the most common histology in women with urethral carcinoma?

A. Transitional cell carcinoma


B. Squamous cell carcinoma
C. Adenosquamous carcinoma
D. Adenocarcinoma

Correct answer is B. RATIONALE: The most common tumor in women with urethral carcinoma
is squamous cell carcinoma, comprising more than 50% of patients. The next most common
histologies in order of frequency are transitional cell and adenocarcinoma.

337. IMRT does NOT decrease the incidence of which of the following complications of
nasopharyngeal cancer?

A. Dysphagia
B. Xerostomia
C. Hearing loss
D. Osteoradionecrosis

Correct answer is D. RATIONALE: Dose constraints have been established for salivary gland,
cochlea, and constrictor muscles.
338. Which of the following statements about genetic breast cancer predispositions is true?

A. Patients with a BRCA2 germline mutation have a higher risk of colon cancer than
patients with a BRCA1 germline mutation.
B. BRCA1 and BRCA2 germline mutations have an autosomal-dominant inheritance
pattern.
C. Peutz-Jegher syndrome is associated with a strong family history of follicular thyroid
cancer.
D. Brain and soft tissue malignancies are associated with a BRCA1 germline mutation.

Correct answer is B. RATIONALE: BRCA1 is associated with breast and ovarian cancers.
BRCA2 is associated with breast, ovarian, prostate, and pancreatic cancers. Li Fraumeni
syndrome is associated with breast, brain, soft tissue sarcoma, osteosarcoma, leukemia, and
adrenocortical carcinomas. Cowden's disease is associated with breast, ovarian, follicular
thyroid, and colon cancers. Peutz-Jegher syndrome is associated with GI and breast cancers.
REFERENCE: Harris, Lippman, Morrow, and Osborne. Diseases of the Breast. 3rd edition.
2004;279.

339. In a radiation survival curve experiment using a human tumor cell line, 300 cells were
seeded into Petri dishes and cultured for 14 days before being stained. There were
150 resulting colonies. In a second set of Petri dishes, 1,000 cells were seeded and
irradiated with 4-Gy x-rays. Fourteen days later, there were 180 resulting colonies.
What was the surviving fraction after 4-Gy x-ray irradiation?

A. 0.090
B. 0.036
C. 0.18
D. 0.36

Correct answer is D. RATIONALE: The normalized cell surviving fraction in a radiation survival
curve experiment is calculated from the absolute surviving fraction after a given dose of
radiation, divided by the plating efficiency (i.e., the absolute surviving fraction obtained from a
cohort of cells prepared during the same experiment and under the same experimental
conditions, but not exposed to radiation). Therefore, for a dose of 4 Gy in this case: SF =
(180/1,000) ÷ (150/300) = 0.18/0.50 = 0.36.

340. Which of the following histologies is most commonly associated with primary vaginal
cancer in women who are younger than 20 years of age?

A. Sarcoma
B. Melanoma
C. Squamous cell carcinoma
D. Adenocarcinoma

Correct answer is D. RATIONALE: Adenocarcinoma is the second most common histology


(after squamous cell) and represents nearly all patients (86%) younger than 20 years of age.
Sarcoma, particularly pediatric sarcomas, was a smaller group. This study covers the years of
1985-1994. REFERENCE: Creasman WT, et al. National cancer data basic report on cancer of
the vagina. Cancer. September 1, 1998; 83(5):1033-40.
341. Which of the following IMRT features is theoretically associated with an increased risk
for secondary malignancies?

A. It increases the whole-body dose.


B. It does not provide effective sparing of critical organs.
C. It is performed with smaller PTV margins.
D. It uses fewer monitor units than conventional radiation therapy.

Correct answer is A. RATIONALE: Because of the increased treatment times and significant
leakage through the MLC leaves, the dose to normal tissue is significantly increased with IMRT
compared to non-IMRT therapy. As a result, this has raised concerns about the potential for
increased incidence of secondary malignancies.

342. Which of the following radiation doses to metastatic sites is recommended after PBSCT
in a 3-year-old child who has high-risk neuroblastoma with multiple bony metastases on
131
I-metaiodobenzylguanidine (MIBG) scan at diagnosis?

A. 21.6 Gy to all initial sites


B. 45 Gy to all initial sites
C. 21.6 Gy to persistent sites on MIBG scan after induction chemotherapy
D. 45 Gy to persistent sites on MIBG scan after induction chemotherapy

Correct answer is C. RATIONALE: Current treatment regimens recommend 20-25 Gy to


metastatic sites, which are resistant to the initial chemotherapy.

343. Which of the following outcomes was demonstrated in the ECOG E1482 (Horning) trial,
which randomized involved-field radiation therapy (IFRT) versus observation after
8 cycles of CHOP in patients with high- and intermediate-risk stage I or stage II
non-Hodgkin lymphoma?

A. Time to progression was the same in both study groups.


B. Overall survival improved in patients who received IFRT after having a complete
response to chemotherapy.
C. Six-year disease-free survival and failure-free survival improved in patients who
received IFRT after having a complete response to chemotherapy.
D. Failure-free survival improved in patients who received IFRT after having a partial
response to chemotherapy.

Correct answer is C. RATIONALE: Patients with a complete response had improved 6-year
disease-free survival (DFS), failure-free survival (FFS), and time to progression in the IFRT arm
over the observation arm, but there was no difference in overall survival (OS). Patients
achieving a complete response (CR) after 8 cycles of CHOP were treated to 30 Gy of IFRT,
while patients with a partial response received 40 Gy of IFRT. Conversion to CR did not
significantly influence clinical outcome of patients with initial PR from chemotherapy.
REFERENCE: Horning, et al. Journal of Clinical Oncology (JCO). 2004;22(15).
344. According to the GITSG 9173 trial, which of the following outcomes was associated with
patients who had resected pancreatic cancer?

A. The median overall survival improved from 11 months to 20 months for patients in
the chemoradiation arm.
B. The number of distant metastasis was decreased for patients in the chemoradiation
arm versus the observation arm after surgery.
C. The same rate of survival was found in patients with positive margins as in those
with negative margins.
D. A radiation dose of 60 Gy was superior to 40 Gy.

Correct answer is A. RATIONALE: This trial established the precedent of treating resected
pancreatic cancer with chemoradiation.

345. What is the 5-year local recurrence rate for a completely resected grade I meningioma?

A. 01%
B. 10%
C. 20%
D. 30%

Correct answer is B. RATIONALE: The recurrence rate for a completely resected grade I
meningioma at 5 years is 10% (1% to 2% per year).

346. Which of the following assays has been used to measure the radiation sensitivity of
bone marrow stem cells in vivo?

A. Fat pad
B. Lung colony
C. Spleen colony
D. Endpoint dilution

Correct answer is C. RATIONALE: Clonogenic assays measure cellular reproductive integrity,


and all have as their endpoint the ability of single cells to form colonies containing at least 50
cells (corresponding to a minimum of approximately 5 successful rounds of cell division).
Clonogenic assays can be conducted both in vitro and in vivo (providing a particular normal
tissue is amenable to substituting for a Petri dish as a vessel allowing colony growth) in
laboratory rodents. The fat pad, lung colony, and endpoint dilution assays are used to assess
the radiosensitivity of tumor cells, whereas the spleen colony assay measures the radiation
sensitivity of normal bone marrow stem cells.

347. Which of the following mediums is most commonly used in the United States to report
the absorbed dose for patients who are NOT on an RTOG protocol?

A. Water
B. Tissue
C. Bone
D. Fat

Correct answer is A. RATIONALE: This item requires knowledge of the definition of absorbed
dose to a medium. Water is the medium of choice, but absorbed doses for patients on an RTOG
protocol are reported as absorbed dose to tissue.
348. Which of the following tests describes a regression method used when the outcome is
censored and the regression coefficients are interpreted as adjusted relative risk or odds
ratios?

A. Log-linear
B. ANCOVA
C. Mantel-Haenszel
D. Cox proportional hazards model

Correct answer is D. RATIONALE: Cox proportional hazards model is a regression method


used when the outcome is censored. The regression coefficients are interpreted as adjusted
relative risk or odds ratio. Log-linear analyzes the relationships among three or more nominal
variables (not censored) and may be used as a regression method to predict a nominal outcome
from nominal independent variables. ANCOVA (analysis of covariance) is a special type of
analysis of variance or regression used to control for effect of a possible confounding factor.
Mantel-Haenszel test is a statistical test of two or more 2 x 2 tables used to compare survival
distributions or to control for confounding factors.

349. Which of the following histologic findings is most characteristic of Ewing's sarcoma
tumor cells?

A. Vimentin staining is usually positive.


B. Cytokeratin staining is usually positive.
C. Cells are large with small hypochromatic nuclei.
D. Cells are in Homer-Wright rosette pattern.

Correct answer is A. RATIONALE: The tumor cells are small with hyperchromatic nuclei,
typically in a pseudorosette pattern. The cells undergo frequent mitosis. Vimentin staining is
uniformly positive, and cytokeratin staining is usually negative.

350. Based on a meta-analysis of randomized breast cancer prospective trials published in


2005 (Mauri), which of the following outcomes is associated with neoadjuvant
chemotherapy compared to adjuvant chemotherapy for patients who received
breast-conserving therapy?

A. The locoregional recurrence rate is increased.


B. The distant recurrence rate is reduced, but the overall survival is improved.
C. There is a lower incidence of involved margins after breast-conserving surgery.
D. Tumors that have a complete clinical response to neoadjuvant chemotherapy and
preoperative irradiation do not require surgical resection.

Correct answer: ABCD. RATIONALE: The meta-analysis showed an increased locoregional


recurrence rate with neoadjuvant chemotherapy, particularly when radiation was given without
surgery. There was no data on margins presented. There was no difference in distant
recurrence or survival. REFERENCE: Mauri, et al. Journal of National Cancer Institute.
2005;97:188-94.
351. Bremsstrahlung radiation is defined as a process in which an electron:

A. passing near a nucleus is accelerated and deviates from its straight-line path
(ie, called "braking" radiation).
B. passing near a nucleus is accelerated but maintains its straight-line path
(ie, called "speeding" radiation).
C. passing near a nucleus is annihilated and produces an electron-positron pair
(ie, called "annihilation" radiation).
D. directly strikes a nucleus, and the electron is annihilated (ie, called "collision"
radiation).

Correct answer is A. RATIONALE: It requires an understanding of the definition of


bremsstrahlung radiation.

352. Which of the following serum levels is LEAST likely to be elevated in a patient with pure
seminoma?

A. Lactate dehydrogenase
B. β-human chorionic gonadotropin
C. α-Fetoprotein
D. Calcium

Correct answer is C. RATIONALE: In pure seminoma, α-fetoprotein (AFP) serum levels should
not be elevated.

353. What is the primary reason that an 8 Gy x-ray dose delivered at 1 Gy/h is less toxic than
the same dose delivered at 1 Gy/min?

A. Fewer free radicals are generated.


B. Free-radical scavenging occurs.
C. SLD repair occurs during irradiation.
D. Cell division occurs during irradiation.

Correct answer is C. RATIONALE: A given total dose delivered at a high dose rate is more
toxic than the same dose delivered at a low dose rate because of the conventional dose rate
effect. This results because sublethal damage repair occurs during the radiation exposure,
which, for a given total dose, takes longer the lower the dose rate.

354. According to the prospective Phase III (Landoni) trial for stage IB-IIA cervical cancer, the
severe treatment morbidity rate with definitive radiation therapy versus radical surgery
was:

A. lower.
B. comparable.
C. higher.
D. not analyzed.

Correct answer is A. RATIONALE: Patients receiving irradiation had a statistically significant


lower rate of severe sequelae (12 vs 28%, P < 0.0004) than women randomized to receive
radical surgery. The markedly higher rate of severe toxicity in the surgical group may be due, in
part, to the addition of adjuvant pelvic irradiation during surgery for patients with adverse
prognostic features. REFERENCE: Landoni, et al. Lancet. 1997;350:535-540 (which
randomized 343 patients with clinical stage IB-IIA cervical cancer to receive definitive irradiation
versus radical surgery).
355. During inverse treatment planning, an end-of-planning correction accounts for:

A. dynamic arc delivery.


B. patient setup and intra-fraction motion.
C. leaf motion and transmission.
D. dose calculation uncertainties.

Correct answer is C. RATIONALE: During inverse treatment planning, beamlets used in


optimization are typically considered "ideal." Practical delivery aspects, particularly leaf motion
and transmission, are accounted at the end of optimization for the plan. The residual errors are
typically small.

356. According to the German Rectal Cancer Study Group (Sauer) trial, patients who
received preoperative chemoradiation experienced which of the following outcomes in
comparison to those treated with postoperative chemoradiation?

A. Increased disease-free survival


B. Increased acute toxicity
C. Decreased local recurrence
D. Decreased overall survival

Correct answer is C. RATIONALE: In the German trial, patients treated with preoperative
chemoradiation had lower rates of locoregional recurrence and acute toxicity, with no difference
in disease-free survival rates, compared to those treated with postoperative chemoradiation.
REFERENCE: Sauer, et al. New England Journal of Medicine (NEJM), 2004;351:1731-40.
357. What is the most likely site of origin for an "unknown primary" head and neck tumor
associated with the human papillomavirus?

A. Oral cavity
B. Oropharynx
C. Nasopharynx
D. Hypopharynx

Correct answer is B. RATIONALE: There is emerging data suggesting that unknown primaries
associated with the human papillomavirus (HPV) nearly always arise in the oropharynx.

358. Rituximab binds to which of the following CD receptors?

A. CD20
B. CD15
C. CD8
D. CD4

Correct answer is A. RATIONALE: Rituximab is a chimeric monoclonal antibody that binds to


the CD20 receptor expressed on B-cells to induce apoptosis. REFERENCE: FDA package
insert.
359. Which of the following subtypes of rhabdomyosarcoma has the worst prognosis?

A. Botryoid
B. Alveolar
C. Embryonal
D. Spindle cell

Correct answer is B. RATIONALE: Alveolar rhabdomyosarcoma subtype, being younger than


1 year of age or older than 10 years of age, primary site, and higher stage or group are all
independent prognostic factors for worse 5-year disease-free survival and overall survival
outcomes.

360. Why is patient positioning potentially more critical in the CT simulator than in the
treatment rooms?

A. The CT simulator has a higher imaging resolution.


B. Patient alignment errors in CT simulators translate into systematic treatment errors.
C. CT simulators cannot provide volumetric and radiographic information.
D. CT simulators are staffed at a lower level.

Correct answer is B. RATIONALE: If a patient is misaligned during CT simulation, this error


can easily translate into a systematic treatment error (the patient can be consistently misaligned
for treatment; therefore, option B is correct). The other answers provided are all false. Although
a CT simulator may have a higher resolution than current electronic portal imagers, the imaging
resolution of film is higher than that of a CT scan for institutions using verification film. In
addition, a scan simulator can provide both a volumetric CT scan of the patient and two
dimensional scout views, which are radiographic type images. Lastly, CT simulators are typically
staffed at the same level as treatment units. Although there are instances when one therapist
may be staffing the simulator, this is not typically the norm. Furthermore, although low staffing
could potentially lead to errors such as positioning, such errors cannot always be associated
with staffing levels.

Вам также может понравиться